Location via proxy:   [ UP ]  
[Report a bug]   [Manage cookies]                

Mikro Quize

You are on page 1of 95

1.

Which one of the following terms (D) Prions (A) Archaebacteria


characterizes the interaction between a (E) Plasmids (B) Protozoa
fungus and algae in a lichen? (C) Viruses
5. A 65-year-old man develops dementia,
(A) Parasitism (D) Prions
progressive over several months, along
(B) Symbiosis (E) Fungi
with ataxia and somnolence. An
(C) Endosymbiosis
electroencephalographic pattern shows 9. Quorum sensing in prokaryotes involves
(D) Endoparasitism
paroxysms with high voltages and slow
(E) Consortia (A) Cell–cell communication
waves, suggestive of Creutzfeldt-Jakob
(B) Production of pheromones
disease. This disease is caused by which of
2. Which one of the following agents lacks (C) An example of multicellular behavior
the following agents?
nucleic acid? (D) Regulation of genes involved in diverse
(A) Bacteria (A) Bacterium physiologic processes
(B) Viruses (B) Virus (E) All of the above
(C) Viroid (C) Viroid
(D) Prions 10. Twenty minutes after ingesting a raw
(D) Prion
(E) Protozoa clam, a 35-year-old man experiences
(E) Plasmid
paresthesias of the mouth and
3. Which one of the following is not a 6. Which of the following cannot be extremities, headache, and ataxia. These
protist? infected by viruses? symptoms are the result of a neurotoxin
produced by algae called
(A) Bacteria (A) Bacteria
(B) Algae (B) Protozoa (A) Amoeba
(C) Protozoa (C) Human cells (B) Blue-green algae
(D) Fungi (D) Viruses (C) Dinoflagellates
(E) Slime molds (E) None of the above (D) Kelp
(E) None of the above
4. Which one of the following agents 7. Viruses, bacteria, and protists are
simultaneously contains both DNA and uniquely characterized by their respective 1.A 22-year-old man presents with a
RNA? size. True or false? painless 1-cm ulcer on the shaft of his
penis. Inguinal lymphadenopathy is
(A) Bacteria (A) True
present. The patient admits trading drugs
(B) Viruses (B) False
for sex and has several sexual partners. An
(C) Viroids
8. Which of the following are prokaryotes? RPR test result is positive, and syphilis is
suspected; however, a Gram stain of a 4. Which of the following transport heat resistance of bacterial spores, such as
swab specimen from the ulcer shows no mechanisms functions without the those of Bacillus anthracis, is partly
bacteria. Treponema pallidum, the requirement for energy? attributable to their dehydrated state and
causative agent of syphilis, cannot be partly to the presence of large amounts of
(A) Binding protein dependent
visualized by light microscopy because
(B) Group translocation (A) Diaminopimelic acid
(A) It is transparent. (C) Symport (B) d-Glutamic acid
(B) It cannot be stained by ordinary stains. (D) Uniport (C) Calcium dipicolinate
(C) It has a diameter of less than 0.2 mm. (E) Facilitated diffusion (D) Sulfhydryl-containing proteins
(D) The wavelength of white light is too (E) Lipid A
5. Which of the following components is
long.
present in gram-negative bacteria but not 8. Which of the following terms does NOT
(E) Rapid movement of the organism
in gram-positive bacteria? describe the bacterial chromosome?
prevents visualization.
(A) Peptidoglycan (A) Haploid
2. Chloramphenicol, an antibiotic that
(B) Lipid A (B) Diploid
inhibits bacterial protein synthesis, will
(C) Capsule (C) Circular
also affect which of the following
(D) Flagella (D) Nucleoid
eukaryotic organelles?
(E) Pili (E) Feulgen positive
(A) Mitochondria
6. Which of the following components is 9. Lysozyme cleaves the β1→4 linkage
(B) Golgi apparatus
present in gram-positive bacteria but not between
(C) Microtubules
in gram-negative bacteria?
(D) Endoplasmic reticulum (A) d-Alanine and the pentaglycine bridge
(E) Nuclear membrane (A) Peptidoglycan (B) N-Acetylmuramic acid and d-alanine
(B) Capsule (C) Lipid A and KDO
3. Which of the following structures is not
(C) Flagella (D) N-Acetylmuramic acid and N-
part of the bacterial
(D) Teichoic acid acetylglucosamine
cell envelope? (E) Diaminopimelic acid (E) d-Alanine and d-alanine

(A) Peptidoglycan 7. In the fall of 2001, a series of letters 10. Mycoplasma species lack which of the
(B) Lipopolysaccharide containing spores of Bacillus anthracis following components?
(C) Capsule were mailed to members of the media and
(A) Ribosomes
(D) Gas vacuole to U.S.Senate offices. The result was 22
(E) S-layer cases of anthrax, with five deaths. The
(B) Plasma membrane 4. An unculturable gram-positive her bladder while having sex. These E coli
(C) Both DNA and RNA microorganism has been visualized in have a generation time of 20 minutes.
(D) Lipids tissue specimens obtained from patients After a lag of 20 minutes, the E coli enter
(E) Peptidoglycan with a previously undescribed disease. the logarithmic phase of growth. After 3
Which of the following techniques would hours of logarithmic growth, the total
1. Eubacteria that lack cell walls and do
be most useful in identifying this number of cells is
not synthesize the precursors of
organism?
peptidoglycan are called (A) 2560
(A) Serology (B) 5012
(A) Gram-negative bacteria
(B) PCR amplification and sequencing of (C) 90
(B) Viruses
rRNA genes (D) 1028
(C) Mycoplasmas
(C) Multilocus enzyme electrophoresis (E) 1,000,000
(D) Serovar variant
(D) SDS-polyacrylamide gel electrophoresis
(E) Bacilli 2. A 73-year-old woman is admitted to the
(E) Pulsed field gel electrophoresis
hospital for intravenous treatment of an
2. Archaebacteria can be distinguished
5. The DNA polymerase from Thermus abscess caused by Staphylococcus aureus.
from eubacteria by their lack of
aquaticus is an important component of Subsequent to her treatment and
(A) DNA DNA amplification methods such as the discharge from the hospital, it is necessary
(B) RNA polymerase chain reaction. This organism to disinfect the hospital room. One
(C) Ribosomes is capable of growing at temperatures thousand of the S aureus cells are exposed
(D) Peptidoglycan above 100°C. Organisms that are capable to a disinfectant. After 10 minutes,90% of
(E) Nucleus of growth at these temperatures are the cells are killed. How many cells remain
referred to as viable after 20 minutes?
3. A 16-year-old cystic fibrosis patient is
admitted to the hospital. Which of the (A) Mesophiles (A) 500
following techniques would be most useful (B) Psychrophiles (B) 100
in this endeavor? (C) Halophiles (C) 10
(D) Thermophiles (D) 1
(A) Culture (E) Chemolithotrophs (E) 0
(B) Ribotyping
(C) 16S rRNA sequencing 1. A 23-year-old woman has 10 Escherichia 3. The action of which of the following
(D) Antimicrobial susceptibility testing coli inoculated into agents or processes on bacteria can be
(E) Nucleic acid sequencing reversed?
(A) A disinfectant 2. The process by which microorganisms grow in a defined medium consisting of
(B) A bactericidal agent form ATP during the fermentation of glucose, mineral salts, and ammonium
(C) A bacteriostatic agent glucose is characterized by chloride. However, it is capable of growth
(D) Autoclaving at 121°C for 15 minutes in this medium if methionine is added. The
(A) Coupling of ATP production with the
(E) Dry heat at 160–170°C for 1 hour methionine is referred to as
transfer of electrons
4. The growth rate of bacteria during the (B) Denitrification (A) An inorganic salt
exponential phase of growth is (C) The reduction of oxygen (B) A sulfur source
(D) Substrate phosphorylation (C) A growth factor
(A) Zero
(E) Anaerobic respiration (D) An energy source
(B) Increasing
(E) A nitrogen source
(C) Constant 3. Which of the following culture
(D) Decreasing techniques and media would enumerate 6. Which of the following is NOT a
(E) Negative the greatest number of microbial species mechanism for generating metabolic
in a soil sample? energy by microorganisms?
5. The growth rate of bacteria during the
maximum stationary phase of growth is (A) Enrichment culture (A) Fermentation
(B) A plate of selective medium (B) Protein synthesis
(A) Zero
(C) A plate of differential medium (C) Respiration
(B) Increasing
(D) A tube of nutrient broth (D) Photosynthesis
(C) Constant
(E) A number of different media and (E) C and D
(D) Decreasing
conditions of incubation
(E) Negative 7. Which of the following terms best
4. Polymerization of building blocks (eg, describes a microorganisms that grows at
1. Most microorganisms pathogenic for amino acids) into macromolecules (eg, 20oC?
humans grow best in the laboratory when proteins) is achieved largely by
(A) Neutralophile
cultures are incubated at
(A) Dehydration (B) Psychrotroph
(A) 15–20°C (B) Reduction (C) Mesophile
(B) 20–30°C (C) Oxidation (D) Osmophile
(C) 30–37°C (D) Assimilation (E) Thermophile
(D) 38–50°C (E) Hydrolysis
8. The ability to assimilate N2 reductively
(E) 50–55°C
5. A strain of E coli undergoes a mutation via NH3 is called
such that it can no longer
(A) Ammonification 2. The synthesis of which of the following 5. The ability to use compounds and ions
(B) Anammox cell components is determined entirely by other than oxygen as terminal oxidants in
(C) Assimilatory nitrate reduction enzyme specificities? respiration is a widespread microbial trait.
(D) Deamination This capacity is called
(A) DNA
(E) Nitrogen fixation
(B) Ribosomal RNA (A) Photosynthesis
9. Which of the following is NOT (C) Flagella (B) Fermentation
assimilated by eukaryotic cells? (D) Lipopolysaccharide (C) Anaerobic respiration
(E) Protein (D) Substrate phosphorylation
(A) Glucose
(E) Nitrogen fixation
(B) Lactate 3. The steps leading to the synthesis of
(C) Sulfate (SO42-) peptidoglycan occur in the cytoplasm, on 6. The primary route of carbon
(D) Nitrogen (N2) the cytoplasmic membrane, and assimilation used by organisms that can
(E) Phosphate (PO43-) extracellularly. Which antibiotic inhibits an use CO2 as a sole source of carbon is
extracellular step in peptidoglycan
10. Bacteria that are obligate intracellular (A) Hexose monophosphate shunt
biosynthesis?
pathogens of humans (eg, Chlamydia (B) Entner-Doudoroff pathway
trachomatis) are considered to be (A) Cycloserine (C) Embden-Meyerhof pathway
(B) Rifampin (D) Glyoxalate cycle
(A) Autotrophs
(C) Penicillin (E) Calvin cycle
(B) Photosynthetic
(D) Bacitracin
(C) Chemolithotrophs 7. The peptidoglycan biosynthetic pathway
(E) Streptomycin
(D) Hyperthermophiles is of particular importance in medicine
(E) Heterotrophs 4. Amino acids are found in the protein, because it provides a basis for selective
peptidoglycan, and capsule of bacteria. antibacterial action of several
1. The synthesis of which of the following
Which of the following amino acids is chemotherapeutic agents. All of the
cell components is dependent on a
found only in peptidoglycan? following antibiotics inhibit steps in
template?
peptidoglycan biosynthesis EXCEPT
(A) l-Lysine
(A) Lipopolysaccharide
(B) Diaminopimelic acid (A) Cycloserine
(B) Peptidoglycan
(C) d-Glutamate (B) Vancomycin
(C) Capsular polysaccharide
(D) l-Alanine (C) Bacitracin
(D) Deoxyribonucleic acid
(E) None of the above (D) Streptomycin
(E) Phospholipids
(E) Penicillin
8. The regulation of enzyme activity (A) Base substitutions (D) Plasmid
provides fine control of metabolic (B) Deletions (E) Transposon
pathways. Which of the following (C) Insertions
5. The formation of a mating pair during
regulatory mechanisms provides fine (D) Rearrangements
the process of conjugation in Escherichia
control of a biosynthetic pathway? (E) All of the above
coli requires
(A) Catabolite repression 2. The form of genetic exchange in which
(A) Lysis of the donor
(B) Induction donor DNA is introduced to the recipient
(B) A sex pilus
(C) Feedback inhibition by a bacterial virus is
(C) Transfer of both strands of DNA
(D) Attenuation
(D) A restriction endonuclease
(E) None of the above (A) Transformation
(E) Integration of a transposon
(B) Conjugation
9. The biosynthetic origin of building
(C) Transfection
blocks and coenzymes can be traced back 1. What is a characteristic of the adaptive
(D) Transduction
to relatively few precursors called focal immune response and not of the innate
(E) Horizontal transfer
metabolites. Which of the following are response?
focal metabolites? 3. The form of genetic exchange in
(A) Physical barriers
bacteria that is most susceptible to the
(A) a-Ketoglutarate (B) Chemical barriers
activity of deoxyribonuclease during the
(B) Oxaloacetate (C) Clonal expansion of effector cells
process of DNA uptake is
(C) Phosphoenolpyruvate (D) Inflammatory mediators
(D) Glucose 6-phosphate (A) Transformation (E) Phagocytosis
(E) All of the above (B) Conjugation
2. Which genetic mechanism increases the
(C) Transfection
10. Which of the following is NOT a number of different antibody molecules
(D) Transduction
component of peptidoglycan? during an immune response without
(E) All of the above
(A) N-Acetyl muramic acid increasing the diversity of the pool of
(B) N-Acetyl glucosamine 4. Replication of which of the following antigen receptor specificities?
(C) Lipid A requires physical integration with a
(A) V gene segment recombination
(D) Pentaglycine bacterial replicon?
(B) Class switching
(E) Diaminopimelic acid
(A) Single-stranded DNA bacteriophage (C) Somatic hypermutation
1. Mutations in bacteria can occur by (B) Double-stranded DNA bacteriophage (D) Junctional variability due to imprecise
which of the following mechanisms? (C) Single-stranded RNA bacteriophage V, D, and J joining
(E) Gene duplication ie, multiple V, D, and pets surface that stimulate the cell to mount a
J gene segments (E) Autoimmune disease response to parasites such as worms?

3. What is the principal function of the 5. Which major antibody molecule has the (A) T cells
class I and class II MHC molecules? ability to cross the placenta? (B) B cells
(C) Promonocytes
(A) They are mediators of T-independent (A) IgG
(D) NK cells
B-cell responses. (B) IgA
(E) Mast cells
(B) They bind peptide antigens for (C) IgM
presentation to antigenspecific receptors (D) IgE 8. Which immunologic test is widely used
on B cells. (E) IgD to precisely enumerate and collect cells
(C) They help in endocytosis of antigens by expressing an antigen bound by a
6. A man in his twenties presents in the
phagocytic cells. fluorescencetagged monoclonal antibody?
emergency room with shortness of breath
(D) They bind carbohydrate antigens
and fatigue. He is also very pale. Two days (A) ELISA
directly for presentation on T cells.
earlier he was given penicillin for an (B) Direct immunofluorescence
(E) They display peptide antigens for
infection. He had penicillin previously (C) Western blotting
review by antigenspecific receptors on T
without problems and stated that he had (D) Fluorescence-activated cell sorting
cells.
“no allergy” to penicillin. Laboratory (E) Indirect immunofluorescence
4. MHC class I molecules need to bind testing shows that antibodies to penicillin
9. In any given immunoglobulin molecule,
peptide antigens to fold properly and to are present in the patient’s serum and
the light chains are:
be expressed at the cell surface. What that he is breaking down his own red
would you expect to be the most common blood cells. He is diagnosed (A) Identical to each other in their
health problem in a child antigenic determinants
with immune hemolytic anemia. The
(B) Identical to each other
with a defect in the function of the patient has which type of hypersensitivity
(C) Identical to each other except in their
peptide transporter (TAP) found in the reaction?
hypervariable regions
endoplasmic reticulum?
(A) Type I (D) Of related but different amino acid
(A) Chronic upper respiratory viral (B) Type II sequences
infections (C) Type III (E) Identical to each other except in their
(B) Parasitic infections (D) Type IV (DTH) overall domain structure
(C) Infections with encapsulated bacteria
7. Which one of the following cell types 10. Antigen–antibody complexes are
(D) Pronounced allergies to household
expresses receptors for IgE on its cell phagocytosed more effectively in the
presence of which complement (D) TNF-α (A) Sexual activity
component? (E) IL-10 (B) Ingesting the microorganisms in her
(A) C3a and C5a food
14. IgE-mediated histamine release is
(B) C3b (C) Holding onto contaminated hand rails
classified as what type of hypersensitivity
(C) C56789 complex when she takes public transportation
reaction?
(D) MBL (D) Handling potting soil
(E) Properdin (A) Type 1 (E) Breathing aerosolized droplets
(B) Type 2 containing the microorganism
11. NK cells express a killer
(C) Type 3
immunoglobulin-like receptor, which 2. During a pandemic of a well-
(D) Type 4
recognizes: characterized disease, a group of 175
15. The interaction of a pathogen airline passengers flew from Lima, Peru, to
(A) MHC class I molecules
molecule with its specific TLR directly Los Angeles.Lunch on the plane included
(B) MHC class II molecules
results in which of the following? crab salad, which was eaten by about two-
(C) Cell adhesion molecules
thirds of the passengers. After landing in
(D) Glycophospholipid molecules (A) Presentation of pathogen molecule to Los Angeles, many of the passengers
(E) CD40 molecules helper T cells transferred to other flights with
(B) Cell activation and production of destinations in other parts of California
12. A cell that plays a critical role in the
cytokines and chemokines and other Western states. Two of the
innate immune response and kills virus
(C) IgG production passengers who stayed in Los Angeles
infected cells is:
(D) Immunoglobulin class switching developed severe watery diarrhea. The
(A) T cell status of the other passengers was
(E) Phagocytosis
(B) Neutrophil unknown. The likely cause of the diarrhea i
(C) NK cell 1. A 22-year-old woman who works in a the two passengers is
(D) Macrophage plant nursery presents with a history of
(E) B cell fever and cough for 2 months. Over this (A) Escherichia coli O157:H7
period of time she has lost 5 kg. Chest (lipopolysaccharide O antigen 157;
13. A cytokine that activates cells to flagellar antigen 7)
radiography shows bilateral upper lobe
express MHC class II antigens and protects (B) Vibrio cholerae type O139
infiltrates with cavities. A stain of her
cells from virus replication is: (lipopolysaccharide O antigen 139)
sputum shows acid-fast bacilli. The likely
(A) Interferon-α means by which the patient acquired her (C) Shigella dysenteriae type 1
(B) IL-6 infection is (D) Campylobacter jejuni
(C) Interferon-γ (E) Entamoeba histolytica
3. A 65-year-old woman has a long-term 4. The first microorganism to satisfy Koch’s This patient’s illness was likely to be
central venous catheter for intravenous postulates (in the late 19th century) was caused by which of the following?
therapy. She develops fever and
(A) Treponema pallidum (A) Lipopolysaccharide
subsequently has multiple blood cultures
(B) Stenotrophomonas maltophilia (B) Peptidoglycan
positive for Staphylococcus epidermidis.
(C) Mycobacterium leprae (C) A toxin that is a superantigen
All of the S epidermidis isolates have the
(D) Bacillus anthracis (D) A toxin that has A and B subunits
same colony morphology and
(E) Neisseria gonorrhoeae (E) Lecithinase (alpha toxin)
antimicrobial susceptibility pattern,
suggesting that they are the same strain. A 5. Which of the following statements 7. The organism most likely to be
S epidermidis biofilm is thought to be about lipopolysaccharide is correct? responsible for the patient’s
present on the catheter. Which one of the disease (Question 6) is
following statements about such an (A) It interacts with macrophages and
infection is correct? monocytes yielding release of cytokines. (A) Escherichia coli
(B) The toxic component is the O side (B) Corynebacterium diphtheria
(A) The biofilm containing the S chain. (C) Clostridium perfringens
epidermidis is likely to wash off the (C) It forms holes in red blood cell (D) Neisseria meningitides
catheter. membranes yielding hemolysis. (E) Staphylococcus aureus
(B) Production of an extracellular (D) It causes hypothermia
polysaccharide inhibits growth of the S 8. Which of the following is most likely to
(E) It causes paralysis.
epidermidis, limiting the infection. be associated with the
(C) The S epidermidis in the biofilm are 6. A 27-year-old man had a rhinoplasty. A formation of a bacterial biofilm?
likely to be more susceptible to nasal tampon was placed to control the
antimicrobial therapy because the bacteria bleeding. Approximately 8 hours later, he
(A) Airway colonization in a cystic fibrosis
have decreased rates of metabolism. developed headache, muscle aches, and
patient with a mucoid (alginate-
(D) The quorum-sensing ability of S abdominal cramps with diarrhea. He then
producing) strain of Pseudomonas
epidermidis results in increased developed an erythematous rash
aeruginosa
susceptibility to antimicrobial therapy. (resembling sunburn) over much of his
(B) Urinary tract infection with Escherichia
(E) The complex molecular interactions body, including the palms and soles.His
coli
within the biofilm make it difficult to blood pressure is 80/50 mm Hg. The nasal
(C) Meningitis with Neisseria meningitides
provide effective antimicrobial therapy, tampon remained in place. His liver
(D) Tetanus
and it is likely the catheter will have to be enzyme tests were elevated, and there
(E) Impetigo caused by Staphylococcus
removed to cure the infection. was evidence of moderate renal failure.
aureus
9. Regarding bacterial type III secretions 11. A 15-year-old Bangladeshi girl blood group antigen. Which of the
systems, which of the following develops severe watery diarrhea.The stool following is likely to be the primary cause
statements is correct? looks like “rice water.” It is voluminous— of her infections?
(A) They are commonly found in gram- more than 1 L in the last 90 minutes. She
(A) Escherichia coli that produce heat-
positive commensal bacteria. has no fever and seems otherwise normal
stable toxin
(B) They play an important role in the except for the effects of loss of fluid and
(B) Escherichia coli with K1 (capsular type
pathogenesis of toxininduced diseases of electrolytes. The most likely cause of her
1) antigen
Clostridium species, tetanus, botulism, gas illness is
(C) Escherichia coli O139
gangrene, and pseudomembranous colitis.
(A) Clostridium difficile enterotoxin (lipopolysaccharide O antigen 139)
(C) They cause release of effectors of
(B) A toxin with A and B subunits (D) Escherichia coli with P-pili (fimbriae)
pathogenesis into the extracellular
(C) Shigella dysenteriae type 1 that (E) Escherichia coli O157:H7
environment, promoting bacterial
produces Shiga toxin (lipopolysaccharide O antigen
colonization and multiplication.
(D) Enterotoxigenic Escherichia coli that 157; flagellar antigen 7)
(D) They directly inject bacterial proteins
produces heat-labile and heat-stable
into host cells across bacterial and host 14. A 55-year-old man presents with
toxins
cell membranes, promoting pathogenesis gradually increasing weight loss,
(E) Staphylococcal enterotoxin F
of infections. abdominal pain, diarrhea, and
(E) Mutations that prevent the bacterial 12. The most important thing that can be arthropathy. During the evaluation
type III secretion from functioning done to treat the patient (Question 11) is process, a small bowel biopsy is done.
enhance pathogenesis After processing, examination of the
(A) To give her ciprofloxacin
specimen by light microscopy reveals
10. Which of the following statements is (B) To give her a toxoid vaccine
periodic acid-Schiff–positive inclusions in
correct? (C) To give her the appropriate antitoxin
the bowel wall. Which of the following
(D) To treat her with fluid and electrolyte
(A) Lipopolysaccharide is part of the cell tests could be done to confirm the
replacement
wall of Escherichia coli.
(E) To culture her stool to make the diagnosis of Whipple disease, caused by
(B) Cholera toxin is attached to the flagella
correct diagnosis and then treat Tropheryma whipplei?
of Vibrio cholerae.
specifically
(C) The lecithinase of Clostridium (A) Culture on agar media
perfringens causes diarrhea. 13. A 23-year-old woman has a history of (B) Polymerase chain reaction
(D) Toxic shock syndrome toxin-1 is recurrent urinary tract infections, amplification and sequencing of an
produced by hemolytic stains of including at least one episode of appropriate segment of DNA
Staphylococcus epidermidis. pyelonephritis. Blood typing shows the P (C) Cocultivation with Escherichia coli
(D) In situ hybridization (D) Candida albicans lower quadrant abdominal pain. Fever
(E) Direct fluorescent antibody test (E) Lactobacillus species develops. The severe pain gradually
subsides and is replaced by a constant
2. Certain microorganisms are never
15. Which of the following best describes aching pain and marked abdominal
considered to be members of the normal
the mechanism of action of diphtheria tenderness. A diagnosis of probable
flora. They are always considered to be
toxin? ruptured diverticulum is made, and the
pathogens.Which one of the following
patient is taken to the operating room.
(A) Forms pores in red blood cells causing organisms fits into that category?
The diagnosis of ruptured diverticulum is
hemolysis
(A) Streptococcus pneumonia confirmed, and an abscess next to the
(B) Degrades lecithin in eukaryotic cell
(B) Escherichia coli sigmoid colon is found. The most likely
membranes
(C) Mycobacterium tuberculosis bacteria to be found in the abscess are
(C) Causes release of tumor necrosis factor
(D) Staphylococcus aureus
(D) Inhibits elongation factor 2 (A) Mixed normal gastrointestinal flora
(E) Neisseria meningitidis
(E) Causes increased adenylate cyclase (B) Bacteroides fragilis alone
activity 3. A 9-year-old girl develops fever and (C) Escherichia coli alone
severe pain on the right side (D) Clostridium perfringens alone
1. A 26-year-old woman visits her
(E) Enterococcus species alone
physician because of an unusual vaginal of her throat. On examination, redness
discharge. On examination, the physician and swelling in the right peritonsillar area 5. Antimicrobial therapy can decrease the
observes a thin, homogeneous, white-gray are seen. A peritonsillar abscess is amount of susceptible bowel flora and
discharge that adheres to the vaginal wall. diagnosed. The most likely organisms to allow proliferation of relatively resistant
The pH of the discharge is 5.5 (normal, be cultured from this abscess are colonic bacteria. Which one of the
<4.3). On Gram stain, many epithelial cells following species can proliferate and
covered with gram-variable rods are seen. (A) Staphylococcus aureus produce a toxin that causes diarrhea?
Bacterial vaginosis is diagnosed. Which (B) Streptococcus pneumonia
(C) Corynebacterium species and (A) Enterococcus species
one of the following normal genital flora
Prevotella melaninogenica (B) Staphylococcus epidermidis
microorganisms is greatly decreased in
(D) Normal oral nasal flora (C) Pseudomonas aeruginosa
bacterial vaginosis?
(E) Viridans streptococci and Candida (D) Clostridium difficile
(A) Corynebacterium species albicans (E) Bacteroides fragilis
(B) Staphylococcus epidermidis
4. A 70-year-old man with a history of 6. Which one of the following
(C) Prevotella species
diverticulosis of the sigmoid colon microorganisms can be part of the normal
experiences a sudden onset of severe left
vaginal flora and cause meningitis in (A) 1011/g 13. Resident flora is absent from the
newborns? (B) 1010/g
(A) Pharynx
(C) 109/g
(A) Candida albicans (B) Lungs
(D) 108 /g
(B) Corynebacterium species (C) Small intestine
(E) 107/g
(C) Staphylococcus epidermidis (D) Synovial fluid
(D) Ureaplasma urealyticum 10. Streptococcus pneumoniae can be part (E) Conjunctiv
(E) Group B streptococci of the normal flora of 5–40% of people. At 14. A 65-year-old woman was admitted
what anatomic site can it be found? with squamous cell carcinoma of the
7. Dental plaque and periodontal disease
cervix. She underwent extensive
can be thought of as a continuum of what (A) Conjunctiva
gynecologic surgery and was maintained
type of physiological process? (B) Nasopharynx
postoperatively on broad-spectrum
(C) Colon
(A) Biofilm formation intravenous antibiotics. The patient had a
(D) Urethra
(B) Normal aging central venous catheter placed on the day
(E) Vagina
(C) Abnormal digestion of surgery. Beginning 3 days
(D) Exaggerated immune response 11. Hundreds of phylotypes have been postoperatively, the patient became
(E) Chewing gum identified in the human stomach; febrile. On day 8, cultures of blood and of
however, the only microorganism that has the tip of the central line both grew gram-
8. Which one of the following positive organisms that were ovoid and
been shown to persist is
microorganisms is closely associated with reproduced by budding. Which of the
dental caries? (A) Lactobacillus casei following microorganisms is most likely
(B) Lactobacillus acidophilus responsible for the patient’s condition?
(A) Candida albicans
(C) Escherichia coli
(B) Streptococcus mutans (A) Staphylococcus aureus
(D) Helicobacter pylori
(C) Prevotella melaninogenica (B) Staphylococcus epidermidis
(E) Bifidobacteria
(D) Neisseria subflava (C) Enterococcus faecalis
(E) Staphylococcus epidermidis 12. Resident flora is commonly found in (D) Candida albicans
the (E) Saccharomyces cerevisae
9. Anaerobic bacteria such as Bacteroides
fragilis occur in the sigmoid colon in a (A) Liver 15. The most likely portal of entry for the
concentration of about 1011/g of stool. At (B) Urethra organism in Question 14 is
what concentration do facultative (C) Kidneys
organisms such as Escherichia coli occur? (D) Salivary glands (A) During gynecologic surgery
(E) Gall bladder (B) Aspiration
(C) During placement of the central line elongation factor EF-2 (D) Streptolysin O
(D) During placement of IV line for (E) Lecithinase (E) Toxin B
administration of antibiotics
3. A young man sustains major soft tissue 5. The reported incubation period for
(E) Intubation while under anesthesia
injury and open fractures of his right leg inhalational anthrax can be up to
after a motorcycle accident. One day later,
(A) 2 days
1. A housewife who lives on a small farm is
he has a temperature of 38°C, increased
brought to the emergency department (B) 10 days
heart rate, sweating, and restlessness. On
complaining of double vision and difficulty (C) 3 weeks
examination, the leg is swollen and tense,
talking. Within the past 2 hours, she noted (D) 6 weeks
with thin, dark serous fluid draining from
a dry mouth and generalized weakness. (E) 6 months
the wounds. The skin of the leg is cool,
Last night she served home-canned green
pale, white, and shining. Crepitus can be 6. A food commonly associated with
beans as part of the meal. She tasted the
felt in the leg. His hematocrit is 20% (~50% Bacillus cereus food poisoning is
beans before they were boiled. None of
of normal), and his circulating hemoglobin
the other family members are ill. On (A) Fried rice
is normal. His serum shows free
examination, there is symmetrical (B) Baked potato
hemoglobin. Which of the following
descending paralysis of the cranial nerves, (C) Hot freshly steamed rice
microorganisms is the most likely cause of
upper extremities, and trunk. The correct (D) Green beans
this infection?
diagnosis is which one of the following? (E) Honey
(A) Clostridium tetani
(A) Tetanus 7. Tetanus toxin (tetanospasmin) diffuses
(B) Staphylococcus aureus
(B) Strychnine poisoning to terminals of inhibitory cells in the spinal
(C) Escherichia coli
(C) Botulism cord and brainstem and blocks which of
(D) Bacillus anthracis
(D) Morphine overdose the following?
(E) Clostridium perfringens
(E) Ricin intoxication
(A) Release of acetylcholine
2. Which one of the following is an (B) Cleavage of SNARE proteins
important virulence factor of Bacillus 4. For the patient described in Question 3, (C) Release of inhibitory glycine and γ-
anthracis? which of the following is likely to be aminobutyric acid
responsible for the hemolysis? (D) Release of PA
(A) Protective antigen
(B) Lipopolysaccharide (E) Activation of acetylcholine esterase
(A) EF
(C) Pili (B) Tetanospasmin
(D) A toxin that inhibits peptide chain (C) Lecithinase
8. A 45-year-old man who immigrated to tetanus is less than 1%. 11. Infant botulinum has been associated
the United States 5 years ago sustained a (D) Double vision is commonly the first with all of the following Clostridium
puncture injury to the lower part of his sign of tetanus. species except:
right leg when his rotary lawn mower (E) Tetanus toxin acts on inhibitor
(A) Clostridium baratii
threw a small stick into his leg. Six days interneuron synapses.
(B) Clostridium septicum
later, he noticed spasms in the muscles of
10. A 67-year-old man had surgery for a (C) Clostridium butyricum
his right leg; on day 7, the spasms
ruptured sigmoid colon (D) Clostridium botulinum
increased. Today—day 8—he had
generalized muscle spasms, particularly diverticulum with an abscess. A repair was 12. Which of the following food items is
noticeable in the muscles of his jaw. He done, and the abscess was drained. He most frequently associated
was unable to open his jaw and came to was treated with intravenous gentamicin
the emergency department (ED). In the with infant botulism?
and ampicillin. Ten days later and 4 days
ED, you see a man who is alert and lying after being discharged from the hospital, (A) Corn syrup
quietly in bed. A door slams down the hall, the patient developed malaise, fever, and (B) Canned infant formula
and suddenly he has general muscle cramping abdominal pain. He had multiple (C) Liquid multivitamins
spasm with arching of his back. The episodes of diarrhea. His stool was (D) Honey
correct diagnosis is which of the positive for occult blood and the presence (E) Jarred baby food
following? of polymorphonuclear cells. On
sigmoidoscopy, the mucosa was 13. All of the following are properties
(A) Botulism characteristic of Bacillus anthracis EXCEPT
erythematous and appeared to be
(B) Anthra
inflamed, and there were many raised (A) Motility on wet mount examination
(C) Gas gangrene
white to yellowish plaques 4–8 mm in (B) Medusa head colonies
(D) Tetanus
diameter. Which of the following is the (C) Poly-d-glutamic acid capsule
(E) Toxic shock syndrome
likely cause of the patient’s problem? (D) In vitro susceptibility to penicillin
9. Which of the following statements (E) Absence of hemolysis on 5% sheep
(A) Staphylococcus aureus enterotoxin
about tetanus and tetanus toxoid is blood agar
(B) Bacillus cereus toxin
correct?
(C) Clostridium difficile toxins 14. Which of the following statements
(A) Tetanus toxin kills neurons. (D) Clostridium perfringens toxin regarding vaccination for Bacillus anthracis
(B) Tetanus toxoid immunization has a (E) Enterohemorrhagic Escherichia coli is correct?
10% failure rate.
(C) The mortality rate of generalized
(A) It is routinely available for all citizens of radiography showed a left upper lobe 4. Movement of Listeria monocytogenes
the United States. infiltrate. Contrastenhanced computed inside of host cells is caused by
(B) Recombinant vaccine trials have tomography showed two lesions in the
(A) Inducing host cell actin polymerization
shown good safety and efficacy. right hemisphere. Gram stain of a purulent
(C) The current vaccine is well tolerated. sputum specimen (B) The formation of pili (fimbriae) on the
(D) A single dose is adequate after listeriae surface
showed branching gram-positive rods that
exposure to spores. (C) Pseudopod formation
were partially acid fast. Which of the
(E) Vaccination of animals is not useful. (D) The motion of listeriae flagella
following organisms is the cause of this
(E) Tumbling motility
15. All of the following statements patient’s current illness?
regarding Clostridium perfringens are 5. An 8-year-old boy develops a severe
(A) Actinomyces israelii
correct EXCEPT sore throat. On examination, a grayish
(B) Corynebacterium
exudate (pseudomembrane) is seen over
(A) It produces an enterotoxin. pseudodiphtheriticum
the tonsils and pharynx. The differential
(B) It produces a double zone of β- (C) Aspergillus fumigatus
diagnosis of severe pharyngitis such as this
hemolysis when grown on blood agar. (D) Nocardia farcinica
includes group A streptococcal infection,
(C) Some strains are aerotolerant. (E) Erysipelothrix rhusiopathiae
Epstein-Barr virus (EBV) infection,
(D) It is the most common cause of
2. The drug of choice to treat this patient’s Neisseria gonorrhoeae pharyngitis, and
antibiotic-associated diarrhea.
infection (Question 1) is diphtheria. The cause of the boy’s
(E) It can cause intravascular hemolysis
pharyngitis is most likely
(A) Penicillin G
(B) Trimethoprim–sulfamethoxazole (A) A gram-negative bacillus
1. Three months ago, a 53-year-old
(C) Gentamicin (B) A single-stranded positive-sense RNA
woman had surgery and chemotherapy for
(D) Amphotericin B virus
breast cancer. Four weeks ago, she
(E) A third-generation cephalosporin (C) A catalase-positive gram-positive
developed a cough occasionally productive
coccus that grows in clusters
of purulent sputum. About 2 weeks ago, 3. It is particularly difficult to differentiate
(D) A club-shaped gram-positive bacillus
she noted a slight but progressive Erysipelothrix rhusiopathiae from
(E) A double stranded RNA virus
weakness of her left arm and leg. On chest
examination, rales were heard over the (A) Corynebacterium diphtheria
6. The primary mechanism in the
left upper back when the patient breathed (B) Bacillus cereus
pathogenesis of the boy’s disease
deeply. Neurologic examination confirmed (C) Actinomyces israelii
(Question 5) is
weakness of the left arm and leg.Chest (D) Nocardia asteroids
(E) Lactobacillus species
(A) A net increase in intracellular cyclic (C) Is also common in northern latitudes (C) Esculin hydrolysis
adenosine monophosphate (D) Results in protective antitoxin levels (D) Tumbling motility
(B) Action of pyrogenic exotoxin (a in most children by the time they are 6–8 (E) Production of H2
superantigen) years old

(C) Inactivation of acetylcholine esterase (E) Yields toxin-mediated cardiomyopathy


12. Listeria monocytogenes is frequently a
(D) Action of enterotoxin A
10. A 45-year-old fisherman imbedded a foodborne pathogen because
(E) Inactivation of elongation factor 2
fishhook into his right forefinger. He
(A) It can survive at 4°C.
7. Corynebacterium jeikeium is removed it and did not seek immediate
(B) It survives under conditions of low pH.
medical therapy. Five days later, he noted
(A) Catalase negative (C) It survives in the presence of high salt
fever, severe pain, and nodular-type
(B) Gram negative concentrations.
swelling of the finger. He sought medical
(C) Often multidrug resistant (D) All of the above are correct.
therapy. The violaceous nodule was
(D) Motile§
aspirated, and after 48 hours of 13. After recovery on laboratory media,
(E) Common but clinically unimportant
incubation, colonies of a gram-positive the aerobic Actinomycetes are best
8. Which of the following aerobic gram- bacillus that caused greenish discoloration identified by
positive bacilli is modified acid-fast of the agar and formed long filaments in (A) An automated system used in the
positive? the broth culture were noted. The most laboratory
likely cause of this infection is (B) Classical biochemical
(A) Nocardia brasiliensis
(C) Antigen detection tests such as an
(B) Lactobacillus acidophilus (A) Lactobacillus acidophilus
ELISA
(C) Erysipelothrix rhusiopathiae (B) Erysipelothrix rhusiopathiae
(D) Molecular methods such as 16SrRNA
(D) Listeria monocytogenes (C) Listeria monocytogenes
gene sequencing
(D) Rhodococcus equi
9. Skin diphtheria as occurs in children in (E) Nocardia brasiliensis 14. Which of the following statements
tropical areas typically
regarding Rhodococcus equi is correct?
11. A biochemical reaction that is useful in
(A) Does not occur in children who have the identification of the causative agent of (A) It is transmitted person to person.
been immunized with diphtheria toxoid the infection in question 10 is (B) It causes tuberculosis in cattle.
(B) Is clinically distinct from skin infections
(C) It is a rare cause of pulmonary
(pyoderma, impetigo) caused by (A) Catalase positivity
infection in humans.
Streptococcus pyogenes and (B) Acid fastness using modified Kinyoun
(D) It produces a black pigment on sheep
Staphylococcus aureus stain
blood agar.
15. A hospitalized patient who had an 11, 8 μg/mL. The patient failed to improve (C) A producer of protein A
indwelling urinary catheter develops fever, with the third course of vancomycin, and (D) Encapsulated
chills, suprapubic pain and difficulty alternative therapy was used. The (E) Catalase positive
voiding 48 hours after the catheter is mechanism that best explains the relative
3. A 36-year-old male patient has an
removed. His bladder appears obstructed, resistance of the patient’s strain of S
abscess with a strain of S aureus that is β-
and he has white blood cells and bacteria aureus to vancomycin is
lactamase positive. This indicates that the
on a urinalysis. Cystoscopy reveals a large
(A) Acquisition of the vanA gene from organism is resistant to which of the
bladder stone, and the urine culture grows
another microorganism following antibiotics?
greater than 10,000 CFU/mL of a short,
(B) Active transport of vancomycin out of
irregular grampositive rod. The most likely (A) Penicillin G, ampicillin, and
the S aureus cell
organism causing this infection is piperacillin
(C) Action of β-lactamase
(B) Trimethoprim–sulfamethoxazole
(A) Corynebacterium urealyticum (D) Increased cell wall synthesis and
(C) Erythromycin, clarithromycin, and
(B) Nocardia brasiliensis alterations in the cell wall structure
azithromycin
(C) Actinomadura (E) Phosphorylation and resultant
(D) Vancomycin
(D) Erysipelothrix inactivation of the vancomycin
(E) Cefazolin and ceftriaxone
1. A 54-year-old woman develops a right 2. An 11-year-old boy develops a mild
4. Seven days ago, a 27-year-old medical
shoulder abscess with a strain of fever and pain in his upper arm. A
student returned from Central America,
Staphylococcus aureus (S aureus) that is radiograph of his arm shows a lytic lesion
where she had spent the summer working
resistant to nafcillin. She was treated with (dissolution) in the upper part of the
in a clinic for indigenous people. Four days
a 2-week course of intravenous humerus with periosteal elevation over
ago, she developed an erythematous
vancomycin and improved. Three weeks the lesion. The patient is taken to surgery,
sunburn-like rash. She also has had
later (week 5), the infection recurred, and where the lesion is debrided (dead bone
headache,muscle aches, and abdominal
she was given 2 more weeks of and pus removed). Culture from the lesion
cramps with diarrhea. Her blood pressure
intravenous vancomycin and again yields gram-positive cocci. A test shows
is 70/40 mm Hg. Pelvic examination shows
improved. Four weeks later (week 11), that the organism is a
she is having her menstrual period with a
the infection recurred and the patient was staphylococcus and not a streptococcus. tampon in place; otherwise, the
again started on intravenous vancomycin. Based on this information, you know the
pelvic examination is normal. Her kidney
The MICs for vancomycin for the S aureus organism is
function test (serum urea nitrogen and
isolates were as follows: initial isolate (day
(A) Susceptible to nafcillin creatinine) results are abnormal,
1), 1 μg/mL;week 5, 2 μg/mL; and week
(B) β-Lactamase-positive indicating mild renal failure. A blood
smear for malaria is negative. Her illness is (C) Closing the nursery and referring Staphylococcus epidermidis. Which one of
likely to be caused by which of the pregnant women to another hospital the following statements is correct?
following? (D) Hiring all new staff for the hospital
(A) The S epidermidis organisms are likely
nursery
(A) A toxin that results in greatly increased to be susceptible to penicillin G.
(E) Culture using mannitol salt agar of the
levels of intracellular cyclic adenosine (B) The S epidermidis organisms are likely
anterior nares of the physicians, nurses,
monophosphate (cAMP) to be from the surface of the urinary tract
and others who cared for the infected
(B) A toxin that degrades sphingomyelin catheter.
babies
(C) A toxin that binds to the class II major (C) The S epidermidis organisms are likely
histocompatibility complex (MHC) of an 6. The exfoliative toxins, TSST-1, and the to be resistant to vancomycin
antigen-presenting cell and the Vβ region enterotoxins are all superantigens. The (D) The S epidermidis organisms are likely
of a T cell genes for these toxins are to be from a skin source.
(D) A two-component toxin that forms (E) The S epidermidis organisms are likely
(A) Present in all strains of S aureus
pores in white blood cells and increases to be in a biofilm on the central venous
(B) Widely distributed on the
cation permeability catheter surface.
staphylococcal chromosome
(E) A toxin that blocks elongation factor 2
(C) On both the staphylococcal 8. A 65-year-old man develops an abscess
(EF2)
chromosome (TSST-1 and exfoliative on the back of his neck. Culture yields S
5. Over a period of 3 weeks, a total of five toxins) and on plasmids (enterotoxins) aureus. The isolate is tested and found to
newborns in the hospital nursery (D) On the staphylococcal chromosome in be positive for the mecA gene, which
developed S aureus infections with S a pathogenicity island means that
aureus bacteremia. The isolates all had the (E) On plasmids
(A) The isolate is susceptible to
same colony morphology and hemolytic
7. A 16-year-old bone marrow transplant vancomycin.
properties and identical antimicrobial
patient has a central venous line that has (B) The isolate is resistant to vancomycin.
susceptibility patterns, suggesting that
been in place for 2 weeks. He also has a (C) The isolate is susceptible to nafcillin.
they were the same. (Later molecular
urinary tract catheter, which has been in (D) The isolate is resistant to nafcillin
methods showed the isolates were
place for 2 weeks as well. (E) The isolate is susceptible to
identical.) Which of
clindamycin.
He develops fever while his white blood
the following should be done now? (F) The isolate is resistant to clindamycin.
cell count is very low and before the
(A) Prophylactic treatment of all newborns transplant has engrafted. Three blood 9. Antimicrobial resistance has become a
with intravenous vancomycin cultures are done, and all grow significant problem. Which one of the
(B) Protective isolation of all newborns following is of major concern worldwide?
(A) Nafcillin resistance in S aureus (D) It is directly responsible for lysis of (A) It regulates production of β-
(B) Penicillin resistance in Streptococcus neutrophils. hemolysins.
pneumonia (E) It is a bacterial surface protein that (B) It is influenced by environmental
(C) Penicillin resistance in Neisseria binds to the Fc portion of IgG1. oxygen.
gonorrhoeae (C) It controls the preferential expression
12. Which of the following staphylococcal
(D) Vancomycin resistance in S aureus of surface adhesins.
organisms produces coagulase and has
(E) Tobramycin resistance in Escherichia (D) It is important in the control of
been implicated in infections following a
coli autolysis.
dog bite?
10. A group of six children younger than 8 15. All of the following are important
(A) Staphylococcus intermedius
years of age live in a semitropical country. infection control strategies in containing
(B) Staphylococcus epidermidis
Each of the children has several crusted spread of MRSA in hospitals except:
(C) Staphylococcus saprophyticus
weeping skin lesions of impetigo
(D) Staphylococcus hominis (A) Aggressive hand hygiene
(pyoderma). The lesions are
(E) Staphylococcus hemolyticus (B) Routine surveillance for nasal
predominantly on the arms and faces.
colonization among highrisk individuals
Which of the following microorganisms is 13. All of the following statements
(C) Contact isolation for patients who are
a likely cause of the lesions? regarding Panton-Valentine leukocidin are
colonized or infected with MRSA
correct except:
(A) Escherichia coli (D) Routine antimicrobial prophylaxis for
(B) Chlamydia trachomatis (A) It is a two-component toxin. all patients hospitalized for more than 48
(C) S aureus (B) It is commonly produced by hours
(D) Streptococcus pneumonia community-associated MRSA strains. (E) Aseptic management of skin lesions
(E) Bacillus anthracis (C) It is an important virulence factor.
1. A 48-year-old man is admitted to a
(D) It is identical to one of the
11. Which of the following statements hospital because of stupor. He is unkempt
staphylococcal enterotoxins.
regarding the role of protein A in the and homeless and lives in an encampment
(E) It forms pores in the membranes of
pathogenesis of infections caused by S with other homeless people, who called
white blood cells.
aureus is correct? the authorities when he could not be
14. Which of the following statements easily aroused. The patient drinks a lot of
(A) It is responsible for the rash in toxic
best describes the function of the fortified wine and drank excessively 2
shock syndrome.
accessory gene regulator in nights previously. His temperature is
(B) It converts hydrogen peroxide into
Staphylococcus aureus? 38.5°C, and his blood pressure 125/80 mm
water and oxygen.
Hg. He moans when attempts are made to
(C) It is a potent enterotoxin.
arouse him. He has positive Kernig and gram-positive diplococcic with a minimum (C) Migration to mediastinal lymph nodes
Brudzinski signs, suggesting meningeal inhibitory concentration to penicillin G of where hemorrhage occurs
irritation. Physical examination and chest greater than 2 μg/mL. The drug of choice (D) Lysis of the phagocytic vacuole and
radiography show evidence of left lower for this patient until further susceptibility release into the circulation
lobe lung consolidation. An endotracheal testing can be done is (E) Inhibition of phagocytosis by a
aspirate yields rust-colored sputum. polysaccharide capsule
(A) Penicillin G
Examination of a Gram-stained sputum
(B) Nafcillin 5. A thirteen-valent capsular
smear shows numerous
(C) Trimethoprim–sulfamethoxazole polysaccharide protein conjugate vaccine
polymorphonuclear cells and numerous
(D) Gentamicin for the pathogen in question 1 is
gram-positive lancet-shaped diplococci.
(E) Vancomycin recommended
On lumbar puncture, the cerebrospinal
fluid is cloudy and has a white blood cell 3. This infection (question 1) might have (A) For children up to age 18 years and for
count of 570/μL with 95% been prevented by selected adults
polymorphonuclear cells; Gram stain (B) Only on exposure to a patient with
shows numerous gram-positive diplococci. (A) Prophylactic intramuscular benzathine
disease caused by the organism
Based on this information, the likely penicillin every 3 weeks
(C) For children ages 2–23 months plus
diagnosis is (B) A 23-valent capsular polysaccharide
selected children through 59 months
vaccine
(D) For children ages 24–72 months
(A) Pneumonia and meningitis caused by (C) A vaccine against serogroups A, C, Y,
(E) For all age groups older than age 2
Staphylococcus aureus and W135 capsular polysaccharide
months
(B) Pneumonia and meningitis caused by (D) A vaccine of polyribosylribitol capsular
Streptococcus pyogenes polysaccharide covalently linked to a 6. An 8-year-old boy develops a severe
(C) Pneumonia and meningitis caused by protein sore throat. On examination, a grayish-
Streptococcus pneumonia (E) Oral penicillin twice daily white exudate is seen on the tonsils and
(D) Pneumonia and meningitis caused by pharynx.The differential diagnosis includes
Enterococcus faecalis 4. The pathogenesis of the organism
group A streptococcal infection, Epstein-
(E) Pneumonia and meningitis caused by causing the infection (question 1) includes
Barr virus infection, severe adenovirus
Neisseria meningitidis which of the following?
infection,and diphtheria. (Neisseria
(A) Invasion of cells lining the alveoli and
gonorrhoeae pharyngitis would alsobe
2. The patient in question 1 is started on entry into the pulmonary venule
included, but the patient has not been
antibiotic therapy to cover many possible circulation
sexually abused.) The cause of the boy’s
microorganisms. Subsequently, culture of (B) Resistance to phagocytosis mediated
pharyngitis is most likely
sputum and cerebrospinal fluid yields by M proteins
(A) A catalase-negative gram-positive One person thought it smelled like throats 2 months ago. A test that if
coccus that grows in chains butterscotch. It agglutinates with group F positive would indicate recent
(B) A single-stranded positive-sense RNA antisera. The organism most likely is Streptococcus pyogenes infections is
virus
(A) Streptococcus pyogenes (group A) (A) Antistreptolysin S antibody titer
(C) A catalase-positive gram-positive
(B) Enterococcus faecalis (group D) (B) Polymerase chain reaction for
coccus that grows in clusters
(C) Streptococcus agalactiae (group B) antibodies against M protein
(D) A catalase-negative gram-positive
(D) Streptococcus anginosus group (C) ASO antibody titer
bacillus
(E) Staphylococcus aureus (D) Esculin hydrolysis
(E) A double-stranded RNA virus
(E) Antihyaluronic acid antibody titer
9. The single most important method for
7. A primary mechanism responsible for
classifying and speciating streptococci is 11. All of the following statements
the pathogenesis of the boy’s disease
regarding the hyaluronic acid capsule of
(question 6) is (A) Agglutination using antisera against
Streptococcus pyogenes are correct
the cell wall groupspecific substance
(A) A net increase in intracellular cyclic except
(B) Biochemical testing
adenosine monophosphate
(C) Hemolytic properties (α-, β-, (A) It is responsible for the mucoid
(B) Action of M protein
nonhemolytic) appearance of the colonies
(C) Action of IgA1 protease
(D) Capsular swelling (quellung) reaction in vitro.
(D) Action of enterotoxin A
(E) None of the above (B) It is antiphagocytic.
(E) Inactivation of elongation factor 2
(C) It binds to CD44 on human epithelial
10. An 8-year-old girl develops
8. A 40-year-old woman develops severe cells.
Sydenham’s chorea (“St. Vitus dance”)
headache and fever. Her neurologic (D) It is an important virulence factor.
with rapid uncoordinated facial tics and
examination findings are normal. A brain (E) A vaccine against the capsule is
involuntary purposeless movements of her
scan shows a ring-enhancing lesion of the currently available.
extremities, strongly suggestive of acute
left hemisphere. During surgery, a brain
rheumatic fever. She has no other major 12. Enterococci can be distinguished from
abscess is found. Culture of the abscess
manifestations of rheumatic fever nonenterococcal group D streptococci on
fluid grows an anaerobic gram-negative
(carditis, arthritis, subcutaneous nodules, the basis of which of the following
bacillus (Bacteroides fragilis) and a
skin rash). The patient’s throat culture is characteristics?
catalase-negative gram-positive coccus
negative for Streptococcus pyogenes
that on Gram stain is in pairs and chains. (A) γ-Hemolysis
(group A streptococci). However, she, her
The organism is β-hemolytic and forms (B) Esculin hydrolysis
brother, and her mother all had sore
very small colonies (<0.5 mm in diameter). (C) Growth in 6.5% NaCl
(D) Growth in the presence of bile 1. A 20-year-old college student goes to the bedclothes. Rose spots are seen on
(E) Gram stain morphology the student health center because of her trunk. The remainder of the physical
dysuria, frequency, and urgency on examination findings are normal. Blood
13. Which of the following statements
urination for 24 hours. She has recently cultures are done, and an intravenous line
regarding the Streptococcus bovis group is
become sexually active. On urinalysis, is placed. The most likely cause of her
correct?
many polymorphonuclear cells are seen. illness is
(A) They possess Lancefield group D The most likely organism responsible for
(A) Enterotoxigenic Escherichia coli (ETEC)
antigen. these symptoms and signs is
(B) Shigella sonnei
(B) Some strains are vancomycin resistant.
(A) Staphylococcus aureus (C) Salmonella enterica subspecies
(C) Infections caused by these organisms
(B) Streptococcus agalactiae enterica serotype Typhimurium
are benign.
(Salmonella Typhimurium)
(D) All subspecies are PYR positive. (C) Gardnerella vaginalis
(E) All subspecies are β-hemolytic. (D) Lactobacillus species (D) Salmonella enterica subspecies
(E) Escherichia coli enterica serotype Typhi (Salmonella
14. Which of the following genera requires
Typhi)
pyridoxal for growth? 2. A 27-year-old woman is admitted to the
(E) Enteroinvasive Escherichia coli (EIEC)
hospital because of fever, with increasing
(A) Aerococcus
anorexia, headache, weakness, and 3. Blood cultures from the patient in
(B) Granulicatella altered mental status of 2 days’ duration. question 2 grow a non–lactose-fermenting
(C) Enterococcus She works for an airline as a cabin gram-negative bacillus. Which of the
(D) Leuconostoc attendant, flying between the Indian following is likely to be a constituent of
(E) Pediococcus subcontinent and other places in this organism?
Southeast Asia and the West Coast of the
15. Which of the following genera is (A) O antigen 157, H antigen 7 (O157:H7)
United States. Ten days before admission,
typically resistant to vancomycin? (B) Vi antigen (capsule; virulence antigen)
she had a diarrheal illness that lasted for
(C) O antigen 139 (O139)
(A) Aerococcus about 36 hours. She has been constipated
(D) Urease
(B) Gemella for the past 3 days. Her temperature is
(E) K1 (capsular type 1)
(C) Pediococcus 39°C, heart rate is 68 beats/min, blood
(D) Streptococcus pressure is 120/80 mm Hg, and 4. A 37-year-old woman with a history of
(E) Abiotrophia respirations are 18 breaths/min. She urinary tract infections comes to the
knows who she is and where she is but emergency department with burning on
does not know the date. She is picking at urination along with frequency and
urgency. She says her urine smells like young adults who smoked marijuana; the lower abdominal pain, and initially watery
ammonia. The cause of her urinary tract same Salmonella was isolated from a diarrhea. His mother became concerned
infection is likely to be specimen of the marijuana.) By what because the stools are now blood tinged
method did the laboratories determine 24 hours into the illness, and the child
(A) Enterobacter aerogenes
that these isolates were the same? appears quite ill. The mother reports that
(B) Proteus mirabilis
two other children who attend the same
(C) Citrobacter freundii (A) Capsular (K antigen) typing
after-school daycare have recently had
(D) Escherichia coli (B) O antigen and H antigen typing
diarrheal disease, one of whom likewise
(E) Serratia marcescens (C) DNA sequencing
had bloody stools. Which of the following
(D) Sugar fermentation pattern
5. An 18-year-old student has abdominal is the most likely pathogen causing the
determination
cramps and diarrhea. illness in these children?
(E) Decarboxylase reaction pattern
A plate of MacConkey agar is inoculated
determination (A) An enterotoxigenic strain of
and grows gramnegative rods. Triple sugar
Escherichia coli
iron agar is used to screen the isolates for 7. A 43-year-old man with diabetes has a
(B) Salmonella enterica subspecies
salmonellae and shigellae. A result 4-cm nonhealing foot ulcer. Culture of the
enterica serotype Typhi (Salmonella Typhi)
suggesting one of these two pathogens ulcer yields Staphylococcus aureus,
(C) Shigella sonnei
would be Bacteroides fragilis, and a gram-negative
(D) Edwardsiella tarda
bacillus that swarms across the blood agar
(A) Production of urease (E) Klebsiella oxytoca
plate covering the entire surface of the
(B) Motility in the medium
agar after 36 hours. The gram-negative 9. A 5-year-old girl attended a birthday
(C) Inability to ferment lactose and
bacillus is a member of the genus party at a local fast food restaurant. About
sucrose
48 hours later, she developed cramping
(D) Fermentation of glucose (A) Escherichia
abdominal pain and a low-grade fever and
(E) Production of gas in the medium (B) Enterobacter
had five episodes of loose, bloody stools.
(C) Serratia
6. An uncommon serotype of Salmonella She is taken to a local emergency
(D) Salmonella
enterica subspecies enterica was found by department the next evening because the
(E) Proteus
laboratories in the health departments of diarrhea has continued, and she now
adjacent states. The isolates were all from 8. A 4-year-old boy from Kansas City who appears pale and lethargic. On
a small geographic area on either side of recently started attending preschool and presentation, she has a temperature of
the border between the states, suggesting after-school daycare is brought to his 38°C, and she is hypotensive and
a common source for the isolates. (All of pediatrician for a diarrheal illness tachycardic.The abdominal examination
the isolates were from otherwise healthy characterized by fever to 38.2°C, severe reveals tenderness in the lower quadrants.
Laboratory work is remarkable for a serum 11. Which of the following statements later, he is seen in clinic for persistent
creatinine of 2.0 mg/dL, a serum regarding O antigens is correct? symptoms, and he is concerned because
hemoglobin of 8.0 mg/dL, he is beginning to lose weight. Given this
(A) All Enterobacteriaceae possess
thrombocytopenia, and evidence of history, you suspect:
identical O antigens.
hemolysis. What is the most likely
(B) They are found in the polysaccharide (A) Enteroinvasive E coli
pathogen causing this child’s illness?
capsules of enteric bacteria. (B) Salmonella typhi
(A) Escherichia coli O157:H7 (C) They are covalently linked to a (C) Enteropathogenic E coli
(B) Salmonella enterica subspecies polysaccharide core. (D) Shigella flexneri
enterica serotype Typhimurium (D) They do not stimulate an immune (E) Enteroaggregative E coli
(C) Enteropathogenic Escherichia coli response in the host.
14. Heat-labile toxin of ETEC acts by which
(E) They are not important in the
(D) Edwardsiella tarda of the following mechanisms?
pathogenesis of infection cause by enteric
(E) Plesiomonas shigelloides
bacteria. (A) Attachment and effacement
10. A 55-year-old homeless man with 12. Which of the following test methods is (B) Activation of adenylyl cyclase
alcoholism presents with severe the least sensitive procedure for diagnosis (C) Aggregative adherence
multilobar pneumonia. He requires of colitis caused by Shiga toxin–producing (D) Ribosomal dysfunction
intubation and mechanical ventilation. A Escherichia coli? (E) None of the above
Gram stain of his sputum reveals
(A) Culture on sorbitol MacConkey agar 15. A young woman presents with
numerous polymorphonuclear leukocytes
(B) Toxin testing using an enzyme recurrent urinary tract infections caused
and gram-negative rods that appear to
immunoassay by the same Proteus mirabilis strain. What
have a capsule. The organism is a lactose
(C) Cell culture cytotoxin assay using Vero is the major concern?
fermenter on MacConkey agar and is very
cells
mucoid. It is nonmotile and lysine (A) She does not take her medication.
(D) Polymerase chain reaction for
decarboxylase positive. What is the most (B) She is pregnant because pregnant
detection of the genes that encode Shiga
likely organism causing this man’s illness? patients are more susceptible to UTIs.
toxin
(C) She has a bladder or kidney stone.
(A) Serratia marcescens
13. An HIV-positive man recently traveled (D) Her partner is infected.
(B) Enterobacter aerogenes
to the Caribbean for a 2-week vacation. He (E) She has occult diabetes and should
(C) Proteus mirabilis
developed acute watery diarrhea and have a glucose tolerance test.
(D) Klebsiella pneumonia
abdominal pain without fever during the
(E) Morganella morganii Answers
second week of his vacation. Three weeks
1. A sputum culture of a patient with cystic (D) Klebsiella pneumonia also is plated on mannitol salt agar, which
fibrosis grows Pseudomonas aeruginosa (E) Stenotrophomonas maltophilia turns yellow (from the baseline pink)
that form very mucoid colonies. The where white colonies of grampositive
3. This gram-negative bacillus, which is
implication of this observation is which cocci are growing; the cocci are catalase
oxidase positive and does
one of the following? positive and
not ferment carbohydrates, is frequently
(A) The P aeruginosa are highly susceptible coagulase positive. The microorganisms
found in human bite infections.
to the aminoglycoside antimicrobial growing on the mannitol salt agar are
tobramycin. (A) Escherichia coli
(A) Burkholderia cepacia
(B) The P aeruginosa is infected with a (B) Neisseria meningitides
(B) Streptococcus pneumonia
pyocin (a bacteriocin). (C) Chromobacterium violaceum
(C) Stenotrophomonas maltophilia
(C) The colonies are mucoid because they (D) Eikenella corrodens
(D) Staphylococcus aureus
have polysaccharide capsule of hyaluronic (E) Proteus mirabilis
(E) Streptococcus pyogenes
acid.
(D) The exotoxin A gene has been disabled 4. A 17-year-old girl with cystic fibrosis has
6. The sputum from the 17-year-old
and the P aeruginosa are no longer able to a slight increase in her frequent cough and
patient with cystic fibrosis (question 4)
block host cell protein synthesis. production of mucoid sputum. A sputum
also is plated on a colistin-containing agar.
(E) The P aeruginosa have formed a specimen is obtained and plated on
After 72 hours of incubation, the colistin-
biofilm in the patient’s airway. routine culture media. The predominant
containing agar grows gram-negative
growths are gram-negative bacilli that
bacilli that are oxidase positive but are
2. An environmental gram-negative form very mucoid colonies after 48 hours
otherwise difficult to identify. This
bacillus that is resistant to cephalosporins, of incubation. These bacilli are oxidase
microorganism is of major concern. It is
aminoglycosides, and quinolones has positive, grow at 42°C, and have a
sent to a reference laboratory so that
become a very important nosocomial grapelike odor. These gram-negative bacilli
molecular methods can be used to identify
pathogen largely because it is selected by are which of the following?
or rule out which of the following?
use of those antibiotics. This gram-
negative bacillus can take 2–3 days to (A) Klebsiella pneumonia
(A) Pseudomonas aeruginosa
grow and must be differentiated from (B) Pseudomonas aeruginosa
(B) Burkholderia cepacia
Burkholderia cepacia. It is (C) Staphylococcus aureus
(C) Haemophilus influenza
(D) Streptococcus pneumonia
(D) Pseudomonas putida
(A) Pseudomonas aeruginosa (E) Burkholderia cepacia
(E) Burkholderia pseudomallei
(B) Acinetobacter baumannii
(C) Alcaligenes xylosoxidans 5. The sputum from the 17-year-old
patient with cystic fibrosis (question 4)
7. When a Burkholderia cepacia complex (D) They can mimic the morphology of (A) Aquamarine green
organism is isolated from a patient with Neisseria species in Gram stains of (B) Aerobactin
cystic fibrosis, great care must be taken in endocervical secretions to diagnose (C) Enterochelin
identifying the organism for which one of gonorrhea in women.
(D) Pyoverdin
the following reasons?
(E) They can be a significant cause of (E) Prodigiosin
(A) B cepacia is commonly susceptible to ventilator-associated pneumonia in
11. Burkholderia cepacia is infrequently
penicillin G, but other similar gram- intensive care unit patients.
found in or on
negative bacilli are not. (F) All of the above.
(B) The presence of B cepacia complex in (A) Swimming pools
9. A 37-year-old firefighter sustains smoke
a CF patient’s airway has major (B) Soil
inhalation and is hospitalized for
implications for the patient’s long-term (C) Pond water
ventilatory support. He has a severe cough
prognosis and therapeutic options. (D) Plants
and begins to expectorate purulent
(C) Only B cepacia produces biofilms.
sputum. Gram stain of his sputum 12. Which of the following statements
(D) B cepacia complex produces an
specimen shows numerous about Pseudomonas aeruginosa is
enzyme, sputolysase, which liquefies
polymorphonuclear cells and numerous correct?
sputum, making it easier for the patient to
gram-negative rods. Sputum culture grows
cough and clear the airway. (A) P aeruginosa are typically susceptible
numerous gram-negative rods that are
(E) Selective media for Pseudomonas to penicillin G.
oxidase-positive. They grow well at 42°C.
aeruginosa commonly used for CF (B) P aeruginosa are readily grown in
On clear agar medium, they produce a
patients’ sputum cultures typically inhibit anaerobic blood cultures.
green color in the agar. The agar where
the B cepacia complex organisms, making (C) P aeruginosa are able to penetrate
the green color is located fluoresces when
it difficult to identify them. intact normal human skin by elaborating
exposed to ultraviolet light. The organism
8. Which of the following statements causing the patient’s infection is the enzyme invasin.
about Acinetobacter species is (are) true? (D) P aeruginosa seldom, if ever, cause
(A) Pseudomonas aeruginosa pneumonia.
(A) They are widespread in nature and in (B) Klebsiella pneumonia (E) P aeruginosa have fimbriae, which
the hospital environment. (C) Escherichia coli promote attachment to epithelial cells.
(B) They are generally nonpathogenic to (D) Burkholderia cepacia
healthy individuals. (E) Burkholderia pseudomallei 13. The mechanism of action of exotoxin A
(C) They may appear as gram-positive of Pseudomonas aeruginosa is
10. The pigment produced by the
cocci.
microorganism in question 9 is
(A) To activate acetylcholine esteras (C) Stenotrophomonas maltophilia (C) Salmonella typhi
(B) To block elongation factor 2 (D) Alcaligenes species (D) Clostridium perfringens
(C) To form pores in white blood cells and (E) Achromobacter species (E) Streptococcus pyogenes (group A
increase cation permeability streptococci)
1. Long-term carriage and shedding is
(D) To increase intracellular cyclic most likely to occur after gastrointestinal 3. A 10-year-old boy was playing in a
adenosine monophosphate infection with which of the following slowly moving stream when he cut his foot
(E) To split lecithin into phosphorylcholine species? on a sharp object. Three days later he was
and diacylglycerol (A) Escherichia coli O157:H7 brought to the emergency department
(B) Shigella dysenteriae because of pain and swelling at the site of
14. The HACEK bacteria sometimes cause
(C) Vibrio cholera the wound and drainage of pus from it.
indolent endocarditis or other infections.
(D) Campylobacter jejuni The most likely cause of the infection is
This acronym represents which of the
(E) Salmonella typhi
following? (A) Vibrio vulnificus
2. A 63-year-old man visited his favorite (B) Escherichia coli
(A) Cardiobacterium hominis
oyster restaurant in a small town on the (C) Aeromonas hydrophila
(B) Eikenella corrodens
eastern shore of the Gulf Coast of Texas. (D) Proteus mirabilis
(C) Kingella kingae
He ate two dozen oysters. Two days later, (E) Salmonella typhimurium
(D) Aggregatibacter aphrophilus (formerly
he was admitted to the hospital because
Haemophilus aphrophilus) 4. A family of four persons ate a meal that
of an abrupt onset of chills, fever, and
(E) All of the above included undercooked chicken. Within 3
lightheadedness when he stood up. (In the
days, three members developed an illness
15. A 30-year-old man, a dog owner, emergency department (ED), his blood
characterized by fever, headache, myalgia,
presents with fever to 39° C, hypotension, pressure was 60/40 mm Hg.) While in the
and malaise. Two of the patients had
and early signs of disseminated ED, he developed erythematous skin
concomitant diarrhea and abdominal pain.
intravascular coagulation. There is a left lesions. These rapidly evolved into
The third person developed diarrhea after
upper quadrant scar on physical hemorrhagic bullae, which then formed
the systemic symptoms had cleared. Stool
examination, petechial lesions on the legs, ulcers. The man drank a six-pack of beer
cultures grew Campylobacter jejuni.
and evidence of scratches from his dog on and one half-bottle of whisky each day.A
Which of the following culture conditions
the arms and legs. Sepsis is suspected. The microorganism of major concern for this
was most likely used to isolate C jejuni?
most likely pathogen is patient is
(A) Thiosulfate-citrate-bile-sucrose
(A) Neisseria meningitides (A) Vibrio vulnificus
medium incubated at 37°C in 5% oxygen
(B) Capnocytophaga canimorsus (B) Escherichia coli
and 10% CO2
(B) Salmonella-Shigella selective medium some patients, it was bloody. The diarrhea (A) Campylobacter jejuni
incubated at 37°C in ambient air usually had an onset within 24 hours after (B) Enterotoxigenic Escherichia coli
(C) MacConkey agar and Hektoen enteric eating the shellfish. Stool cultures typically (C) Salmonella typhimurium
agar incubated at 42°C in 5% oxygen and yielded a pathogenic gram-negative (D) Vibrio cholera
10% CO2 bacillus. The microorganism of concern in (E) Shigella dysenteriae
(D) 5% sheep blood agar incubated at 37°C this setting is
9. Age and geography are major factors in
in ambient air
(A) Enterotoxigenic Escherichia coli the prevalence of colonization by
(E) A medium containing vancomycin,
(B) Vibrio cholera Helicobacter pylori. In developing
polymyxin B, and trimethoprim incubated
(C) Enterohemorrhagic Escherichia coli countries, the prevalence of colonization
at 42°C in 5% oxygen and 10% CO2
(D) Vibrio parahaemolyticus may be greater than 80% in adults. In the
(E) Shigella dysenteriae United States, the prevalence of
5. Bacteremia associated with a colonization with this microorganism in
7. A patient presents to the emergency
gastrointestinal infection is most likely to adults older than age 60 years is
department with nonbloody diarrhea for
occur with which of the following?
12 hours. The patient lives in Washington, (A) 1–2%
(A) Salmonella typhi DC, and has not recently traveled out of (B) 5–10%
(B) Vibrio cholera the area. Which one of the following is (C) 15–20%
(C) Shigella boydii unlikely to be the cause of your patient’s (D) 40–60%
(D) Vibrio parahaemolyticus diarrhea? (E) 80–95%
(E) Campylobacter jejuni
(A) Salmonella typhimurium 10. A 59-year-old man comes to the
6. During the El Niño years in the mid- to (B) Campylobacter jejuni emergency department in the afternoon
late 1990s, the waters of Puget Sound (C) Shigella sonnei because of acute swelling and pain in his
between Washington State and British (D) Vibrio cholerae right leg. Earlier that morning, he had
Columbia warmed considerably. During been working on a small sport fishing boat
8. An 18-year-old woman in rural
this time, many people who ate clams and in an estuary on the Gulf Coast of Texas.
Bangladesh develops profuse (8 L/d)
oysters from these waters became ill with While walking around the boat in shallow
diarrhea. She has no symptoms other than
a disease characterized by explosive water, he scratched his leg, breaking the
the diarrhea and the manifestations of the
diarrhea and moderately severe skin at the site of the current pain and
fluid and electrolyte loss caused by the
abdominal cramps. The diarrhea was swelling. He was not wearing boots. About
diarrhea. The most likely cause of her
usually watery, but in 1 hour after the injury, the scratch became
diarrhea is
red and painful. Swelling developed.
Within 3 hours, the leg below the knee (A) Banning the sale of apples at the local (B) The presence or absence on endotoxin
had become markedly swollen. The skin markets in the cell wall.
was red and tender. There was serous (B) Removing the handle of the Broad (C) Production or lack of production of a
drainage from the wound, which had Street water pump heat-labile enterotoxin
ulcerated and was now much enlarged. (C) Stopping the sale of shellfish imported (D) Positivity or negativity on the
Near the wound, bullae were forming— from Normandy enterocyte invasion test
the largest approximately 2.5 cm in (D) Pasteurizing milk
15. In the United States, public health
diameter. The most likely cause of this
13. A 45-year-old man develops a gastric officials often warn people to thoroughly
medical emergency is
ulcer that can be visualized on a contrast cook chicken commercially obtained from
(A) Staphylococcus aureus medium–enhanced radiograph of his supermarkets and stores. What
(B) Streptococcus pyogenes stomach. A biopsy specimen is taken from percentage of chickens obtained from
(C) Clostridium perfringens the gastric mucosa at the site of the ulcer. these sources are likely to be
(D) Escherichia coli A presumptive diagnosis can be reached contaminated with Campylobacter jejuni?
(E) Vibrio vulnificus most rapidly by inoculating part of the
(A) 1–5%
specimen on which of the following?
11. The Vibrio cholerae factor responsible (B) 6–15%
for diarrhea is a toxin that (A) A medium used to detect urease (C) 15–30%
incubated at 37°C (D) 30–50%
(A) Blocks EF-2
(B) A medium containing vancomycin, (E) Greater than 50%
(B) Yields increased intracellular levels of
polymyxin B, and trimethoprim incubated
cAMP 1. A 68-year-old woman was seen in the
at 42°C
(C) Cleaves SNARE clinic because she had felt feverish and
(C) MacConkey agar medium incubated at
(D) Blocks EF-1-dependent binding of had been experiencing increasing pain and
37°C
amino-acyl-tRNA to ribosomes
(D) Thiosulfate-citrate-bile-sucrose swelling in her left knee during the past 3
(E) Cleaves VAMP
medium incubated at 42°C weeks. Four years earlier, a prosthetic
12. In September 1854, a severe epidemic (E) Blood agar medium incubated at 37°C joint had been placed in her left knee. On
of cholera occurred in the Soho/Golden examination, the knee was swollen, and
14. Which of the following is useful in
Square area of London. Dr. John Snow, a fluid could be detected. An aspirate of the
differentiating Vibrio species from
father of epidemiology, studied the fluid was obtained. There were 15,000
Aeromonas species?
epidemic and helped stop it by which of polymorphonuclear cells/mL in the fluid.
the following actions? (A) Growth or no growth on a medium No organisms were seen on Gram stain. A
containing NaCl routine culture was done. On the fourth
day of incubation, colorless colonies (D) Fresh orange juice polysaccharide–protein conjugate
smaller than 1 mm in diameter were seen (E) Green tea vaccines?
on the blood and chocolate agar plates.
3. A 55-year-old game warden in Vermont (A) The conjugate protein is diphtheria
The organism was a tiny gram-negative
found a dead muskrat on the bank of a toxoid, and the goal is for the infant to
coccobacillus that was catalase positive
stream. He picked up the animal, thinking develop simultaneous immunity to
and oxidase positive. A urea slant was
it might have been illegally trapped or diphtheria.
inoculated and was positive for urease
shot; it was not, and the game warden (B) Infants 2 months to 2 years of age do
activity after overnight incubation. The
buried it. Four days later, he developed a not immunologically respond to
patient was probably infected with which
1.5-cm painful ulcer on the index finger of polysaccharide vaccines that are not
of the following microorganisms?
his right hand, a 1-cm ulcer on his right conjugated to a protein.
(A) Haemophilus influenza forehead, and pain in his right axilla. (C) The conjugate vaccine is designed for
(B) Haemophilus ducreyi Physical examination also revealed right older children and adults as well as
(C) Francisella tularensis axillary lymphadenopathy. This patient is infants.
(D) Brucella species most likely infected with (D) Maternal (transplacental) antibodies
(E) Staphylococcus aureus against Haemophilus influenzae are gone
(A) Brucella species
from the infant’s circulation by 2 months
2. After the culture (question 1) turned (B) Rickettsia rickettsia
of age.
positive, additional history (C) Salmonella Typhi
(E) None of the above
was obtained. Approximately 4 weeks (D) Haemophilus ducreyi
before the onset of her knee pain, the (E) Francisella tularensis 5. An 11-year-old boy from Peru was
patient had visited relatives in Israel and referred to the Brain Tumor Institute.
4. An 18-month-old boy has been playing
traveled Three months earlier he had developed
with a child who develops Haemophilus
headaches
to other countries in the Mediterranean influenzae meningitis. The boy’s parents
area. She had a particular fondness for consult his pediatrician, who says she is and then slowly progressive right-sided
one food product that was the probable comfortable that the child will be fine weakness. A CT scan showed a mass lesion
vehicle for her infection. The product most because he has been fully immunized with in the left hemisphere. He was thought to
likely was the polyribitol ribose phosphate (PRP)– have a brain tumor. A lumbar puncture
protein conjugate vaccine. For what was not done because of concern about
(A) Bananas
reason is it necessary to immunize infants increased intracranial pressure and brain
(B) Unpasteurized goat’s cheese
of 2 months to 2 years of age with herniation through the tentorium
(C) Rare hamburger
cerebelli. During surgery, a mass lesion in
the left hemisphere was found. Frozen this thirdgeneration cephalosporin used (A) Haemophilus influenza
sections of the tissue were done while the rather than ampicillin? (B) Haemophilus ducreyi
patient was in the operating room. (C) Aggregatibacter aphrophilus
(A) About 80% of H influenzae organisms
Microscopy of the sections showed a (D) Actinobacillus hominis
have modified penicillin-binding proteins
granulomatous inflammatory reaction. No (E) Haemophilus parainfluenzae
that confer resistance to ampicillin.
tumor was seen. Tissue was submitted for
(B) The drug of choice, trimethoprim– 8. All of the following statements
culture for Mycobacterium tuberculosis.
sulfamethoxazole, cannot be used because regarding acellular pertussis vaccines are
Middlebrook 7H9 broth medium was used.
the child is allergic to sulfonamides. correct except
Six days after the culture was set up, the
(C) It is easier to administer intravenous
automated machine detected that the (A) All formulations of the vaccine contain
cefotaxime than intravenous ampicillin.
culture result was positive. Results of an at least two antigens
(D) There is concern that the child will
acid-fast stain and a Gram stain were both (B) The acellular vaccine has replaced the
rapidly develop a penicillin (ampicillin)
negative. Subcultures were done. Two whole cell vaccine in the childhood vaccine
allergy.
days later, very small colonies were seen series.
(E) About 20% of H influenzae organisms
on the sheep blood agar plate.The (C) All children should receive five doses of
have a plasmid that encodes for beta-
organism was a tiny gram-negative the vaccine before school entry.
lactamase.
coccobacillus that was catalase positive (D) The vaccine is approved only for
and oxidase positive. It showed urease 7. A 55-year-old man with severe dental young children and adolescents.
activity after 2 hours of incubation on caries presented with 1 month of fever, (E) The vaccine is safer than and as
urea-containing medium. This child had malaise, and back pain and now presents immunogenic as wholecell vaccines.
infection with with moderately severe shortness of
9. Which of the following subspecies of
breath. The examination reveals a febrile
(A) Brucella species Francisella tularensis is the most virulent
man who appears pale and dyspneic.
(B) Mycobacterium tuberculosis for humans?
Other physical findings include
(C) Francisella tularensis conjunctival petechiae, a grade III/VI (A) tularensis
(D) Haemophilus influenza systolic murmur, and an enlarged spleen. (B) holarctica
(E) Moraxella catarrhalis Blood cultures grow a pleomorphic gram-
negative rod that is not hemolytic and that (C) mediasiatica
6. A 3-year-old child develops (D) novicida
when tested is X and V factor negative.
Haemophilus influenzae meningitis.
The most likely causative pathogen is 10. All of the following statements
Therapy is begun with cefotaxime. Why is
regarding the etiologic agent of chancroid
are correct except
(A) The organism is a small gram-negative (C) An A/B structured toxin complaining of fever, pain in his left groin,
rod. (D) A heat-labile toxin and diarrhea for the past 2 days. On
(B) The organism requires X factor but not (E) A neuraminidase examination, he was afebrile, had a pulse
V factor. rate of 126 beats/min, a respiratory rate
13. All of the following cause zoonotic
(C) The organism grows well on standard of 20 breaths/min, and a blood pressure of
infections except
chocolate agar. 130/80 mm Hg. Left groin swelling and
(D) On Gram stain of lesions, the organism (A) Francisella tularensis tenderness were noted. A groin muscle
occurs in strands. (B) Brucella melitensis strain was diagnosed, attributed to a fall 2
(E) The organism is susceptible to (C) Bordetella pertussis days earlier. He was treated with
erythromycin. (D) Bacillus anthracis nonsteroidal anti-inflammatory drugs and
(E) Leptospira interrogans released. The next day, the patient
11. A 3-month-old infant is brought to the
reported feeling weak, had difficulty
pediatric emergency department in severe 14. Which of the following is not a breathing, and collapsed while taking a
respiratory distress. The child appears recognized virulence factor of Bordetella shower. He was transported to a hospital
dehydrated, and there is a prominent pertussis? ED and pronounced dead shortly after
peripheral lymphocytosis. The chest
(A) Heat-labile toxin arrival.
radiograph reveals perihilar infiltrates. The
child’s grandmother, who watches the (B) Filamentous hemagglutinin Cultures of blood samples obtained in the
infant now that the mother has returned (C) Tracheal cytotoxin ED were positive form Yersinia pestis. An
to work, has had a dry hacking cough for (D) Pertussis toxin epidemiologic investigation indicated that
about 2 weeks. The most likely causative (E) Dermonecrotic toxin the patient most likely became infected as
agent is 15. Which of the following pathogens a result of bites by Yersinia pestis–infected
discussed in this chapter is on the select fleas while walking through a prairie dog
(A) Haemophilus influenzae type b
agent list? colony (see Chapter 48.) Which of the
(B) Bordetella pertussis
following statements about the
(C) Streptococcus agalactiae (A) Haemophilus influenza pathogenesis of plague is correct?
(D) Chlamydia pneumonia (B) Aggregatibacter aphrophilus
(E) Bordetella bronchiseptica (C) Bordetella pertussis
(D) Francisella tularensis (A) Y pestis produces a coagulase when
12. In question 11, the factor responsible
(E) All of the above incubated at 28°C.
for the profound lymphocytosis is
(B) There is no risk for pneumonia caused
(A) A hemagglutinin 1. An 18-year-old male resident of Arizona by person-to-person transmission of Y
(B) A polysaccharide capsule came to the emergency department (ED) pestis.
(C) Y pestis organisms multiply in 4. Which of the following is generally not 7. In a patient who has the bubonic form
polymorphonuclear cells. considered a potential agent of of plague, all of the following specimens
(D) After the bite of an infected flea, Y bioterrorism and biologic warfare? are acceptable for diagnosis except
pestis infection seldom,
(A) Yersinia pestis (A) Stool culture on hektoen enteric agar
if ever, disseminates beyond the site of
(B) Botulinum toxin (B) Blood culture using routine laboratory
the flea bite and the regional lymph
(C) Streptococcus pyogenes media
nodes.
(D) Brucella species (C) Culture of a lymph node aspirate on
(E) Y pestis is transmitted to animals (and
(E) Bacillus anthracis blood and MacConkey agars
humans) in flea feces excreted when the
(D) Acute and convalescent serology
flea is feeding. 5. An 8-year-old boy was bitten by a stray
(E) Immunohistochemical staining of
cat. Two days later, the wound was red
2. The drug of choice to treat the patient lymph node tissue
and swollen and drained purulent fluid.
in question 1 would have been
Pasteurella multocida was cultured from 8. All of the following statements
(A) Ampicillin the wound. The drug of choice to treat this regarding the pFra/pMT plasmid of
(B) Cefotaxime infection is Yersinia pestis are true except
(C) Levofloxacin
(A) Amikacin (A) It encodes the capsular protein
(D) Erythromycin (fraction FI) that confers
(B) Erythromycin
(E) Streptomycin
(C) Gentamicin antiphagocytic properties.
3. Yersinia pestis entered North America (D) Penicillin G (B) It contains genes that yield
through San Francisco in the 1890s, (E) Clindamycin plasminogen-activating protease that has
carried by rats on ships that had sailed temperature-dependent coagulase
6. Intimate contacts of patients with
from Hong Kong, where a plague epidemic activity.
suspected plague pneumonia should
occurred. The current reservoir for Y pestis (C) It contains genes that encode
receive which of the following agents as
in the United States is phospholipase D, which is required for
chemoprophylaxis?
organism survival in the flea midgut.
(A) Urban feral cats
(A) Gentamicin (D) It is unique to Y pestis.
(B) Urban rats
(B) Cefazolin (E) It encodes factors that are important
(C) Domestic cows
(C) Rifampin for survival in both the flea and the
(D) Coyotes
(D) Penicillin human.
(E) Rural wild rodents
(E) Doxycycline
9. All of the following statements (C) Hektoen-enteric agar (A) Contact with the patient’s pet cat
regarding the epidemiology of infections (D) Regan-Lowe medium saliva
caused by Yersinia enterocolitica are (E) MacConkey agar (B) Accidental ingestion of prairie dog
correct except feces
12. Which of the following organisms is
(C) Ingestion of contaminated water or
(A) Most human infections are caused by likely to cause a transfusion reaction even
food
serotype O:1. if the donor is asymptomatic?
(D) Direct contact with another infected
(B) Humans acquire the infection from
(A) Pasteurella multocida individual
ingestion of food or drinks contaminated
(B) Escherichia coli (E) Bite of an infected arthropod
by animals or animal products.
(C) Pasteurella bettyae
(C) Person-to-person spread is quite 15. An organism suspected of being
(D) Yersinia enterocolitica
common. Yersinia pestis is recovered from a patient
(E) None of the above
(D) A large inoculum is required to cause with sepsis. The isolate has bipolar
infection. 13. A 25-year-old graduate student is staining is
(E) Infection is more prevalent in persons rushed to the operating room for fever,
catalase positive but is oxidase and urease
with histocompatibility antigen HLA-B27. acute abdominal pain, and leukocytosis
negative and is nonmotile. At this point,
suggestive
10. Which of the following Pasteurella what should be done?
species has been associated of acute appendicitis. During surgery, the
(A) Nothing; the laboratory has confirmed
appendix appears normal, but numerous,
with infections of the female genital tract the diagnosis.
enlarged mesenteric lymph nodes are
and of newborns? (B) Inoculate the isolate to an
present. A likely diagnosis is
identification kit or automated system for
(A) Pasteurella multocida
(A) Epstein-Barr virus infection causing confirmation.
(B) Pasteurella pneumotropica
atypical infectious mononucleosis (C) Call the police because there is a
(C) Pasteurella ureae
(B) Mesenteric lymphadenitis caused by possible bioterrorism event.
(D) Pasteurella bettyae
Y pseudotuberculosis (D) Send the isolate to the nearest public
11. Optimum recovery of Yersinia (C) Gastrointestinal plague health laboratory for confirmation.
enterocolitica from the stools of patients (D) An unusual presentation of shigellosis (E) Send the isolate to the hospital across
with gastroenteritis requires which of the (E) Lymphadenitis caused by Pasteurella town for sequencing.
following specialized media? pneumotropica
1. The inhabitants of a group of small
(A) Cefsulodin-Irgasan-novobiocin agar 14. A typical source of the infection in the villages in rural subSaharan Africa
(B) Xylose-lysine decarboxylase agar case in question 13 is experienced an epidemic of meningitis.
Ten percent of the people died, most of (D) β-lactamase production and high-level department, she has bilateral tenderness
them younger than the age of 15 years. resistance to tetracycline are both when the uterus is palpated. A mass 2–3
The microorganism that most likely caused mediated by genes on the bacterial cm in diameter is felt on the left,
this epidemic was chromosome. suggestive

(A) Streptococcus agalactiae (group B) 3. A 6-year-old boy develops a fever and of tubo-ovarian abscess. Subsequently,
headache. He is taken to the emergency Neisseria gonorrhoeae is cultured from
(B) Escherichia coli K1 (capsular type 1)
department, where he is noted to have a her endocervix. The diagnosis is
(C) Haemophilus influenzae serotype b
stiff neck, suggesting meningeal irritation. gonococcal pelvic inflammatory disease. A
(D) Neisseria meningitidis serogroup A
A lumbar puncture is done, and culture of common sequela of this infection is
(E) West Nile virus
the cerebrospinal fluid grows Neisseria
(A) Cancer of the cervix
2. A 19-year-old man presented to the meningitidis serogroup B. Which of the
(B) Urethral stricture
clinic with a urethral discharge for the past following should be considered for his
(C) Uterine fibroid tumors
24 hours. Neisseria gonorrhoeae was family (household) members?
(D) Infertility
cultured from the specimen and found to
(A) No prophylaxis or other steps are (E) Vaginal-rectal fistula
be β-lactamase positive and resistant to
necessary.
high levels (≥32 μg/mL) of tetracycline. 5. A 38-year-old vice squad police officer
(B) They should be given N meningitidis
Which of the following statements about comes to the emergency department with
pilin vaccine.
these antimicrobial resistance factors is a chief complaint expressed as follows: “I
(C) They should be given N meningitidis
correct? have disseminated gonococcal infection
serogroup B polysaccharide capsule
again.” He is correct. Cultures of his
(A) β-lactamase production and high-level vaccine.
urethra and knee fluid yield Neisseria
resistance to tetracycline are both (D) They should be given rifampin
gonorrhoeae.He has previously had five
mediated by genes on plasmids. prophylaxis.
episodes of disseminated gonococcal
(B) Whereas β-lactamase production is (E) They should be given sulfonamide
infection. The patient should be evaluated
mediated by a gene on the bacterial prophylaxis.
for
chromosome, high-level tetracycline
4. An 18-year-old woman who reports
resistance is mediated by a gene on a (A) Selective IgA deficiency
unprotected sex with a new partner 2
plasmid. (B) A polymorphonuclear cell chemotactic
weeks previously develops fever and left
(C) Whereas β-lactamase production is defect
lower quadrant abdominal pain with onset
mediated by a gene on a plasmid, high- (C) Deficiency of a late-acting
in association with her menstrual period.
level tetracycline resistance is mediated by complement component C5, C6, C7, or C8
On pelvic examination in the emergency
a gene on the bacterial chromosome. (D) Absent lymphocyte adenosine
deaminase activity who have sex with men in the United for butyrate esterase. What is the most
(E) Myeloperoxidase deficiency States? likely organism causing this man’s illness?

6. Which of the following individuals (A) Single dose of an oral fluoroquinolone (A) Neisseria gonorrhoeae
should routinely receive (B) Seven days of oral doxycycline (B) Neisseria lactamica

vaccination with the conjugate (C) Ceftriaxone given intramuscularly as a (C) Moraxella catarrhalis
meningococcal vaccine? single dose (D) Haemophilus influenza
(D) Spectinomycin given intramuscularly as (E) Neisseria meningitidis
(A) A healthy young adolescent entering
a single dose
high school 11. One major advantage of the conjugate
(E) Seven days of oral amoxicillin
(B) A healthy child entering kindergarten meningococcal vaccines compared with
(C) A 60-year-old man with insulin- 9. Which of the following cell components the polysaccharide vaccine is
dependent diabetes produced by Neisseria gonorrhoeae is
(A) Stimulation of mucosal secretory IgA
(D) A healthy 40-year-old technician who responsible for attachment to host cells?
(B) Fewer side effects
works in a cancer
(A) Lipooligosaccharide (C) A T cell–dependent response to
(E) A 65-year-old woman with coronary
(B) Pili (fimbriae) vaccine is induced
artery disease
(C) IgA1 protease (D) Inclusion of serogroup B
7. A 25-year-old sexually active woman (D) Outer membrane porin protein
12. A 25-year-old woman presents with
presents with purulent vaginal discharge (E) Iron-binding protein
septic arthritis of the knee. The fluid
and dysuria 7 days after having
10. A 60-year-old man with severe chronic aspirate grows a gram-negative
unprotected sexual intercourse with a new
lung disease presents with fever, cough diplococcus on chocolate agar after 48
partner. Of the choices below, what is the
productive of purulent sputum, and hours of incubation. The isolate is oxidase
most sensitive diagnostic method for
worsening hypoxemia. A sputum sample is positive and oxidizes glucose but not
determining the likely etiologic agent?
collected, and the specimen is sent maltose, lactose, or sucrose.
(A) Gram stain promptly to the laboratory. Microscopic
You suspect infection with
(B) An enzyme immunoassay examination of a Gram stain reveals
(C) Bacterial culture on selective media numerous polymorphonuclear leukocytes (A) Neisseria meningitides
(D) A nucleic acid amplification test and predominately gram-negative (B) Neisseria lactamica
(E) Serology diplococci that are both intracellular and (C) Moraxella catarrhalis
extracellular. The organism grows well on (D) Neisseria gonorrhoeae
8. What is the currently recommended
5% SBA and chocolate agar and is positive (E) None of the above
treatment for gonococcal urethritis in men
13. All of the following are virulence 1. A 55-year-old man visits his physician suggest infection with anaerobic bacteria
factors associated with N gonorrhoeae complaining of a severe cough and include
except production of purulent sputum. His breath
(A) Negative aerobic culture results
has a
(A) Pili (B) Gas in tissues
very unpleasant fetid odor. Chest (C) Proximity to the mucosal surface
(B) Por
radiography shows a large amount of fluid
(C) Lipooligosaccharide (D) Foul-smelling discharge
in the left pleural space and a 5-cm lung
(D) Opa proteins (E) All of the above
cavity with an air-fluid level. A needle is
(E) A thick polysaccharide capsule
inserted through the chest wall, and some 3. A 63-year-old man with diabetes
14. The prevalence of gonococcal of the fluid in the pleural space is routinely injects insulin into the muscles of
infections increased between 2009 and removed; it is thick, yellow-gray in color, his left thigh. He has recently developed
2010. and malodorous. Which of the following severe pain with swelling in his left thigh.
organisms or sets of organisms are most On examination, his thigh is swollen and
(A) True likely to be cultured from the pleural fluid? red. Crepitus is noted on palpation,
(B) False
indicating gas in the tissue. Gas also is
(A) Bacteroides fragilis, Escherichia coli,
15. A useful test to differentiate Moraxella visible in the fascial planes on radiography
and enterococci
catarrhalis from saprophytic neisseriae in of the leg. Gas gangrene caused by
(B) Prevotella bivia, peptostreptococci,
respiratory samples is Clostridium perfringens is considered a
and Staphylococcus epidermidis
likely diagnosis. What other infections
(A) Butyrate esterase (C) Prevotella melaninogenica,
must be considered?
(B) Gram stain Fusobacterium species, and viridans
(C) Growth on 5% sheep blood agar streptococci (A) Anaerobic streptococcal myonecrosis
(D) PYR (D) Propionibacterium species, (B) Synergistic nonclostridial anaerobic
(E) Oxidase peptostreptococci, and Staphylococcus myonecrosis
aureus (C) Infected vascular gangrene
Answers (E) Streptococcus pneumoniae (D) Aeromonas hydrophila myonecrosis
1. D 5. C 9. B 13. E (E) All of the above
2. A 23-year-old man develops a perirectal
2. A 6. A 10. C 14. A abscess, which is 4. An 18-year-old man develops fever with
drained surgically. A specimen is cultured pain in the right lower
3. D 7. D 11. C 15. A and grows anaerobic bacteria. Clues that quadrant of his abdomen. After initial
4. D 8. C 12. D evaluation, he is taken to the operating
room. During surgery, a ruptured appendix (C) Prevotella melaninogenica 9. The drug of choice for treatment of
with an abscess is found. Bacteroides (D) Clostridium tetani infections caused by Actinomyces species
fragilis is cultured from the abscess fluid. (E) Actinomyces israelii is
Which of the following factors promote
7. Which of the following statements (A) Tigecycline
abscess formation by B fragilis?
regarding Lactobacilli is correct? (B) Cefoxitin
(A) Lipopolysaccharide
(A) They are anaerobic gram-positive (C) Metronidazole
(B) Capsule cocci. (D) Imipenem
(C) Superoxide dismutase (B) They are most commonly found in the (E) Penicillin
(D) Pili oral cavity.
10. Infections commonly caused by
(E) Leukocidin toxin (C) The major product of metabolism is
Clostridium perfringens include all of the
propionic acid.
5. Infections caused by Bacteroides following except
(D) They rarely cause disease in humans.
species can be treated with all of the
(E) They form endospores. (A) Gas gangrene
following antibiotics except
(B) Lumpy jaw
8. Which of the following statements best
(A) Ampicillin–sulbactam (C) Food poisoning
describes the pathogenesis of Clostridium
(B) Clindamycin (D) Bacteremia
botulinum?
(C) Metronidazole
11. All of the following statements
(D) Penicillin (A) It elaborates a toxin that inhibits the
regarding anaerobes are true except
(E) Cefoxitin release of acetylcholine at cholinergic
synapses. (A) They possess the enzyme cytochrome
6. A 17-year-old high school senior
(B) It elaborates an exotoxin that is a oxidase
develops infectious mononucleosis. About
lecithinase that causes tissue necrosis. (B) Many species are part of the normal
2 weeks later, he develops a significantly
(C) It produces a polysaccharide capsule human microbiota
higher fever, a worsening sore throat, an
that inhibits phagocytosis and contributes (C) They are often found along with
inability to swallow, and severe neck and
to invasion of the central nervous system. aerobes in complicated infections
chest pain. Upon admission, he has signs
(D) It elaborates a toxin that suppresses (D) Special techniques are required to
of sepsis and respiratory distress. What is
the release of inhibitory ensure their recovery from clinical
the most likely organism causing this
neurotransmitters. specimens
complication?
(E) It produces a leukotoxin that leads to
(A) Fusobacterium necrophorum abscess formation.
(B) Bacteroides ovatus
12. Lemierre’s disease is a serious (A) Bacteroides fragilis headache, vomiting, and severe migratory
infection of the head and neck associated (B) Lactobacillus acidophilus arthralgias (joint pain) and myalgias
with which of the following anaerobes? (C) Clostridium perfringens (muscle pain). Two days later, she
(D) Actinomyces israelii developed a maculopapular rash over her
(A) Prevotella melaninogenica
(E) Eggerthella lenta palms, soles, and extremities. At the same
(B) Bacteroides thetaiotamicron
time, her left knee became extremely
(C) Porphyromonas gingivalis 15. The recommended treatment in the
painful and swollen. On examination, fluid
(D) Peptococcus nige non-allergic patient for the
was demonstrated in the knee. Further
(E) Fusobacterium necrophorum
condition caused by the organism in history disclosed that the patient had a
13. Definitive identification of an anaerobe question 14 is pet rat that she frequently played with.
is likely best accomplished by Culture of the fluid from her knee on 5%
(A) Ampicillin sheep blood agar showed 2-mm colonies
(A) Colony morphology on anaerobic (B) Cefoxitin after 3 days of incubation. Broth culture
media (C) Imipenem showed small puffballlike growth. Gram
(B) The presence of pigment (D) Clindamycin staining showed a gram-negative bacillus
(C) Susceptibility to a variety of (E) Vancomycin 0.5 μm wide and 1–4 μm long. Some
antimicrobial disks extremely long forms (up to 150 μm) with
1. Humans become infected with
(D) Cell wall fatty acid analysis using gas beadlike chains, fusiform swellings, and
Legionella pneumophila by
liquid chromatography large round bodies were seen. The
(E) None of the above (A) Drinking water contaminated with microbiologist who observed the Gram-
Acanthamoeba castellani containing stained smear immediately knew the
14. A patient who has not maintained
Legionella pneumophila cause of thegirl’s infection to be
good dentition presents with induration
(B) Kissing a person who is a legionella
and swelling of the mandibular area. On (A) Treponema pallidum
carrier
examination, you note purulent material (B) Streptobacillus moniliformis
(C) Breathing aerosols from
draining from a small opening. The (C) Francisella tularensis
environmental water sources
material appears yellowish, and there are (D) Bartonella bacilliformis
(D) Receiving a mosquito bite
some visible granules. You perform a (E) Yersinia pestis
(E) Consuming undercooked pork
Gram stain, and pleomorphic grampositive
rods with short branches are noted along 2. An 11-year-old girl developed an acute 3. A 70-year-old man presents with
with cells suggestive of acute and chronic onset of fever, chills, bilateral pneumonia. His
inflammation. You suspect which of the
following organisms?
Legionella urinary antigen test result is normal flora. Treatment with ampicillin for 7. A 23-year-old woman presents with a 3-
positive. Which of the following is the 2 days yields no improvement. A diagnosis day history of lowgrade fever and
likely cause of his pneumonia? of Legionnaires’ disease is considered, and headache. Examination reveals enlarged
bronchoscopy is done to obtain bronchial and slightly tender lymph nodes near her
(A) Legionella pneumophila serogroup 1
alveolar lavage fluid and deep airway left elbow and in the left axilla.
(B) Legionella micdadei serogroup 4
specimens. Which of the following would Approximately 2 weeks earlier, she had
(C) Legionella bozemanii serogroup 2
suggest a diagnosis of disease caused by visited a friend whose cat had scratched
(D) Legionella longbeachae serogroup 2
Legionella pneumophila serotype 1? her on the left arm; the site later
(E) All of the above because the urinary
developed a reddish papule. Which of the
antigen test is genus specific and not (A) Legionella urinary antigen assay
following statements about cat-scratch
species or serotype specific (B) Direct fluorescent antibody on the
disease is most correct?
bronchial alveolar lavage fluid
4. A 70-year-old man comes to the
(C) Culture of the bronchial alveolar lavage (A) The diagnosis is based on a suggestive
emergency department feeling feverish
on charcoal yeast extract medium with history and physical examination.
and “really tired.” He has a chronic
antibiotics (B) The diagnosis is based on negative
cigarette cough, but this has dramatically
(D) Antibody assay on paired (acute phase routine bacterial cultures of pus aspirated
increased in the past week and has been
and convalescent phase) sera from involved lymph nodes.
producing whitish sputum. The previous
(E) All of the above (C) The disease is usually self-limited in
day he had a temperature of 38°C and
immunocompetent people.
watery diarrhea. Physical examination 6. Charcoal is present in buffered charcoal
(D) The etiologic agent is Bartonella
reveals inspiratory and expiratory wheezes yeast extract agar used to isolate
henselae.
and rales over the right lower lung field. Legionella pneumophila to
(E) All of the above.
Chest radiography shows a patchy right
(A) Provide the growth factors ordinarily
lower lobe infiltrate. The differential 8. Which of the following statements
provided by freeliving amebas present in
diagnosis of this patient’s disease is about bacillary angiomatosis is most
environmental water
correct?
(A) Streptococcus pneumoniae pneumonia (B) Serve as a carbon source for the
(B) Legionella pneumophila pneumonia growth of Legionella pneumophila (A) It is caused by Bartonella bacilliformis.
(C) Haemophilus influenzae pneumonia (C) Prevent hemolysis of the red blood (B) It is typically confined to the skin.
(D) Mycoplasma pneumoniae pneumonia cells in the medium (C) The major differential diagnosis is
(E) All of the above (D) Provide a dark background Kaposi sarcoma.
(E) Act as a detoxifying agent (D) The etiologic agent can be grown in 1–
5. Routine sputum cultures for the patient
2 days in routine
in question 5 grow
culture on sheep blood agar. 11. All of the statements below regarding cellular invasion.
(E) Dogs are the reservoir for the etiologic infections with Legionella are correct (E) None of the above.
agent. except
13. Pontiac fever is a severe form of
9. An important factor in the pathogenesis (A) Hospitals that care for patients at risk pneumonia caused by Legionella
of Legionnaires disease is that for Legionella infections should know if pneumophila serotypes 1 and 6.
their potable water systems contain
(A) Legionella pneumophila kills (A) True
Legionella.
polymorphonuclear cells. (B) False
(B) Human-to-human transmission is the
(B) Alveolar macrophages phagocytose
major mechanism of transmission of 14. All of the following statements
Legionella pneumophila using coiled
Legionella infection. regarding Streptobacillus moniliformis are
pseudopods.
(C) Legionella species can be visualized correct except
(C) Legionella pneumophila invades
with Gram stain if carbolfuchsin is used for
pulmonary capillaries, leading to (A) It is susceptible to penicillin.
the counter stain.
dissemination and systemic illness. (B) It causes the disease rat-bite fever.
(D) The chest radiograph of a patient who
(D) Legionella pneumophila induces (C) It causes Haverhill fever from ingestion
has Legionella pneumonia is
alveolar macrophage phagosomes to fuse of contaminated food.
indistinguishable from that of patients
with lysosomes. (D) The morphology of the organism is
with pneumonia caused by other
(E) Legionella pneumophila outer surface spiral shaped.
pathogens.
protein A (OspA) is important for invasion
(E) A macrolide or quinolone are the drugs 15. The diagnosis of Whipple’s disease is
of alveolar macrophages.
of first choice for treatment of Legionella best made by
10. True statements regarding T whipplei infections.
include all of the following except (A) Paired serum obtained 8 weeks apart
12. Which of the following best represents (B) Prolonged culture on mycobacterial
(A) It is easy to cultivate on chocolate the role of the Mip protein in Legionella media
agar after 3 days of incubation. pathogenesis? (C) Nucleic acid amplification testing
(B) It is a gram-positive Actinomycete. performed on tissue
(A) It prevents phagosome–lysosome
(C) It causes fever, abdominal pain, (D) Histopathology
fusion
diarrhea, weight loss, and migratory (E) None of the above
(B) It acts as a siderophore to capture iron.
polyarthralgia.
(C) It prevents phagocytosis. 1. A 60-year-old-man has a 5-month
(D) It stains with PAS.
(D) It facilitates adherence to the history of progressive weakness and a
macrophage and stimulates weight loss of 13 kg along with
intermittent fever, chills, and a chronic 3. A 47-year-old woman presents with a 3- (A) Mycobacterium leprae
cough productive of yellow sputum, month history of progressive cough, (B) Mycobacterium fortuitum
occasionally streaked with blood. A weight loss, and fever. Chest radiography (C) Mycobacterium ulcerans
sputum specimen is obtained, and shows bilateral cavitary disease suggestive (D) Mycobacterium gordonae
numerous acid-fast bacteria are seen on of tuberculosis. Sputum culture grows an (E) Mycobacterium tuberculosis
the smear. Culture of the sputum is acid-fast bacillus that is a
positive for Mycobacterium tuberculosis. photochromogen (makes an orange
5. It is very important that the patient in
Which treatment regimen is most pigment when exposed to light). The
question 4 also be evaluated for
appropriate for initial therapy? organism most likely is
(A) HIV/AIDS
(A) Isoniazid and rifampin (A) Mycobacterium tuberculosis
(B) Typhoid fever
(B) Sulfamethoxazole–trimethoprim and (B) Mycobacterium kansasii
(C) Liver abscess
streptomycin (C) Mycobacterium gordonae
(D) Lymphoma
(C) Isoniazid, rifampin, pyrazinamide, and (D) Mycobacterium avium complex
(E) Malaria
ethambutol (E) Mycobacterium fortuitum
(D) Isoniazid, cycloserine, and 6. Of concern regarding the patient in
4. A 31-year-old Asian woman is admitted
ciprofloxacin question 4 is that she could be infected
to the hospital with a 7-week history of
(E) Rifampin and streptomycin with a mycobacterium that is
increasing malaise, myalgia,
2. If the patient’s Mycobacterium nonproductiven cough, and shortness of (A) Susceptible only to isoniazid
tuberculosis isolate (question breath. She has daily fevers of 38–39°C (B) Resistant to streptomycin
and a recent 5-kg weight loss. She had a (C) Resistant to clarithromycin
1) proves to be resistant to isoniazid, the
negative chest radiograph when she (D) Susceptible only to ciprofloxacin
likely mechanism for resistance is
entered the United States 7 years ago. The (E) Resistant to isoniazid and rifampin
(A) β-lactamase
patient’s grandmother died of tuberculosis 7. You observe a 40-year-old man begging
(B) Mutations in the catalase-peroxidase when the patient was an infant. A current on a street in a town in India. He has
gene chest radiograph is normal; results of clawing of the fourth and fifth digits with
(C) Alterations in the β subunit of RNA other tests show a decreased hematocrit loss of distal parts of the digits of both
polymerase and liver function test abnormalities. Liver hands, strongly suggesting leprosy. The
(D) Mutations in the DNA gyrase gene and bone marrow biopsies show causative agent of this disease
(E) Mutations in the genes encoding the granulomas with giant cells and acid-fast
bacilli. She is probably infected with (A) Is susceptible to isoniazid and rifampin
S12 protein and 16S rRNA
(B) Grows in parts of the body that are
cooler than 37°C joint disease. One week after the (D) In primary tuberculosis, all of the
(C) Can be cultured in the laboratory using procedure, she has fever and joint pain. infecting M tuberculosis organisms are
Middlebrook 7H11 medium The hip is aspirated, and the fluid is killed by the patient’s immune response.
(D) Is seen in high numbers in biopsies of submitted for routine culture and for (E) In primary tuberculosis, the immune
tuberculoid leprosy lesions culture for acid-fast organisms. After 2 system is primed, but the PPD skin test
(E) Commonly infects people in Texas days of incubation, there is no growth on result remains negative until there is a
because armadillos are hosts of any of the media. After 4 days, however, second exposure to M tuberculosis.
Mycobacterium leprae bacilli are seen growing on the sheep
11. Which of the following statements
blood agar plate, and similar-appearing
8. Which of the following statements regarding interferon-gamma release
acidfast bacilli are growing on the culture
about the purified protein derivative (PPD) assays is correct?
for acid-fast bacteria. The patient is most
and the tuberculin skin test is most
likely infected with (A) They are useful for evaluating
correct?
immunocompromised patients for active
(A) Mycobacterium tuberculosis
(A) It is strongly recommended that tuberculosis.
(B) Mycobacterium chelonae
medical and other health science students (B) They detect antigens present in all
(C) Mycobacterium leprae
have PPD skin tests every 5 years. Mycobacterium species.
(D) Mycobacterium kansasii
(B) Persons immunized with BCG rarely, if (C) They are not available yet for testing in
(E) Mycobacterium avium complex
ever, convert to positive PPD skin test the United States.
results. 10. A 10-year-old child has a primary (D) They are performed using molecular
(C) The intradermal skin test is usually pulmonary Mycobacterium tuberculosis probes that detect organism DNA.
read 4 hours after being applied. infection. Which of the following features (E) They are used as alternatives to the
(D) A positive tuberculin test result of tuberculosis is most correct? tuberculin skin test to evaluate for latent
indicates that an individual has been tuberculosis.
(A) In primary tuberculosis, an active
infected with Mycobacterium
exudative lesion develops and rapidly 12. Mycobacterium abscessus most often
tuberculosis in the past and may continue
spreads to lymphatics and regional lymph causes pulmonary disease among which
to carry viable mycobacteria.
nodes. group of individuals?
(E) A positive PPD skin test result implies
(B) The exudative lesion of primary
that a person is immune to active (A) Young children exposed to dirt
tuberculosis often heals slowly.
tuberculosis. (B) African American smokers
(C) If tuberculosis develops years later, it is
(C) Elderly, nonsmoking white females
9. A 72-year-old woman has an artificial a result of another exposure to M
(D) Hispanic men who work outdoors
hip joint placed because of degenerative tuberculosis.
(E) Persons living in the Northwestern for prenatal care. She has a history of painless. What is the most likely
United States treatment for syphilis 7 years previously. pathogen?
The results of serologic tests for syphilis
13. A newly characterized rapidly growing (A) Adenovirus infection
are as follows: nontreponemal test, RPR,
mycobacterium that has emerged as an (B) Papilloma virus infection
nonreactive; treponemal test (TP-PA),
important cause of central venous (C) Neisseria gonorrhoeae infection
reactive. Which of the following
catheter associated infections is (D) Chlamydia trachomatis cervicitis
statements is most correct?
(E) Treponema pallidum infection
(A) Mycobacterium phlei
(A) The mother’s previous treatment for
(B) Mycobacterium mucogenicum 4. A 42-year-old woman went camping in
syphilis was effective.
(C) Mycobacterium xenopi the Sierra Nevada Mountains, where she
(B) The baby is at high risk for congenital
(D) Mycobacterium smegmatis slept for two nights in an abandoned log
syphilis.
(E) Mycobacterium terrae cabin. After the second night, a tick was
(C) The mother needs to be treated again
found on her shoulder. Six days later, she
14. The definition of extensively drug for syphilis.
developed fever to 38°C, which lasted for
resistant (XDR) tuberculosis includes (D) The mother needs a lumbar puncture
4 days. Ten days later, she had another
and a VDRL test of her CSF for
(A) Resistance to isoniazid similar episode of
neurosyphilis.
(B) Resistance to a fluoroquinolone
fever. Examination of a blood smear
(C) Resistance to capreomycin, amikacin or 2. Infections with which of the following
stained with Wright stain showed
kanamycin agents can result in a false-positive
spirochetes suggestive of Borrelia species.
(D) Resistance to rifampin nontreponemal (VDRL or RPR) test result
Which of the following statements about
(E) All of the above for syphilis?
relapsing fever and Borrelia hermsii is
15. All of the following organisms are (A) Lupus erythematosus correct?
rapidly growing mycobacteria except (B) Measles
(A) Each relapse is associated with an
(C) Leprosy
(A) Mycobacterium fortuitum antigenically distinct variant
(D) Pregnancy
(B) Mycobacterium abscessus (B) Blood smears should be made when
(E) Blood transfusions
(C) Mycobacterium mucogenicum the patient is afebrile.
(F) Malaria
(D) Mycobacterium nonchromogenicum (C) Borreliae do not pass transovarially
(G) All of the above
(E) Mycobacterium chelonae from one generation to the next in ticks.
3. A 20-year-old woman presents with a 2- (D) The main reservoir for the Borrelia is
1. A 28-year-old woman who is 10 weeks cm ulcer on her labia majora. The lesion deer.
pregnant presents to the obstetrics clinic has a raised border and is relatively
(E) Borrelia hermsii is resistant to penicillin frequently gone swimming in an irrigation result of an EIA for Lyme borreliosis is
and tetracycline. canal that bordered land where cows were positive. What should be done now?
pastured. Blood tests done shortly after
5. A 23-year-old man presented with a (A) A biopsy specimen of the synovium of
admission indicated renal function
maculopapular rash over much of his a knee joint should be examined for
abnormality and elevated bilirubin and
trunk but not in his mouth or on his palms. Borrelia burgdorferi.
other liver function test results. Routine
Because secondary syphilis was (B) The patient should be given an
blood, urine, and CSF culture results were
considered in the differential diagnosis, a antibiotic to treat Lyme disease.
negative. Leptospirosis is suspected.
RPR test was done, and the result was (C) PCR on the patient’s plasma should be
Which of the following would be most
positive at a 1:2 dilution. However, the TP- done to detect Borrelia burgdorferi.
likely to confirm this diagnosis?
PA test result was negative. Which of the (D) A serum specimen should be
following diseases can be ruled out? (A) Testing acute and convalescent phase submitted for immunoblot assay to
sera using the RPR test detect antibodies reactive with Borrelia
(A) Secondary syphilis
(B) Culture of urine on human diploid burgdorferi antigens.
(B) Atypical measles
fibroblast cells
(C) Coxsackie virus infection 9. Which of the following organisms
(C) Testing serum by dark-field
(D) Acute HIV 1 infection principally infects the liver and kidneys?
examination for the presence of
(E) Allergic drug reaction
leptospires (A) Streptobacillus moniliformis
6. Which of the following animals is the
(D) Testing acute and convalescent phase (B) Leptospira interrogans
source of Leptospira interrogans?
sera for antileptospiral antibodies (C) Staphylococcus aureus
(A) Cattle (E) Culture of CSF on blood and chocolate (D) Escherichia coli
(B) Dogs agar (E) Enterococcus faecalis
(C) Mice (F) Treponema pallidum
8. A 47-year-old man presents with slowly
(D) Rats
progressive arthritis in his knees. He 10. Which of the following spirochetes are
(E) Swine
enjoys hiking in the coastal areas of transmitted primarilyby sexual contact?
(F) All of the above
Northern California, where the prevalence
of Borrelia burgdorferi in the Ixodes ticks (A) Treponema pallidum subspecies
7. A 27-year-old medical resident was
is known to be 1–3% (considered low). The pallidum
admitted to the hospital because of
patient is concerned about Lyme disease. (B) Treponema pallidum subspecies
sudden onset of fever to 39°C and
He never noticed a tick on his body and endemicum
headache. Two weeks previously, he had
did not see an expanding red rash. The (C) Borrelia recurrentis
vacationed in rural Oregon, where he had
(D) Borrelia burgdorferi 13. Nucleic acid amplification test results 1. Ureaplasma urealyticum is so named
(E) All of the above are sensitive methods for diagnosing because
neuroborreliosis.
11. Reverse sequence testing for the (A) It thrives in the upper urinary tract.
diagnosis of syphilis (A) True (B) It requires urea as a growth substrate.
(B) False (C) It is a frequent cause of symptomatic
(A) Involves screening with a high-
urinary bladder infections in young
throughput EIA or CIA assay followed by 14. Dark-field microscopy may be used to
women.
confirmation using a nontreponemal test diagnose spirochetes in which of the
(D) It causes chronic urinary tract
(B) May be associated with a high level of following scenarios?
infections in premature
false-positive results
(A) To detect spirochetes in the
(C) May be more sensitive than the babies born to mothers with ureaplasmas
cerebrospinal fluid in a patient with
traditional algorithm in detecting early as part of the genital flora.
tertiary syphilis
disease
(B) To detect spirochetes in a suspicious 2. An 18-year-old sexually active woman
(D) Is replacing the traditional algorithm in
lesion in the oral cavity in a patient with develops left lower quadrant pain and
many laboratories
secondary syphilis fever. On pelvic examination, she has
(E) All of the above
(C) To detect spirochetes in the urine of a tenderness in the left adnexa, and a mass
12. All of the following statements patient with suspected leptospirosis suggestive of a uterine tube abscess is
regarding relapsing fever are palpated. The patient is diagnosed with
(D) To detect spirochetes in the blood of a
pelvic inflammatory disease. Which of the
correct except patient with a positive RPR result but no
following bacteria is considered to be a
symptoms
(A) Epidemic disease carries a higher common cause of pelvic inflammatory
mortality rate than endemic disease. 15. Streptobacillus moniliformis is a cause disease?
(B) Endemic disease is North America is of rat-bite fever. What other pathogen can
(A) Bacillus cereus
caused by B recurrentis. also cause rat-bite fever?
(B) Haemophilus influenza
(C) The recurrent febrile episodes are
(A) Treponema pallidum subspecies (C) Neisseria subflava
caused by antigenic variation among the
pertenue (D) Mycoplasma pneumonia
spirochetes.
(B) Leptospira interrogans (E) Chlamydia trachomatis
(D) Penicillin is the drug of choice.
(C) Borrelia recurrentis
(E) Crushing a tick could transmit the 3. Which of the following is important in
(D) Spirillum minor
spirochetes. the pathogenesis of mycoplasmal
(E) Brachyspira aalborgi
infections?
(A) The peptidoglycan in the mycoplasmal sputum culture results are negative. confirmation of Mycoplasma pneumoniae
cell wall Pneumonia caused by Mycoplasma infection?
(B) The presence of lacto-N-neotetraose pneumoniae is considered. All of the
(A) Culture in broth containing serum,
with a terminal galactosamine as the host following are methods to confirm the
glucose, and a penicillin (to inhibit other
cell receptor clinical suspicion except
flora)
(C) The structures and the interactive
(A) PCR amplification of M pneumoniae (B) PCR
proteins that mediate adhesion to host
DNA in sputum (C) Electron microscopy
cells
(B) Culture of sputum for M pneumonia (D) EIA tests on acute and convalescent
(D) The absence of cilia on the surface of
(C) Gram stain of sputum smear phase sera
the host cells
(D) Culture of a lung aspirate for M 9. A 13-year-old boy develops infection
(E) Growth in an anatomic site where
pneumonia with Mycoplasma pneumoniae. What is
anaerobic organisms thrive
(E) Enzyme immunoassay test of acute and the risk for infection in other members of
4. A 25-year-old woman is referred to the convalescent sera his household?
sexually transmitted diseases clinic
6. Each of the following is associated with (A) None; it is sexually transmitted
because of contact with a male partner
genital tract infections except (B) 1–3%
with gonorrhea. The woman has had 15
(C) 10–15%
male sex partners since becoming sexually (A) Mycoplasma hominis
(D) 20–40%
active. The likelihood that she also has (B) Neisseria gonorrhoeae
(E) 50–90%
genital Mycoplasma hominis infection is (C) Mycoplasma pneumonia
(D) Chlamydia trachomatis 10. A 19-year-old man develops cough and
(A) 1%
(E) Mycoplasma genitalium fever. A chest radiograph shows
(B) 5%
consolidation of the left lower lobe. A
(C) 15% 7. Mycoplasmas have all of the following
diagnosis of pneumonia is made. Which of
(D) 40% characteristics except:
the following bacteria is a frequent cause
(E) 90%
(A) Possession of both DNA and RNA of community-acquired pneumonia?
5. A 25-year-old medical student has (B) Capability for cell-free growth
(A) Legionella pneumophila
contact with a patient who has pneumonia (C) Susceptibility to penicillin G
(B) Chlamydia pneumonia
with fever and cough. Four days later, the (D) Extracellular parasitism in vivo
(C) Streptococcus pneumonia
medical student develops fever and cough,
8. Which type of test is most readily used (D) Mycoplasma pneumonia
and chest radiographs show consolidation
to obtain laboratory (E) All of the above
of the right lower lobe. Routine bacterial
11. All of the following statements (D) Mycoplasma pneumonia (B) Add 15% gelatin to standard blood
regarding mycoplasmas are correct except (E) All of the above culture bottles.
(C) L forms cannot be cultivated from
(A) They are susceptible to β-lactam 14. A 53-year-old man with known
human blood.
agents. prosthetic valve endocarditis presents
(D) Add antibiotics that inhibit other
(B) They lack a cell wall. with fever about 2 weeks after
microbes to standard blood culture media.
(C) They have an affinity for mammalian discontinuation of antibiotics. The
(E) Place the blood into an isolator tube
cell membranes. infectious diseases physician in charge of
and subculture onto Mycoplasma media.
(D) They can be cultivated in cell-free the patient would like the laboratory to
media. look for L forms. Which of 1. Morulae (intracellular inclusions in
(E) Some species are barely larger in size leukocytes) are characteristic of which of
the following statements best
than viruses. the following diseases?
characterizes L forms?
12. Initiation of infection by M (A) Malaria caused by Plasmodium
(A) They are cell wall–deficient forms that
pneumoniae begins with falciparum infection but not Plasmodium
are unequivocally associated with
malariae infection
(A) Elaboration of a polysaccharide capsule infectious endocarditis.
(B) Dengue
that inhibits phagocytosis (B) They are cell wall–defective microbial
(C) Babesia infection
(B) Secretion of a potent exotoxin forms that can replicate serially as
(D) Ehrlichiae infection
(C) Endocytosis by ciliated respiratory nonrigid cells.
(E) Loa loa
epithelial cells (C) All L forms are stable and are different
(D) Adherence to respiratory epithelial from the parent forms. 2. Which of the following statements
cells mediated by P1 adhesin (D) All L forms are derived from gram- about epidemic typhus(Rickettsia
(E) None of the above positive bacteria. prowazekii disease) is most correct?
(E) L forms are genetically related to
13. Which of the following Mollicutes has (A) The disease occurs primarily in sub-
mycoplasmas.
the characteristic “fried egg” appearance Saharan Africa.
on suitable agar media within 5–7 days of 15. In the case above, what is the best way (B) It is transmitted by ticks.
incubation? to recover the L forms (C) Mice are the reservoir.
(D) Historically, the disease occurs in times
(A) Mycoplasma fermentans from blood cultures?
of prosperity.
(B) Mycoplasma orale
(A) Place the blood into an isolator tube (E) Recrudescence can occur many years
(C) Mycoplasma hominis
and subculture onto blood-containing after the initial infection.
media.
3. The most useful drug to treat (E) Epidemic typhus can be prevented by a investigators contacted the ranch, they
ehrlichiosis is vaccine. were told that there was no Q fever there
and that no one who lived at the ranch
(A) Doxycycline 5. Which of the following statements
had been sick. The most likely explanation
(B) Penicillin G about Ehrlichiae and ehrlichiosis is most
for the teenagers’ illnesses and the lack of
(C) Trimethoprim–sulfamethoxazole correct?
illness at the ranch is
(D) Gentamicin
(A) Dogs and mice are reservoirs.
(E) Nitrofurantoin (A) There was no Q fever at the ranch, and
(B) Mosquitoes are the vectors.
it was acquired elsewhere.
4. A disease characterized by malaise, (C) Ampicillin is the treatment of choice.
(B) The people at the ranch had been
headache, rigors, and fever developed in (D) Culture is a good method to confirm
previously immunized against Q fever.
members of several families living in an the diagnosis.
(C) The teenagers acquired Q fever at the
unheated war-damaged house in an (E) Ehrlichiae are typically found in
ranch, and the people who lived there
Eastern European country. Erythematous lymphocytes.
had all previously had Q fever and were
2–6 mm macular red rashes appeared on
6. A group of urban teenagers visited a now immune to it.
the peoples’ trunks and later on their
sheep ranch in a large Western state for a (D) The teenagers had other illnesses, and
extremities. Some of the people had
2-week experience. While they were the positive Q fever serology result was
coughs. One elderly person, although sick,
there, many of the pregnant ewes unrelated.
was much less sick than other adults. The
delivered lambs to the delight of the (E) The public health laboratory had errors
people huddled together to keep warm;
closely observing teenagers. About 10 in the Q fever serologic tests.
body lice were common. Which of the
days later, three of the teenagers
following statements is most correct? 7. A middle-aged sportsman, a resident of
developed flulike illnesses characterized
Oklahoma, took a hike through a rural
(A) The disease that these people had is by malaise, cough, and fever. One had an
wooded and brushy area near his home.
common in the Rocky Mountain states. infiltrate on chest radiography, indicating
The next morning, he noticed and
(B) The elderly person may have had pneumonia. The three teenagers had
removed a large (>1 cm) tick from his
acute epidemic typhus many years ago different doctors, but the physicians each
upper arm. About 1 week later, he
and recrudescent typhus now. drew a blood specimen and submitted it
experienced a gradual onset of fever and
(C) Fleas from rodents in the house were to the city health department for serologic
malaise. He now seeks medical attention
spreading Rickettsia typhi. testing. All three specimens were positive
because he is concerned about a possible
(D) The primary host of the body louse for Q fever. Public health investigators
infection transmitted by the tick. Which of
infecting the people is the rat. determined that all of the teenagers had
the following diseases is most likely to be
been to the sheep ranch. When the
acquired from a tick?
(A) Dengue was macular, but it quickly evolved into as goats and cows are infected. The
(B) Rocky Mountain spotted fever maculopapules, some with central presently recommended
petechiae. Rocky Mountain spotted fever conditions of “high-temperature, short-
(C) Typhus
caused by Rickettsia rickettsii was time” pasteurization
(D) Yellow fever
diagnosed. Which of the following are adequate to destroy viable Coxiella
(E) Malaria
statements about Rocky Mountain spotted organisms.
8. Which of the following drugs should not fever is correct?
(A) True
be used to treat Rocky Mountain spotted
(A) The vectors of Rickettsia rickettsii are (B) False
fever (Rickettsia rickettsii infection)?
ticks of the genus Ixodes.
13. The histopathological hallmark of
(A) Trimethoprim–sulfamethoxazole (B) A rash consistently appears by day 4 of
infection caused by Rickettsia rickettsiae is
(B) Chloramphenicol illness.
(C) Doxycycline (C) Rickettsia rickettsii forms inclusions in (A) Morulae within granulocytes
monocytes. (B) Morulae within monocytes
9. Which of the following should be used (D) The patient’s antibody response may (C) Granulomatous inflammation
to prevent Rocky Mountain spotted fever not occur until after the second week of (D) Intracellular vacuoles
(Rickettsia rickettsii infection)? illness. (E) Perivascular lymphocytes
(A) Attenuated Rickettsia rickettsii vaccine (E) The highest incidence of this disease is
in the Rocky Mountain states. 14. All of the following statements
(B) Prophylactic doxycycline
regarding Rickettsialpox are correct except
(C) Preventing tick bites by wearing 11. The recommended treatment for Q-
protective clothing fever endocarditis is (A) The cause of the disease is R akari.
(D) Delousing with insecticide (B) Ticks of the genus Amblyomma are
(A) Emergent surgery; antibiotics are not responsible for transmission.
10. One week after deer hunting in a effective (C) The disease is mild.
wooded area, a 33-year-old man (B) Levofloxacin monotherapy for 6 weeks (D) Disease is more common in urban than
developed fever to 39°C with headache (C) 18 months of combination therapy rural areas.
and malaise. Over the subsequent 24 with doxycycline and hydroxychloroquine
hours, he developed nausea, vomiting, (D) Penicillin and gentamicin combination 15. Reasons why C burnetii could be a
abdominal pain, and diarrhea. On day 4, therapy using IgG titers to determine potential agent of bioterrorism include
he developed a rash, initially around the duration (A) It is acquired by the inhalation.
wrists and ankles, which then
12. Coxiella burnetii can be transmitted by (B) It is highly infectious.
progressively evolved, involving the arms,
milk when animals such (C) It can be difficult to treat depending on
trunk, palms, and soles. Initially the rash
the phase of infection. 3. All of the following statements about (A) A serologic test for syphilis is not
(D) Pneumonia may be severe. perinatal Chlamydia trachomatis infections indicated because her symptoms are not
(E) All of the above. are correct except those of syphilis.
(B) A Gram stain of her endocervical
1. Which of the following statements (A) Between 15% and 40% of infants born
specimen would show Chlamydia
about chlamydial antigens is correct? to infected women develop inclusion
trachomatis inside polymorphonuclear
conjunctivitis. (B) Between 10% and 20%
(A) Chlamydiae have shared group or cells.
of infants born to infected women develop
genus-specific antigens. (C) The differential diagnosis includes
infant pneumonia.
(B) There is no cross-reaction between infection with Neisseria gonorrhoeae,
(C) The incubation period for C
Chlamydia trachomatis and Chlamydophila Chlamydia trachomatis, or both.
trachomatis inclusion conjunctivitis is 1–2
pneumoniae antigens. (D) The endocervical specimen should be
days.
(C) All five serovars of Chlamydophila analyzed for herpes simplex.
(D) The incubation period for infant
pneumoniae cross-react with Chlamydia (E) Initial treatment is with ampicillin.
pneumonia is typically 2–12 weeks.
psittaci.
(E) Ocular prophylaxis with erythromycin 5. The following statements about
(D) One serovar of Chlamydia trachomatis
or tetracycline for neonatal Neisseria trachoma are correct except
causes eye infections, and the second
gonorrhoeae infection is generally not
serovar causes genital infections. (A) It follows chronic or recurrent eye
effective against neonatal C trachomatis
infection with Chlamydia trachomatis.
2. The following are part of the control of infection.
(B) Millions of people worldwide have
Chlamydia psittaci and psittacosis in birds (F) Infant pneumonia caused by C
trachoma.
except trachomatis often presents with a staccato
(C) Trachoma is readily prevented by a
cough.
(A) Quarantine of psittacine birds chlamydial vaccine.
imported into the United States (D) Progression of trachoma can be slowed
4. An adolescent girl came to the clinic by intermittent treatment with
(B) Only allowing sale of psittacine birds because of a new and unusual vaginal azithromycin.
hatched in the United States discharge. She had recently become (E) Trachoma involves scarring of the
(C) Testing of birds for C psittaci infection sexually active and had two new partners conjunctiva, eyelid deformities, and
(D) Controlling the shipment of psittacine during the previous month. On pelvic eyelash injury to the cornea.
birds examination, a purulent discharge was
(E) Putting tetracycline in the feed of 6. Elimination of blinding trachoma
seen at the opening of her endocervical
psittacine birds involves all of the following except
canal. Which of the following statements
about this patient is most correct?
(A) Periodic administration of azithromycin pneumoniae serovar. (B) The disease is more common in
(B) Face washing and hygiene (B) Serovars L1, L2, and L3 are associated northern latitudes.
(C) Periodic culture screening of with lymphogranuloma venereum. (C) There may be marked systemic
conjunctiva swab specimens for (C) The same Chlamydia trachomatis symptoms, including fever, nausea,
Chlamydia trachomatis serovars are associated with blinding vomiting, headache, and meningismus.
(D) Environment improvements to sewage trachoma and sexually transmitted (D) Chronic inflammation with LGV can
systems to decrease the number of flies infections. lead to lymphatic obstruction.
(E) Surgery on deformed eyelids (D) The antibody titer rise seen beginning (E) Inguinal lymph nodes may become
about around 6–8 years follows infections enlarged and matted, draining pus
7. Which one of the following statements
with Chlamydia trachomatis serovars D–K. through the skin.
about Chlamydophila pneumoniae is most
(F) A few days or weeks after exposure,
correct? 9. In the United States, it has long been
the disease manifests itself as a genital
known that the positive seroprevalence
(A) Transmission from person to person is papule or vesicle.
for Chlamydia trachomatis infection
by the airborne route.
increases greatly during the primary 11. Which of the following methods are
(B) It makes glycogen-rich inclusions that
school years (ages 6–10 years). A likely considered the diagnostic tests of choice
stain with iodine.
reason for this is for urogenital infections caused by
(C) There are multiple serovars, including
(A) Frequent adenovirus infections Chlamydia trachomatis?
three that cause a systemic illness.
(B) Increased incidence of infections with
(D) They are resistant to macrolides. (A) Serology using complement fixation
C trachomatis
(E) The reservoir is house cats. (B) Cell culture using cycloheximide
(C) Cross-reactive antibodies with M
containing McCoy cells
8. The serovars of Chlamydia trachomatis protein of group A streptococci
(C) Direct fluorescent antibody testing on
generally can be divided (Streptococcus pyogenes)
urethral and cervical specimens
into groups representing their clinical (D) Children often have psittacosis (D) Nucleic acid amplification methods
infections and anatomic (E) Frequent infections with (E) Enzyme immunoassays performed on
site infected. Which of the following Chlamydophila pneumoniae genital tract specimens
statements about the Chlamydia
10. All of the following statements about 12. Nucleic acid amplification tests that
trachomatis serovars is most correct?
lymphogranuloma venereum (LGV) are are currently available in the United States
(A) There is no immunologic cross-reaction correct except for diagnosing chlamydial infections are
between Chlamydia trachomatis serovars approved for testing all of the following
(A) Chronic LGV proctitis can lead to rectal
A, B, Ba, and D and the Chlamydophila specimens except
strictures and fistula formation.
(A) Self-collected vaginal swabs in women (A) A nucleic acid amplification test that 3. Streptococcus pneumoniae resistance
(B) First void urine samples obtained from the targets the ompA gene to the drug shown in Question 1 is caused
men (B) Culture of respiratory secretions in by
(C) Rectal swabs obtained from children McCoy cells or other cell lines
(A) The action of acetyltransferase
12 years of age or younger (C) Enzyme immunoassay testing of
(B) The action of β-lactamase
(D) Urethral swab samples obtained from respiratory secretions
(C) Substitution of the d-Ala-d-Ala
adult men (D) IgG antibodies detected by
dipeptide with d-Ala-dLac dipeptide in the
(E) Cervical swab samples obtained from complement fixation
cell wall peptidoglycan
adolescent girls
1. The antimicrobial agent whose structure (D) Decreased permeability of the
13. Chlamydia pneumoniae pneumonia is shown below is considered the drug of bacterial cell wall
most resembles infection caused which of choice to treat infections caused by which (E) Genetically modified binding proteins
the following organisms? one of the following microorganisms? in the bacterial cell wall
*CHEMICAL FORMULA*
(A) Streptococcus pneumonia 4. All of the following statements about
(B) Mycoplasma pneumonia (A) Bacteroides fragilis antimicrobial resistance of enterococci are
(C) Haemophilus influenza (B) Pseudomonas aeruginosa correct except
(D) Chlamydia trachomatis (C) Herpes simplex virus (A) Enterococci are resistant to
(E) Rhinovirus (D) Streptococcus pyogenes (group A sulfamethoxazole–trimethoprim in vivo.
streptococci) (B) Cephalosporins are not active against
14. Inclusion conjunctivitis of the newborn
(E) Mycobacterium tuberculosis enterococci.
(A) Is a mucopurulent conjunctivitis that (C) Resistance to the streptogramins
2. Resistance of Staphylococcus aureus to
occurs 7–12 days after delivery (quinupristin–dalfopristin) has emerged.
the drug shown in Question 1 is caused by
(D) Vancomycin-resistant enterococci are
(B) Is caused by C psittaci rare in Europe and the United States.
(A) The action of acetyltransferase
(C) Is a result of exposure to pet birds in (E) Vancomycin-resistant enterococci once
(B) The action of β-lactamase
the home consistently clonal are now
(C) Substitution of the d-Ala-d-Ala
(D) Is treated with systemic penicillin heterogeneous.
dipeptide with the d-Alad-Lac dipeptide in
because it may progress to pneumonia
the cell wall peptidoglycan
(E) None of the above 5. A 20-year-old Asian woman, a recent
(D) Decreased permeability of the
immigrant to the United States, develops
15. The diagnostic method of choice for C bacterial cell wall to the drug
fever and a cough productive of blood-
trachomatis pneumonia in the newborn is (E) Staphylococcus aureus being an
streaked sputum. She has lost 6 kg of body
intracellular pathogen
weight in the past 6 weeks. Her chest (A) Fluoroquinolones hip joint, artificial heart valve, urinary
radiograph shows bilateral upper lobe (B) Aminoglycosides catheter)
infiltrates with cavities. Given the history (C) Penicillins (E) Waiting for culture and susceptibility
and chest radiography findings, which of (D) Glycopeptides (eg, vancomycin) test results
the following drug regimens would be the (E) Polymyxins
10. All of the following agents have good
best appropriate initial therapy while
8. There are many bacterial–antimicrobial activity against grampositive organisms
awaiting culture results?
resistance combinations. Which one of the except
(A) Isoniazid, rifampin, pyrazinamide, and following is of major international
(A) Daptomycin
ethambutol concern?
(B) Vancomycin
(B) Penicillin G and rifampin
(A) Sulfonamide resistance in Neisseria (C) Aztreonam
(C) Cefotaxime, clindamycin, and
meningitides (D) Quinupristin–dalfopristin
trimethoprim–sulfamethoxazole
(B) Penicillin G resistance in Neisseria (E) Tigecycline
(D) Ampicillin–sulbactam gonorrhoeae
11. Tigecycline, a new glycylcycline
(E) Vancomycin, gentamicin, and (C) Ampicillin resistance in Haemophilus
antibiotic with good activity against a
clindamycin influenza
variety of pathogens, is best used for
(D) Erythromycin resistance in
6. Aminoglycoside antibiotics typically treatment of which of the following
Streptococcus pyogenes (group A
cause which of the following adverse infections?
streptococci)
events?
(E) Vancomycin resistance in (A) Meningitis
(A) They cause aplastic anemia. Staphylococcus aureus (B) Intra-abdominal infections caused by
(B) They cause nonspecific stimulation of B mixed aerobic and anaerobic bacteria
9. Which of the following factors is not
cells. (C) Neonatal sepsis
generally considered when selecting initial
(C) They cause ototoxicity and (D) Urethritis caused by Chlamydia
antimicrobial therapy for an infection?
nephrotoxicity. trachomatis
(D) They cause photosensitivity. (A) Age of the patient (E) As monotherapy for bacteremia caused
(B) Anatomic site of the infection (eg, by Acinetobacter baumannii
7. Which one of the following groups of
meningitis or urinary tract infection)
antimicrobial agents acts on 12. Which of the following carbapenem
(C) Whether or not the patient is
microorganisms by inhibiting protein antibiotics has no activity against
immunocompromised
synthesis? Pseudomonas aeruginosa?
(D) Whether or not the patient has
implanted devices in place (eg, artificial
(A) Imipenem 1. Some viruses are characterized by ismost accurate about cellular receptors
(B) Meropenem helical symmetry of the viral nucleocapsid. for viruses?
(C) Doripenem Which of the following statements about
(A) Cellular receptors for viruses have no
(D) Ertapenem viruses with helical symmetry is most
known cellular function.
accurate?
13. Which of the following agents would (B) All viruses within a given family use the
not be expected to demonstrate (A) All enveloped viruses with helical same cellular receptor.
postantibiotic affect against gram-negative symmetry are classified into the same (C) All cells in a susceptible host express
bacilli? virus family. the viral receptor.
(B) Helical nucleocapsids are found (D) Successful infection of a cell by a virus
(A) Imipenem
primarily in DNAcontaining viruses. may involve interaction with more than
(B) Ciprofloxacin
(C) All human viruses with helical one type of receptor.
(C) Gentamicin
nucleocapsids possess an envelope.
(D) Ampicillin 4. Which of the following can be used to
(D) Excess empty helical particles
quantitate the infectious titer of viruses?
14. All of the following are common containing no nucleic acid are commonly
mechanisms of resistance to the produced in infected cells. (A) Plaque assay
penicillins except (B) Electron microscopy
2. Virus-infected cells often develop
(C) Hemagglutination
(A) Production of β-lactamases morphologic changes referred to as
(D) Polymerase chain reaction
(B) Alterations in target receptors (PBPs) cytopathic effects. Which of the following
(E) Enzyme immunoassay
(C) Inability to activate autolytic enzymes statements about virus-induced cytopathic
(D) Failure to synthesize peptidoglycans changes is most accurate? 5. Which one of the following states a
(E) Methylation of ribosomal RNA principle regarding viral nucleic acid?
(A) They are pathognomonic for an
15. The drug of first choice for the infecting virus. (A) Viruses contain both RNA and DNA.
treatment of serious anaerobic infections (B) They are rarely associated with cell (B) Some viruses contain a segmented
caused by Bacteroides fragilis is death. genome.
(C) They may include giant cell formation. (C) Purified viral nucleic acid from any
(A) Clindamycin (D) They can only be seen with an electron virus is usually infectious.
(B) Ampicillin microscope. (D) Viral genome sizes are similar among
(C) Cefoxitin
known human viruses.
(D) Metronidazole 3. Viruses usually initiate infection by first
(E) Amoxicillin–clavulanate interacting with receptors on the surface 6. Two mutants of poliovirus have been
of cells. Which of the following statements isolated, one (MutX) with a mutation in
gene X and the second (MutY) with a (A) Poxviruses (E) Helical nucleocapsids are found with
mutation in gene Y. If cells are infected (B) Filoviruses single-stranded DNA viruses.
with each mutant alone, no virus is (C) Herpesviruses
11. Many viruses can be grown in the
produced. If a cell is coinfected with both (D) Influenza viruses
laboratory. Which of the following
MutX and MutY, which one of the (E) Caliciviruses
statements about virus propagation is not
following is most likely to occur?
9. Some viruses encode for a viral RNA- true?
(A) Reassortment of genome segments dependent RNA polymerase. Which of the
(A) Some viruses can be propagated in
may occur and give rise to a viable wild- following states a principle about viral
cell-free media.
type virus. RNA polymerases?
(B) Some mammalian viruses can be
(B) The genomes may be reverse
(A) All RNA viruses carry RNA polymerase cultivated in hen’s eggs.
transcribed to DNA and both MutX and
molecules inside virus particles because (C) Some viruses with broad host ranges
MutY viruses produced.
they are needed to initiate the next can multiply in many types of cells.
(C) Complementation between the
infectious cycle. (D) Some human viruses can be grown in
mutant gene products may occur and
(B) Antibodies against the viral RNA mice.
both MutX and MutY viruses produced.
polymerase neutralize virus infectivity. (E) Most virus preparations have particle-
(D) The cells will transform at high
(C) Negative-strand RNA viruses supply to-infectious unit ratios greater than 1.
frequency because they will not be killed
their own RNAdependent RNA
by the poliovirus mutants. 12. Laboratory infections can be acquired
polymerase because eukaryotic cells lack
when working with viruses unless good
7. Which one of the following viruses such enzymes.
laboratory safety practices are followed.
possesses an RNA genome that is (D) The viral RNA polymerase protein also
Which of the following is not a good
infectious when purified? serves as a major core structural protein in
biosafety practice?
the virus particle.
(A) Influenza virus
(A) Use of biosafety hoods
(B) Poliovirus 10. Which of the following statements
(B) Use of laboratory coats and gloves
(C) Papillomavirus regarding virus morphology is true?
(C) Avoidance of pipetting by mouth
(D) Measles virus
(A) All RNA viruses are spherical in shape. (D) Flushing experimental waste down
(E) Rotavirus
(B) Some viruses contain flagella. laboratory sink
8. Viruses belonging to which of the (C) Some viruses with DNA genomes (E) Not eating or drinking in the laboratory
following groups are likely to establish contain a primitive nucleus.
13. Small viruses are in the same size
latent infections? (D) Viral surface proteins protect the viral
range as which of the following?
genome from nucleases.
(A) Staphylococcus species (B) It coats viral particles and blocks their that virus.
(B) Serum globulin attachment to cells. (D) A particular disease syndrome has a
(C) It induces synthesis of one or more single viral cause.
(C) Red blood cells
cellular proteins that inhibit translation
(D) Eukaryotic ribosomes 4. The skin is an impenetrable barrier to
or transcription.
(E) Mitochondria virus entry, but a few viruses are able to
(D) It protects the virus-infected cell that
breach this barrier and initiate infection of
14. Which of the following is not an produced it from cell death.
the host. Which of the following is an
important factor contributing to the
2. A 9-month-old girl is taken to the example of a virus that enters through skin
phenomenon of emerging viral diseases?
emergency room because of abrasions?
(A) International air travel
fever and persistent cough. Rales are (A) Adenovirus
(B) Antibiotic resistance
heard in her left chest on physical (B) Rotavirus
(C) Deforestation
examination. An infiltrate in her left lung is (C) Rhinovirus
(D) War
seen on the chest radiograph. Pneumonia (D) Papillomavirus
(E) Organ and tissue transplantation
is diagnosed. Which of the following is the (E) Influenza virus
15. Arboviruses are classified into several most likely cause?
5. A 40-year-old man has HIV/AIDS
different virus families but are grouped
(A) Rotavirus characterized by a low CD4 count and a
together based on which of the following
(B) Rhinovirus high viral load. Highly active antiretroviral
common characteristics?
(C) Adenovirus therapy (HAART) will be initiated. One of
(A) Replicate only in humans (D) Respiratory syncytial virus the drugs under consideration is a
(B) Contain both RNA and DNA (E) Coxsackievirus nucleoside analog that inhibits viral
(C) Are transmitted by vectors reverse transcriptase and is active against
3. Which one of the following is a
(D) Cause hemorrhagic fevers both HIV and hepatitis B virus. That drug is
fundamental principle of viral disease
(E) Cause encephalitis
causation? (A) Acyclovir
1. Interferons are an important part of the (B) Amantadine
(A) One virus type induces a single disease
host defense against viral infections. What (C) Ribavirin
syndrome.
is interferon’s principal mode of action? (D) Saquinavir
(B) Many viral infections are subclinical
(E) Lamivudine
(A) It is present in the serum of healthy and do not produce clinical disease.
(F) Fuzeon
individuals, providing a viral surveillance (C) The type of disease produced by a virus
role. can be predicted by the morphology of
6. Regarding the HIV/AIDS patient in 8. Which one of the following statements particle
Question 5, a peptidomimetic agent that describes an advantage of killed-virus (B) Appear in the host sooner after viral
blocks virus-mediated cleavage of viral vaccines over attenuated live-virus infection than interferon
structural protein precursors is chosen as vaccines? (C) Directed against viral nucleic acid
a second drug. That drug is sequences
(A) Killed-virus vaccines induce a broader
(D) Induced only by disease-causing
(A) Acyclovir range of immune responses than do
viruses
(B) Amantadine attenuated live-virus vaccines.
(E) Of little importance to immunity to
(C) Ribavirin (B) Killed-virus vaccines more closely
viral infection
(D) Saquinavir mimic natural infections than do
(E) Lamivudine attenuated live-virus vaccines. 11. Many viruses use the respiratory tract
(F) Fuzeon (C) Killed-virus vaccines pose no risk that as the route of entry
vaccine virus might be transmitted to
7. A 63-year-old woman is hospitalized for to initiate infections. Which of the
susceptible contacts.
treatment of leukemia. One day after following virus groups does not?
(D) Killed-virus vaccines are efficacious
admission she develops chills, fever,
against respiratory virus infections (A) Adenovirus
cough, headache, and myalgia. She states
because they induce good mucosal (B) Coronavirus
that her husband had a similar illness a
immunity. (C) Hepadnavirus
few days earlier. There is major concern
(D) Paramyxovirus
about a respiratory virus outbreak in the 9. What type of hepatitis B vaccine is
(E) Poxvirus
staff of the chemotherapy ward and in the currently in use in the United States?
patients on that ward. A synthetic amine 12. Which of the following licensed virus
(A) Synthetic peptide vaccine
that inhibits influenza A virus by blocking vaccines is a subunit vaccine prepared
(B) Killed-virus vaccine
viral uncoating is chosen for prophylactic using recombinant DNA technology?
(C) Attenuated live-virus vaccine
treatment of the staff and patients. That
(D) Subunit vaccine produced using (A) Measles–mumps–rubella
drug is
recombinant DNA (B) Varicella
(A) Acyclovir (C) Hepatitis A
10. Which one of the following phrases
(B) Amantadine (D) Papilloma
accurately describes viral neutralizing
(C) Ribavirin (E) Rotavirus
antibodies?
(D) Saquinavir (F) Rabies
(E) Lamivudine (A) Directed against viral protein
(F) Fuzeon determinants on the outside of the virus
13. Which of the following viruses is the concern. 3. The 8-year-old child in Question 2 had
most common cause of (C) With inactivated vaccines, IgA an illness with more than one phase.
neonatal infections in the United States? mucosal immunity is usually induced. Which symptoms coincide with the second
(D) With inactivated vaccines, protective phase of the illness?
(A) Rubella
immunity is mainly caused by the
(B) Parvovirus B19 (A) Sore throat
production of IgG.
(C) Hepatitis B
(B) Skin rash
(D) Cytomegalovirus 1. Which one of the following best
(C) Headache
(E) Varicella describes a physicochemical property of
(D) Diarrhea
(F) Human immunodeficiency virus parvoviruses?
(E) Cough
14. Which one of the following statements (A) Enveloped virus particle
4. A 42-year-old man with HIV/AIDS
concerning interferons is least accurate? (B) Single-stranded DNA genome
presented with aplastic anemia. Using the
(C) Infectivity is inactivated by ether
(A) Interferons are proteins that influence polymerase chain reaction, parvovirus B19
treatment
host defenses in many ways, one of which was detected in his serum. The patient
(D) Virion exhibits helical symmetry
is the induction of an antiviral state. presumably acquired his parvovirus B19
(E) Virion is about the same size as
(B) Interferons are synthesized only by infection from another person. The most
herpesviruses
virus-infected cells. likely route of transmission is
(C) Interferons inhibit a broad range of 2. An 8-year-old child recently had
(A) By contact with respiratory secretions
viruses, not just the virus that induced the erythema infectiosum. Her 33-year-old
or droplets
interferon. mother subsequently developed arthralgia
(B) By contact with a skin rash
(D) Interferons induce the synthesis of a followed by painful arthritis with swelling
(C) Through sexual activity
ribonuclease that degrades viral mRNA. in the small joints of both hands. In
(D) Through a recent blood transfusion
addition to the apparent tropism for
15. Each of the following statements
joints, human parvovirus B19 is highly 5. Which one of the following is a disease
concerning viral vaccines is correct except
tropic for which cell type? in which the role of parvovirus B19 has not
(A) In live attenuated vaccines, the virus been established?
(A) CD4 T lymphocytes
has lost its ability to cause disease but has
(B) Renal tubule cells (A) Erythema infectiosum (fifth disease)
retained its ability to induce neutralizing
(C) Erythroid cells (B) Transient aplastic crisis
antibody.
(D) Glial cells (C) Hydrops fetalis
(B) In live attenuated vaccines, the
(E) Peyer patches (D) Fulminant hepatitis
possibility of reversion to virulence is of
6. Which one of the following best (C) Respiratory secretions (C) Penton
describes the replication of human (D) Fetal liver (D) Terminal protein
parvovirus B19? (E) Bone marrow (E) E1 region protein
(F) Cysteine proteinase
(A) Stimulates resting cells to proliferate 9. Which of the following is available as a
(G) E3 region protein
treatment or preventative for parvovirus
(B) Uses blood group antigen P as cellular
B19 infections? 2. What adenovirus protein serves as
receptor
primer for initiation of viral DNA
(C) Readily establishes persistent (A) Commercial immunoglobulin
synthesis?
infections (B) Vaccine containing recombinant VP2
(D) Entire replication cycle occurs in viral antigen (A) Fiber
cytoplasm (C) Bone marrow transplantation (B) Hexon
(E) Production of infectious progeny (D) Antiviral drug that blocks virus– (C) Penton
requires the presence of a helper virus receptor interaction (D) Terminal protein
(E) E1 region protein
7. Which one of the following statements 10. Human erythroviruses and bocaviruses
(F) Cysteine proteinase
is most accurate concerning human share the following properties except for
(G) E3 region protein
infections by parvovirus B19? which one?
3. What adenovirus protein comprises the
(A) Parvovirus B19 is transmitted readily (A) Small, nonenveloped virus particles
majority of capsomeres making up the
by sexual intercourse. (B) Difficult to culture
virus capsid?
(B) Patients with disseminated disease (C) Cause anemia
caused by parvovirus B19 should be (D) Global distribution (A) Fiber
treated with acyclovir. (E) No vaccine exists (B) Hexon
(C) Parvovirus B19 does not cause any (C) Penton
In what follows, singular may be construed
human disease. (D) Terminal protein
as plural (or vice versa) as the sense
(D) There is no vaccine for human (E) E1 region protein
dictates.
parvovirus. (F) Cysteine proteinase
1. What adenovirus protein or proteins (G) E3 region protein
8. Human bocavirus is a newly discovered
regulate early transcription of the viral
parvovirus. It has been detected most 4. A 3-month-old infant had watery
genes and modulate the cell cycle?
frequently in which type of sample? diarrhea and fever for 10 days. Rotavirus
(A) Fiber or adenovirus types 40 and 41 are the
(A) Urine
(B) Hexon suspected agents. What type of specimen
(B) Cord blood
would be most appropriate for detection 7. Which adenovirus types are frequent DNA. Of the following, which one is the
of adenovirus type 40 and 41 infection in causes of acute respiratory disease among most appropriate conclusion you could
this patient? military recruits? draw?

(A) Blood (A) Types 40 and 41 (A) There is a high likelihood that the
(B) UrinE (B) Types 8, 19, and 37 agent is a rotavirus.
(C) Conjunctival swab (C) Types 1, 2, 5, and 6 (B) You need to determine the viral
(D) Stool (D) Types 3, 4, and 7 serotype to establish whether the virus
(E) Throat swab (E) Types 21, 22, 34, and 35 was important in causing the disease.
(F) Cerebrospinal fluid (C) The patient should have been treated
8. Which of the following events led to
with the antiviral drug amantadine to
5. Which of the following human diseases reappearance of acute respiratory disease
shorten the duration of symptoms.
has not been associated with outbreaks among United States military
(D) The virus particle would contain a
adenoviruses? recruits in the late 1990s?
reverse transcriptase enzyme.
(A) Cancer (A) Emergence of a new virulent strain of
10. Which of the following groups of
(B) Common colds adenovirus
individuals is at the lowest risk of
(C) Acute respiratory diseases (B) Cessation of adenovirus vaccination
adenovirus disease?
(D) Keratoconjunctivitis program for recruits
(E) Gastroenteritis (C) Change in military housing and training (A) Healthy adults
(F) Hemorrhagic cystitis conditions for recruits (B) Young children
(D) Cessation of adenovirus antiviral drug (C) Bone marrow transplant recipients
6. A 2½-year-old child attending nursery
therapy program for recruits (D) Military recruits
school acquires a mild respiratory
(E) AIDS patients
infection. Other children in the nursery 9. Your summer research project is to
school have similar illnesses. Which study the viruses that cause 11. Adenoviruses can cause eye infections
adenovirus types are the most likely gastroenteritis. You recover a virus from a that are highly contagious. Which of the
causes of the illnesses? stool sample and notice that the growth following is least likely to be a means of
medium on the infected cultures is highly transmission during an outbreak of
(A) Types 40 and 41
acidic. You find that the viral genome is epidemic keratoconjunctivitis?
(B) Types 8, 19, and 37
double-stranded
(C) Types 1, 2, 5, and 6 (A) Swimming pools
(D) Types 3, 4, and 7 (B) Hand towels
(E) Types 21, 22, 34, and 35 (C) Mosquito bites
(D) Hand-to-eye 4. A 19-year-old female college student
(E) Contaminated ophthalmic equipment 1. A previously healthy 3-year-old boy has a fever, sore throat, and
develops a classic viral childhood illness. lymphadenopathy accompanied by
12. There are 51 known types of human
Which of the following primary viral lymphocytosis with atypical cells and an
adenoviruses. Which of the following
infections of childhood is usually increase in sheep cell agglutinins. The
statements is most accurate?
symptomatic? diagnosis is most likely
(A) Types cannot be distinguished
serologically. (A) Cytomegalovirus (A) Infectious hepatitis
(B) All cause respiratory infections in (B) Epstein-Barr virus (B) Infectious mononucleosis
children. (C) Hepatitis B virus (C) Chickenpox
(C) Most types replicate well in T (D) Varicella-zoster virus (D) Herpes simplex infection
lymphocytes. (E) Parvovirus B19 (E) Viral meningitis
(D) Two types can cause gastroenteritis.
2. Which one of the following is a 5. A Tzanck smear of a scraping obtained
13. Each of the following statements recommended therapy for herpes simplex from a vesicle on the skin demonstrates
concerning adenoviruses is correct except virus genital infection? multinucleated giant cells. Multinucleated
giant cells are associated with which of the
(A) Adenoviruses are composed of a (A) Acyclovir
following viruses?
double-stranded DNA genome and a (B) Attenuated live virus vaccine
capsid without an envelope. (C) Herpes immune globulin (A) Varicella-zoster
(B) Adenoviruses cause both sore throat (D) Interferon-α (B) Variola major
and pneumonia. (E) Ribavirin (C) Coxsackievirus
(C) Adenoviruses have only one serologic (D) Molluscum contagiosum
3. Most herpesvirus infections are
type.
endemic worldwide. Which one of the 6. Which of the following statements
(D) Adenoviruses are implicated as a cause
following viruses shows marked about betaherpesviruses is not true?
of tumors in animals but not humans.
geographic differences in seroprevalence?
(A) They establish latent infections and
14. Which of the following conditions is
(A) Cytomegalovirus persist indefinitely in infected hosts.
least likely to be caused by adenoviruses?
(B) Epstein-Barr virus
(B) They are reactivated in
(A) Conjunctivitis (C) Herpes simplex virus type 2
immunocompromised patients.
(B) Pneumonia (D) Kaposi sarcoma herpesvirus
(C) Most infections are subclinical.
(C) Pharyngitis (E) Varicella-zoster virus
(D) They can infect lymphoid cells.
(D) Glomerulonephritis
(E) They have short, cytolytic growth resonance imaging shows a left temporal virus.
cycles in cultured cells. lobe lesion. Which of the following tests (D) Lesions typically last 1 month or
would be most appropriate to confirm a longer.
7. A 28-year-old woman has recurrent
diagnosis of herpes simplex encephalitis in (E) Students should be vaccinated before
genital herpes. Which of
this patient? participating in wrestling tournaments.
the following statements about genital
herpes infections is true? (A) Brain biopsy 12. The shingles vaccine is recommended
(B) Tzanck smear for which of the following groups?
(C) Polymerase chain reaction assay for
(A) Reactivation of latent virus during (A) Healthy adolescents
viral DNA in cerebrospinal fluid
pregnancy poses no threat to the (B) Individuals older than age 60 years
(D) Serologic test for viral IgM antibody
newborn. (C) Pregnant women
(B) Virus cannot be transmitted in the 10. Which of the following tumors is (D) Those who never had chickenpox
absence of apparent lesions. caused by a virus other than Epstein-Barr
13. The most common congenital infection
(C) Recurrent episodes caused by virus?
is caused by
reactivation of latent virus tend to be
(A) Posttransplant lymphomas
more severe than the primary infection. (A) Varicella-zoster virus
(B) Hodgkin disease
(D) They can be caused by either herpes (B) Herpes simplex virus type 2
(C) Kaposi sarcoma
simplex virus type 1 or type 2. (C) Human herpesvirus 8 (Kaposi sarcoma
(D) AIDS-related central nervous system
(E) Latent herpes simplex virus can be herpesvirus)
non-Hodgkin lymphomas
found in dendritic cells. (D) Cytomegalovirus
(E) Burkitt lymphoma
(E) Parvovirus
8. Which of the following viruses causes a
11. An outbreak of a rash called “mat
mononucleosis-like syndrome and is 14. Which of the following groups are at
herpes” occurred among high school
excreted in the urine? increased risk for herpes zoster?
students who had competed in a wrestling
(A) Cytomegalovirus tournament. Which of the following (A) Persons at advanced age
(B) Epstein-Barr virus statements is most accurate? (B) Patients with atopic dermatitis
(C) Human herpesvirus 6 (C) Pregnant women
(A) The rash is not contagious among
(D) Varicella-zoster virus wrestlers. (D) Persons who have been vaccinated
(E) Herpes simplex virus type 2 (B) The causative agent is herpes simplex with varicella vaccine
virus type 1. (E) Infants with congenital infections
9. A 53-year-old woman develops fever
(C) The causative agent is varicella-zoster
and focal neurologic signs. Magnetic
15. Which one of the following is the best infected cells by cocultivation with (A) HSV-1 causes lesions above the
explanation for the selective action of susceptible cells. umbilicus more frequently than HSV-2
acyclovir (acycloguanosine) in herpes does.
17. Vaccines have been demonstrated to
simplex virus (HSV)-infected cells? (B) Infection by HSV-1 is not associated
be efficacious in preventing herpesvirus
with any tumors in humans.
(A) Acyclovir binds specifically to viral disease in which one of the following
(C) Antiserum to HSV-1 neutralizes HSV-1
receptors only on the surface of the HSV- situations?
much more effectively than HSV-2.
infected cell.
(A) Herpes simplex virus type 1 primary (D) Whereas HSV-1 causes frequent
(B) Acyclovir is phosphorylated by a virus-
infection recurrences, HSV-2 infection rarely
encoded phosphokinase only within HSV-
(B) Herpes simplex virus type 2 recurs.
infected cells.
reactivation
(C) Acyclovir selectively inhibits the RNA 20. Each of the following statements
(C) Varicella-zoster reactivation
polymerase in the HSV virion. concerning Epstein-Barr virus is correct
(D) Cytomegalovirus primary infection
(D) Acyclovir specifically blocks the matrix except
(E) Epstein-Barr virus reactivation
protein of HSV, thereby preventing release
(A) Many infections are mild or
of progeny HSV. 18. Herpes simplex virus and
inapparent.
cytomegalovirus share many features.
16. Each of the following statements (B) The earlier in life primary infection is
Which one of the following features is
concerning herpesvirus latency is correct acquired, the more likely the typical
least likely to be shared?
except picture of infectious mononucleosis will
(A) Important cause of morbidity and be manifest.
(A) Exogenous stimuli can cause
mortality in the newborn (C) Latently infected lymphocytes regularly
reactivation of latent infection, with
(B) Congenital abnormalities caused by persist after an acute episode of infection.
induction of symptomatic disease.
transplacental passage (D) Infection confers immunity against
(B) During latency, antiviral antibody is
(C) Important cause of serious disease in second episodes of infectious
not demonstrable in the sera of infected
immunosuppressed individuals mononucleosis.
individuals.
(D) Mild or inapparent infection
(C) Reactivation of latent herpesviruses is
more common in 19. Herpes simplex virus type 1 (HSV-1) is
distinct from herpes simplex virus type 2
patients with impaired cell-mediated 1. A 40-year-old emergency services
(HSV-2) in several different ways. Which
immunity than in immunocompetent worker is considering smallpox vaccination
one of the following is the least accurate
patients. because of the potential for bioterrorism.
statement?
(D) Virus can be recovered from latently He inquires about the vaccine and finds
that the most common adverse reaction (D) Cowpox (A) Viral replication cycle takes place in the
or complication after vaccinia (smallpox) (E) Yaba tumor virus cytoplasm of infected cells.
vaccination is (B) The uncoating step leading to release
4. A 7-year-old boy has pox-like lesions on
of the viral genome requires a newly
(A) Generalized vaccinia his left hand and arm. He has a pet rodent
synthesized viral protein.
(B) Eczema vaccinatum imported from West Africa. Monkeypox is
(C) Early transcription of more than 50
(C) Progressive vaccinia diagnosed in the boy and the rodent.
viral genes occurs within viral cores and
(D) Severe allergic reaction Which of the following statements about
precedes viral DNA replication.
(E) Inadvertent autoinoculation monkeypox virus is most correct?
(D) Newly formed virus particles mature
2. The emergency services worker in (A) Clinical disease resembles smallpox. by budding through the nuclear
Question 1 also learns about (B) Human infections are never fatal. membrane.
(C) Smallpox vaccination is not protective.
contraindications to smallpox vaccination. 7. A 37-year-old emergency room nurse is
(D) Infections are readily transmitted
Which one of the following conditions is vaccinated for smallpox because of the
among family members.
not a contraindication for the use of bioterrorism threat. Twelve days later, she
(E) Virus particles can be distinguished
vaccinia (smallpox) vaccine under routine has a major complication from the
from smallpox virus by electron
nonemergency conditions? vaccine. Vaccinia immune globulin therapy
microscopy.
is considered. For which of the following
(A) Immunosuppression conditions is treatment with vaccinia
5. Which of the following best describes
(B) Severe allergy to a component of the immune globulin of no benefit?
the currently licensed vaccinia (smallpox)
vaccine
vaccines?
(C) Household contact with a person with (A) Severe generalized vaccinia
eczema (A) Live attenuated smallpox virus (B) Progressive vaccinia
(D) Pregnancy (B) Inactivated smallpox virus (C) Postvaccinial encephalitis
(E) Previous smallpox vaccination (C) Live vaccinia virus (D) Eczema vaccinatum
(D) Inactivated vaccinia virus (E) Ocular vaccinia
3. Which of the following poxviruses
(E) Reassortant vaccine containing both
infects only humans? 8. Another nurse from the same
vaccinia and smallpox viruses
emergency room as the patient in
(A) Monkeypox
6. Which of the following does not apply
Question 7 also is vaccinated for smallpox.
(B) Molluscum contagiosum to vaccinia virus replication in cultured
At what point is she considered to be fully
(C) Tanapox cells?
protected from smallpox?
(A) Ten days after the first dose of vaccine (D) A colleague with a variola virus clone. 13. The eradication of smallpox was
regardless of the response at the site of (E) Distribution of a full-length viral facilitated by several features of the virus.
administration genome is prohibited. Which one of the following contributed
(B) Ten days after the second dose of least to eradication?
11. Laboratory scientists who work with
vaccine regardless of the response at the
vaccinia virus-infected cultures or animals (A) It has one antigenic type.
site of administration
are at risk of accidental exposure to the (B) Inapparent infection is rare.
(C) After the appearance of any reaction at
virus. Which of the following procedures (C) Administration of live vaccine reliably
the site of administration
by the laboratory worker is of least benefit induces immunity.
(D) After the appearance of a vesicular or
in protecting against inadvertent infection (D) It multiplies in the cytoplasm of
pustular lesion at the site of
with vaccinia virus? infected cells.
administration
(E) After the appearance of a generalized (A) Proper use of personal protective 14. Vaccination with the vaccinia
rash in the vaccinated person equipment such as gloves and goggles (smallpox) vaccine protects against
(B) Cleaning of laboratory work space infections by the following poxviruses
9. Which of the following does not fulfill
before experimentation except
the criteria for exposure to vaccinia?
(C) Smallpox vaccination
(A) Molluscum contagiosum
(A) Smallpox vaccination (D) Safe needle-handling practices
(B) Variola
(B) Close contact with a recent smallpox (E) Use of biosafety hoods
(C) Cowpox
vaccine
12. Vaccinia virus has all of the following (D) Monkeypox
(C) Intrauterine exposure
attributes except
(D) Injection of vaccinia immune globulin 1. A 24-year-old woman in New York City
(A) Can cause severe localized or is admitted to the hospital because of
10. A researcher wishes to obtain a full-
disseminated disease. jaundice. On workup, she is found to have
length genome of variola virus for vaccine
(B) Is a live, attenuated smallpox virus. HCV infection. The major risk factor for
studies. Which of the following is the
(C) Can induce immunity that lasts only a HCV infection in the United States is
appropriate source of the viral DNA?
few years.
(A) Tattoos
(A) The Centers for Disease Control and (D) Has been in use for more than 200
(B) Injecting drug use
Prevention. years.
(C) Blood transfusion
(B) A World Health Organization (E) Gene sequences coding for other viral (D) Sexual activity
collaborating center. proteins can be inserted into its genome. (E) Working in health care occupations
(C) The American Type Culture Collection.
2. Which of the following exposures poses infection with HBV. The following is most (70–90%) HCVinfected persons.
a risk for hepatitis infection? correct (C) HCV-associated liver disease is the
major cause for liver transplantation.
(A) A nurse sustains a needlestick while (A) HDV is a defective mutant of HBV.
(D) HCV viremia occurs transiently during
drawing up insulin to administer to an (B) HDV depends on HBV surface antigen
early stages of infection.
HBV-infected patient with diabetes. for virion formation.
(E) HCV-infected patients are at high risk
(B) While cleaning the bathroom, a (C) HDV induces an immune response
(5–20%) for liver cancer.
housekeeper’s intact skin has contact with indistinguishable from that induced by
feces. HBV. 7. A middle-aged man complained of acute
(C) An operating room technician with (D) HDV is related to HCV. onset of fever, nausea, and pain in the
chapped and abraded hands notices (E) HDV contains a circular DNA genome. right upper abdominal quadrant. There
blood under his gloves after assisting in was jaundice, and dark urine had been
5. A 23-year-old woman is planning a 1-
an operation on a patient with HCV observed several days earlier. A laboratory
year trip through Europe, Egypt, and the
infection. test was positive for HAV IgM antibody.
Indian subcontinent and receives a vaccine
(D) A child drinks out of the same cup as The physician can tell the patient that
for hepatitis A. The current hepatitis A
her mother, who has an HAV infection.
vaccine is (A) He probably acquired the infection
(E) A shopper eats a sandwich prepared by
from a recent blood transfusion.
a worker with an asymptomatic HBV (A) A live attenuated virus vaccine
(B) He will probably develop chronic
infection. (B) A recombinant DNA vaccine
hepatitis.
(C) A formalin-inactivated virus vaccine
3. An epidemic of jaundice caused by HEV (C) He will be at high risk of developing
(D) An envelope glycoprotein subunit
occurred in New Delhi. HEV is hepatocellular carcinoma.
vaccine
(D) He will be resistant to infection with
(A) Found in rodents and pigs (E) A chimeric poliovirus that expresses
hepatitis E.
(B) A major cause of bloodborne hepatitis HAV neutralizing epitopes
(E) He may transmit the infection to
(C) The cause of a disease that resembles
6. The following statements about HCV family members by person-to-person
hepatitis C
infection and associated chronic liver spread for up to 2 weeks.
(D) Capable of establishing chronic
disease in the United States are correct
infections 8. Several different viruses can cause
except
(E) Associated with an increased risk of hepatitis. One of the following statements
liver cancer (A) HCV is responsible for 40% of chronic applies to all four viruses: HAV, HCV, HDV,
liver disease. and HEV.
4. HDV (delta agent) is found only in
(B) Chronic infection develops in most
patients who have either acute or chronic
(A) Contains a single-stranded RNA (C) Has been infected with HAV and HCV in for infection
genome the past and has hepatitis B now (E) Persons who have a clotting factor
(B) Is transmitted primarily by the (D) Has been infected with HAV in the disorder
parenteral route past, has not been infected with HBV, and (F) Susceptible persons who have chronic
(C) Is transmitted primarily by the fecal– has hepatitis C now liver disease
oral route (E) Has been infected with HAV and HCV in (G) Teachers in elementary schools
(D) Is associated with fulminant hepatitis the past, has not been infected with HBV,
12. There is global variation in the
(E) Undergoes sequence variation during and has hepatitis E now
prevalence of HBV infection. Which of the
chronic infection
10. A 36-year-old nurse is found to be following geographic areas has low
9. A 30-year-old student goes to the both HBsAg positive and HBeAg positive. endemicity (HBsAg prevalence of <2%)?
emergency room because of fever and The nurse most likely
(A) Southeast Asia
anorexia for the past 3 days. She appears
(A) Has acute hepatitis and is infectious (B) The Pacific Islands
jaundiced. Her liver is enlarged and
(B) Has both HBV and HEV infections (C) Eastern Europe
tender. A laboratory test shows elevated
(C) Has a chronic HBV infection (D) Australia
aminotransferases. She reports a history
(D) Has cleared a past HBV infection (E) Sub-Saharan Africa
of having received hepatitis B vaccine 2
(E) Was previously immunized with HBV
years ago but has not had hepatitis A 13. Which of the following persons are not
vaccine prepared from healthy HBsAg-
vaccine. The results of her hepatitis recommended to receive hepatitis B
positive carriers
serologic tests are as follows: HAV IgM- vaccine because they have a risk factor for
negative, HAV IgG-positive, HBsAg- 11. The following persons are at increased HBV infection?
negative, risk for HAV infection
(A) Sexually active persons who are not in
HBsAb-positive, HBcAb-negative, HCV Ab- and should be routinely vaccinated except long-term, mutually monogamous
positive. The most accurate conclusion is for which group? relationships
that she probably (B) Injection drug users
(A) Persons traveling to or working in
(C) Pregnant women
(A) Has hepatitis A now, has not been countries that have high levels of HAV
(D) Persons who live in a household with a
infected with HBV, and had hepatitis C in infection
person who is HBsAg positive
the past (B) Men who have sex with men
(E) Persons seeking treatment for a
(B) Has hepatitis A now and has been (C) Users of illegal drugs (both injecting
sexually transmitted disease
infected with both HBV and HCV in the and noninjecting)
past (D) Persons who have an occupational risk
14. Which of the following statements (A) Infection does not elicit the production become chronic carriers of HCV and are
regarding HBIG is not true? of antibody. predisposed to hepatocellular carcinoma.
(A) HBIG provides temporary protection (B) The liver is an “immunologically
19. Which of the following statements
when administered in standard doses. sheltered” site.
about HBV is false?
(B) HBIG typically is used instead of (C) Viral DNA can persist within the host
(A) Replication involves reverse
hepatitis B vaccine for postexposure cell.
transcriptase.
immunoprophylaxis to prevent HBV (D) Many humans are immunologically
(B) Infected persons may have large
infection. tolerant to HBsAg.
numbers of noninfectious viral particles
(C) No evidence exists that HBV, HCV, or
17. A 35-year-old man addicted to circulating in their bloodstream.
HIV have ever been transmitted by HBIG in
intravenous drugs has been a carrier of (C) Infection can result in cirrhosis.
the United States.
HBsAg for 10 years. He suddenly develops (D) Asymptomatic infections can last for
(D) HBIG is not used as protection against
acute fulminant hepatitis and dies within years.
HCV infection.
10 days. Which of the following laboratory (E) In the United States, the incidence of
15. Each of the following statements tests would contribute most to diagnosis? infection has been steadily increasing
concerning HAV is correct except over the past few years.
(A) Anti-HBs antibody
(A) The hepatitis A vaccine contains (B) HBeAg 1. Which of the following statements
inactivated HAV as the immunogen. (C) Anti-HBc antibody about rhinoviruses is correct?
(B) HAV commonly causes asymptomatic (D) Anti-delta virus antibody
(A) There are three antigenic types.
infection in children.
18. Each of the following statements (B) Amantadine protects against infection.
(C) The diagnosis of hepatitis A is usually
concerning HCV and HDV is correct except (C) They do not survive on environmental
made by isolating
surfaces.
(A) HCV is an RNA virus that causes
HAV in cell culture. (D) They are the most frequent causative
posttransfusion
(D) Gamma globulin is used to prevent agent of the common cold.
hepatitis A in exposed persons. hepatitis. (E) They share physicochemical similarities
(B) HDV is transmitted primarily by the with coronaviruses.
16. Which one of the following is the most
fecal–oral route
reasonable explanation 2. A 26-year-old man develops
(C) HDV is a defective virus that can
for the ability of HBV to cause chronic myopericarditis with mild congestive heart
replicate only in a cell that is also infected
infection? failure that increases over several weeks.
with HBV.
Coxsackievirus B5 infection is diagnosed.
(D) People infected with HCV commonly
Which of the following clinical syndromes
is not associated with coxsackievirus 4. Picornavirus vaccines have been used (D) The blood and lymph system
infections? for several decades in the prevention of (E) The respiratory system
human disease. Which of the following
(A) Herpangina 6. Which of the following properties of
statements is correct?
(B) Myocarditis or pericarditis enteroviruses is not shared by
(C) Aseptic meningitis (A) The live, attenuated poliovirus rhinoviruses?
(D) Acute hemorrhagic conjunctivitis vaccine produces gastrointestinal tract
(A) Single-stranded RNA genome
(E) Progressive postpolio muscle atrophy resistance.
(B) Production by cleavage of viral proteins
(B) There is an effective killed vaccine
3. A 3-month-old child develops fever, from a polyprotei precursor
against the three major types of
restlessness, and unusual crying. These are (C) Resistance to lipid solvents
rhinoviruses.
followed by apparent lethargy. Physical (D) Stability at acid pH (pH 3.0)
(C) The live, attenuated poliovirus vaccine
examination shows a normal-appearing (E) Icosahedral symmetry
induces protective immunity against the
infant who is minimally responsive to
closely related coxsackie B viruses. 7. A person with asthma has an acute
stimuli. A lumbar puncture yields
(D) None of the available echovirus exacerbation with increased lower
cerebrospinal fluid with 200 white blood
vaccines should be given to respiratory illness. A virus is recovered.
cells per microliter, predominantly
immunocompromised patients. The isolate is most likely to be which of
lymphocytes. Acute aseptic meningitis is
(E) Only the live attenuated poliovirus the following virus types?
diagnosed, probably caused by an
vaccine is currently recommended for use
enterovirus. Enteroviruses are (A) Parainfluenza virus
in the United States.
characterized by (B) Parechovirus
5. One month after school has been let out (C) Rhinovirus
(A) Latency in sensory ganglia and
for the summer, a 16-year-old girl (D) Respiratory syncytial virus
reactivation primarily in
develops fever, myalgia, and headache. An (E) Echovirus
immunocompromised patients
outbreak of an illness with similar
(B) Transmission primarily by the fecal– 8. The use of live oral polio vaccine has
symptoms caused by an echovirus is
oral route been replaced by inactivated polio vaccine
known to be occurring in the community.
(C) The presence of a DNA polymerase in many countries. Which of the following
The primary anatomic site of echovirus
enzyme is the primary reason?
multiplication in the human host is
(D) The entry of cells following binding to
the intercellular adhesion molecule-1 (A) The muscular system (A) It is more cost effective to use the
(ICAM-1) receptor (B) The central nervous system inactivated vaccine.
(E) Undergoing antigenic shift and drift (C) The alimentary tract (B) There is a greater risk of vaccine-
induced disease than wild virus-induced
disease in areas where poliovirus has (D) Members of the group are all classified immunosuppressive therapy
been eradicated. as coxsackie A and B viruses. (E) Family contacts of
(C) Only a single dose of inactivated (E) This core group determines the immunocompromised patients
vaccine is necessary compared with majority of enterovirus disease.
13. Which of the following statements
multiple doses of the oral vaccine.
11. Each of the following statements about enteroviral meningitis is true?
(D) Circulating poliovirus strains have
concerning rhinoviruses is correct except
changed and the live vaccine is no longer (A) Vaccines are generally available to
effective in many countries. (A) Rhinoviruses are one of the most protect against the disease.
frequent causes of the common cold. (B) The main symptom is muscle paralysis.
9. Outbreaks of hand-foot-and-mouth
(B) Rhinoviruses grow better at 33°C than (C) Transmission is usually by the fecal–
disease, characterized by oral ulcerations
at 37°C; thus, they tend to cause disease in oral route.
and vesicular rashes, occur and may result
the upper respiratory tract rather than the (D) The causative agents do not survive
in infant deaths. The disease is caused by
lower respiratory tract. well in the environment.
(A) Foot-and-mouth disease virus (C) Rhinoviruses are members of the (E) Recovery is rarely complete.
(B) Chickenpox virus picornavirus family and
14. The major barrier to the control of
(C) Nonpolio enteroviruses resemble poliovirus in their structure and rhinovirus upper respiratory infections by
(D) Rhinoviruses replication. immunization is
(E) Rubella virus (D) The immunity provided by the
(A) The poor local and systemic immune
rhinovirus vaccine is excellent because
10. Epidemiologic studies indicate that a response to these viruses
there is only one serotype.
core group of enteroviruses is consistently (B) The large number of rhinovirus
circulating in the United States. Which of 12. Live, attenuated oral polio vaccine serotypes
the following statements is most (OPV) and inactivated polio vaccine (IPV) (C) The side effects of the vaccine
accurate? are both available. In which one of the (D) The inability to grow the viruses in cell
following situations is the use of OPV culture
(A) Members of the core group all display
preferred?
an epidemic pattern of outbreaks of 15. Each of the following clinical
disease. (A) Routine infant vaccination syndromes is associated with infection by
(B) The group includes about half of the (B) Mass immunization programs in areas picornaviruses except
known enteroviruses. of high poliomyelitis endemicity
(A) Myocarditis or pericarditis
(C) Disease occurs predominantly in (C) Adult immunization
(B) Hepatitis
adolescents and adults. (D) Patients who are receiving
(C) Mononucleosis coliforms. The most likely source of the variation.
(D) Meningitis outbreak was (E) A double-stranded RNA genome

1. A 36-year-old man enjoyed a meal of (A) Mosquitoes or ticks, present in high 6. Because rotavirus infections can be
raw oysters. Twenty-four hours later, he numbers in the area serious, a vaccine would be beneficial.
became ill, with a sudden onset of (B) Contaminated food served at the Which of the following is most correct
vomiting, diarrhea, and headache. The party regarding a rotavirus vaccine?
most likely cause of his gastroenteritis is (C) A nearby stream used for fishing
(A) A killed human rotavirus group A
(D) A visiting parent who was developing
(A) Astrovirus vaccine is licensed for use in the United
pneumonia
(B) Hepatitis A virus (B) Live attenuated vaccines are licensed
(E) The swimming pool
(C) Norwalk virus for use in the United States.
(D) Rotavirus, group A 4. This viral gastroenteritis agent has a (C) Vaccine development is complicated by
(E) Echovirus segmented, doublestranded RNA genome rapid antigenic variation by the virus.
and a double-shelled capsid. It is a (D) Available antiviral drugs make a
2. This virus is the most important cause of
member of which virus family? vaccine unnecessary.
gastroenteritis in infants and young
(E) Vaccine development is complicated
children. It causes infections that are often (A) Adenoviridae
because the virus cannot be grown in cell
severe and may be life threatening, (B) Astroviridae
culture.
especially in infants. (C) Caliciviridae
(D) Reoviridae 7. Rotaviruses and astroviruses share a
(A) Echovirus
(E) Coronaviridae number of characteristics. Which of the
(B) Norwalk virus
following is not shared?
(C) Rotavirus, group A 5. Rotavirus and Norwalk virus are
(D) Orbivirus distinctly different viruses. (A) Multiple serotypes exist

(E) Parvovirus However, they share which one of the (B) Can cause gastroenteritis in infants and
following characteristics? children
3. An outbreak of epidemic gastroenteritis
(C) Can cause gastroenteritis in elderly
occurred at a wooded summer camp 24 (A) Fecal–oral mode of transmission.
institutionalized patients
hours after a party for visiting families. (B) They mainly cause disease in infants
(D) Live vaccine available
Some of the visiting parents also became and young children.
(E) Fecal–oral route of transmission
ill. Samples taken 2 weeks later from the (C) They induce generally mild disease in
well that was the source of drinking water young children. 8. A 20-year-old man was on a 3-week
at the camp were negative for fecal (D) Infection patterns show no seasonal tour of Italy with other college students.
One day he abruptly became ill with (D) Avoidance of local drinking water stupor. Eastern equine encephalitis is
nausea and vomiting followed 5 hours (E) Avoidance of raw oysters diagnosed. Control of this disease in
later by abdominal cramps and watery humans could be accomplished by
11. Which of the following statements
diarrhea. No fever was noted. Which of eradication of which of the following?
about noroviruses is false?
these viruses is the most likely cause of
(A) Horses
the man’s illness? (A) They cause almost half the cases of
(B) Birds
viral gastroenteritis in the United States.
(A) Calicivirus (C) Sandflies
(B) They can be responsible for epidemics
(B) Rotavirus (D) Mosquitoes
of gastroenteritis.
(C) Reovirus (E) Ticks
(C) They generally produce an illness
(D) Adenovirus
lasting 1–2 weeks. 2. An arbovirus common in the Middle
(E) Astrovirus
(D) Similar viruses are widespread among East, Africa, and Southwest Asia first
9. Which statement about rotaviral marine animals. appeared in New York in 1999. By 2002
gastroenteritis is false? (E) They typically cause disease in children the virus
and adults rather than infants.
(A) The name of the causative agent was had spread throughout the continental
suggested by its appearance. 12. Each of the following statements United States. This arbovirus, a member of
(B) Most of the estimated 600,000 deaths regarding rotaviruses is correct except the Japanese B encephalitis antigenic
occurring worldwide complex, is which of the following?
(A) The rotavirus vaccine contains
from this disease are from dehydration.
recombinant RNA polymerase as the (A) Japanese B encephalitis virus
(C) Most cases of the disease occur in
immunogen. (B) Tick-borne encephalitis virus
infants and children.
(B) Rotaviruses are a leading cause of (C) West Nile virus
(D) The causative agent infects mainly the diarrhea in young children. (D) Dengue virus
stomach. (C) Rotaviruses are transmitted primarily (E) Rift Valley fever virus
(E) The disease is transmitted by the fecal– by the fecal–oral route.
3. Which of the following descriptions of
oral route. (D) Rotaviruses belong to the reovirus
or statements about Lassa fever is
family, which has a double-stranded
10. Norwalk virus illness might be correct?
segmented RNA genome.
prevented by any of the following except
(A) It is found in eastern Africa.
1.A 74-year-old man develops fever,
(A) Avoidance of raw fruits (B) Human-to-human transmission does
alaise, and a sore throat followed shortly
(B) Live, reassortant vaccine not occur.
thereafter by nausea, vomiting, and then
(C) Careful handwashing (C) It seldom causes death or
complications. 6. Regarding the patient in Question 5, (D) All of the above
(D) It occurs from contact with the house yellow fever occurs in which region or (E) None of the above
rat Mastomys natalensis. regions of the world?
9. Hantaviruses, which are emerging
(E) There is no drug that is effective in
(A) Asia pathogens in the United States, can be
treating Lassa fever.
(B) Africa and South America described by which of the following?
4. Arboviruses are transmitted by (C) North America
(A) They are arenaviruses.
bloodsucking arthropods from one (D) Africa and Middle East
(B) They are readily transmitted human to
vertebrate host to another. Arboviruses (E) Throughout the world
human.
are found in the following virus families
7. African hemorrhagic fevers, Marburg (C) They cause influenza-like symptoms
except which of the following?
and Ebola, are severe diseases often followed rapidly by acute respiratory
(A) Togaviridae ending in death. Which of the following is failure.
(B) Flaviviridae most accurate about Ebola virus? (D) They are acquired by inhalation of
(C) Bunyaviridae aerosols of deer urine.
(A) Is spread by contact with blood or
(D) Reoviridae other body fluids (E) They show a high frequency of
(E) Arenaviridae (B) Is transmitted by mosquitoes antigenic variation.
(C) Is a flavivirus
5. A 27-year-old man develops fever, 10. A microbiologist was performing a
(D) Causes infections but no disease in
chills, headache, and backache. Four days necropsy in a laminar flow biosafety
nonhuman primates
later he develops a high fever and cabinet on a blue-jay submitted as part of
(E) Is antigenically related to Lassa fever
jaundice. Yellow fever is diagnosed. Which a state’s arbovirus surveillance program.
virus
of the following statements concerning He lacerated his thumb while using a
yellow fever is correct? 8. Which of the following groups can be scalpel to remove the bird’s brain. Four
vaccinated routinely with yellow fever days later, he developed a headache,
(A) The virus is transmitted by culicine
vaccine without special safety myalgia, and malaise followed by chills,
mosquitoes in the urban form of disease.
considerations? sweats, and lymph node swelling. Two
(B) Monkeys in the jungle are a major
days later, a rash began on his face and
reservoir of yellow fever virus. (A) Children younger than 9 months old
spread to the trunk, arms, and legs,
(C) Yellow fever often has long-term (B) Pregnant women
persisting for about 3 days. He sought
complications. (C) Persons with compromised immune
medical care and reported a history of
(D) All infections lead to apparent disease. systems
dengue fever and vaccinations with yellow
(E) Ribavirin is specific therapy.
fever and Japanese B encephalitis
vaccines. A serum sample taken the day of 12. Which of the following diseases for the causative agent.
the injury contained anti-flavivirus occurring in the United States lacks a (D) Outbreaks of the disease could occur
immunoglobulin G (IgG) antibody by known vector? in the United States because a suitable
enzyme-linked immunosorbent assay. A vector is present.
(A) Hantavirus pulmonary syndrome
serum sample collected 13 days after the (E) An attenuated vaccine is widely used to
(B) West Nile fever
onset of illness showed an increased titer prevent the disease.
(C) La Crosse encephalitis
of antiflavivirus IgG antibody and the
(D) Colorado tick fever 15. Which of the following statements
presence of West Nile virus IgM antibody.
(E) St. Louis encephalitis about hantaviruses in the United States is
The physician could conclude that the
correct?
most likely cause of the microbiologist’s 13. Each of the following statements
illness was which virus? concerning arboviruses is correct except (A) Are limited in range to southwestern
states
(A) Dengue virus (A) The pathogenesis of dengue
(B) Are carried only by deer mice
(B) Yellow fever virus hemorrhagic shock syndrome
(C) Infect human beings with a fatality
(C) West Nile virus is associated with the heterotypic
rate that can be above 30%
(D) St. Louis encephalitis anamnestic response.
(D) Were first identified in the early 1970s
(E) Not identifiable until neutralizing
(B) Wild birds are the reservoir for (E) Are contracted mainly in bat caves
antibody titers from paired sera could be
assessed against a panel of arboviruses encephalitis viruses but not for yellow
1. Which of the following statements
fever virus.
regarding the prevention and treatment of
11. Which of the following statements (C) Ticks are the main mode of
influenza is correct?
about dengue virus is not true? transmission for both encephalitis viruses
and yellow fever virus. (A) Booster doses of vaccine are not
(A) It is the most important mosquito-
(D) There is a live, attenuated vaccine that recommended.
borne viral disease affecting humans.
effectively prevents yellow fever. (B) Drugs that inhibit neuraminidase are
(B) It is distributed worldwide in tropical
active only against influenza A.
regions. 14. Which of the following statements
(C) As with some other live vaccines, the
(C) It can cause a severe hemorrhagic about yellow fever is false?
influenza vaccine should not be given to
fever.
(A) There is no animal reservoir. pregnant women.
(D) There is a single antigenic type.
(B) The name “yellow” comes from the (D) The influenza vaccine contains several
(E) One form of disease is characterized by
fact that many victims have jaundice. serotypes of virus.
increased vascular permeability.
(C) Certain mosquitoes are biological hosts
(E) The virus strains in the influenza (A) Fever (A) It results in major antigenic changes.
vaccine do not vary from year to year. (B) Muscular aches (B) It is exhibited only by influenza A
(C) Malaise viruses.
2. Which of the following statements
(D) Dry cough (C) It is caused by frameshift mutations in
about the neuraminidase of influenza virus
(E) Rash viral genes.
is not correct?
5. The type-specific antigen (A, B, or C) of (D) It results in new subtypes over time.
(A) Is embedded in the outer surface of
influenza viruses is found on which viral (E) It affects predominantly the matrix
the viral envelope
constituent? protein.
(B) Forms a spike structure composed of
four identical monomers, each with (A) Hemagglutinin 8. A 32-year-old male physician developed
enzyme activity (B) Neuraminidase a “flu-like” syndrome with fever, sore
(C) Facilitates release of virus particles (C) Nucleocapsid throat, headache, and myalgia. To provide
from infected cells (D) Polymerase complex laboratory confirmation of influenza, a
(D) Lowers the viscosity of the mucous film (E) Major nonstructural protein culture for the virus was ordered. Which
in the respiratory tract (F) Lipid in the viral envelope of the following would be the best
(E) Is antigenically similar among all specimen for isolating the virus
6. A 70-year-old nursing home patient
mammalian influenza viruses responsible for this infection?
refused the influenza vaccine and
3. Which of the following statements subsequently developed influenza. She (A) Stool
reflects the pathogenesis of influenza? died of acute pneumonia 1 week after (B) Nasopharyngeal washing
contracting the flu. The most common (C) Vesicle fluid
(A) The virus enters the host in airborne
cause of acute postinfluenza pneumonia is (D) Blood
droplets.
which of the following? (E) Saliva
(B) Viremia is common.
(C) The virus frequently establishes (A) Legionella 9. Which of the following statements
persistent infections in the lung. (B) Staphylococcus aureus about isolation of influenza viruses is
(D) Pneumonia is not associated with (C) Measles correct?
secondary bacterial infections. (D) Cytomegalovirus
(A) Diagnosis of an influenza virus
(E) Viral infection does not kill cells in the (E) Listeria
infection can only be made by isolating the
respiratory tract.
7. Which of the following statements virus.
4. Which of the following symptoms is not concerning antigenic drift in (B) Isolation of influenza virus is done
typical of influenza? influenza viruses is correct? using newborn mice.
(C) Isolation of virus can help determine diagnostic test because it is the most rapid given each year because the antigenicity
the epidemiology of the disease. assay. of the virus drifts.
(D) Primary influenza virus isolates grow (C) Patient antibody responses are highly (C) Oseltamivir is effective against both
readily in cell culture. specific for the strain influenza A and influenza B viruses.

10. The principal reservoir for the of infecting influenza virus. (D) The main antigen in the vaccine that
antigenic shift variants of influenza virus (D) Reverse transcription polymerase induces protective antibody is the
appears to be which of the following? chain reaction is preferred hemagglutinin.
for its speed, sensitivity, and specificity.
(A) Chronic human carriers of the virus 15. Each of the following statements
(B) Sewage 13. The mechanism of “antigenic drift” in concerning the antigenicity of influenza A
(C) Pigs, horses, and fowl influenza viruses includes all but one of virus is correct except
(D) Mosquitoes the following
(A) Antigenic shifts, which represent major
(E) Rodents
(A) Can involve either hemagglutinin or changes in antigenicity, occur infrequently
11. Highly pathogenic H5N1 avian neuraminidase antigens and are caused by the reassortment of
influenza (HPAI) can infect humans with a (B) Mutations caused by viral RNA segments of the viral genome.
high mortality rate, but it has not yet polymerase (B) Antigenic shifts affect both the
resulted in a pandemic. The following are (C) Can predominate under selective host hemagglutinin and the neuraminidase.
characteristics of HPAI, except for one. population immune pressures (C) The worldwide epidemics caused by
Which one is not? (D) Reassortment between human and influenza A virus are caused by antigenic
animal or avian reservoirs shifts.
(A) Efficient human-to-human
(E) Can involve genes encoding structural (D) The protein involved in antigenic drift
transmission
or nonstructural proteins is primarily the internal
(B) Presence of avian influenza genes
ribonucleoprotein.
(C) Efficient infection of domestic poultry 14. Each of the following statements
(D) Contains segmented RNA genome concerning the prevention and treatment 16. Which of the following infectious
of influenza is correct except agents is most likely to cause a pandemic?
12. Which of the following statements
about diagnostic testing for influenza is (A) The inactivated influenza vaccine (A) Influenza A virus
true? contains H1N1 virus but the live, (B) Streptococcus pyogenes
attenuated influenza vaccine contains (C) Influenza B virus
(A) Clinical symptoms reliably distinguish
H3N2 virus. (D) Respiratory syncytial virus
influenza from other respiratory illnesses.
(B) The vaccine is recommended to be (E) Influenza C virus
(B) Viral culture is the “gold standard”
1. A 4-year-old boy develops an acute (C) Influenza virus associated with congenital rubella
febrile illness. His pediatrician diagnoses (D) Metapneumovirus syndrome.
mumps. The organ most commonly (E) Measles virus (D) Only rare strains of rubella virus are
exhibiting signs of mumps is the teratogenic.
4. Several paramyxoviruses can cause
(E) None of the above
(A) Lungs pneumonia in infants or children. For
(B) Ovary which of the following paramyxoviruses is 6. A 5-year-old child develops a low-grade
(C) Parotid glands there an effective vaccine available that fever, coryza, conjunctivitis, and Koplik
(D) Skin would prevent pneumonia? spots. The physician can conclude that
(E) Testes
(A) Parainfluenza virus type 1 (A) The child has probably not been
2. The paramyxoviruses include the most (B) Measles virus successfully vaccinated with the MMR
important causes of respiratory infections (C) Respiratory syncytial virus vaccine.
in infants and young children. Which of (D) Mumps virus (B) The child’s pregnant mother is at risk
the following is not characteristic of (E) Metapneumovirus of becoming infected and her unborn child
paramyxoviruses? developing congenital abnormalities,
5. A 27-year-old woman who is 2 months’
including mental retardation.
(A) Genome is negative-sense RNA. pregnant develops fever, malaise, and
(C) A rash will soon develop on the child’s
(B) Envelope contains a glycoprotein with arthralgia. A fine maculopapular rash
face and will last only 2–3 days.
fusion activity. appears on her face, trunk, and
(D) Treatment of the child with the
(C) Paramyxoviruses do not undergo extremities. Rubella is diagnosed, and
antiviral drug ribavirin should be initiated
genetic reassortment. there is concern that the fetus will be
immediately to minimize the chance of
(D) Replication cycle occurs in cytoplasm infected, resulting in the congenital
development of acute encephalitis.
of susceptible cells. rubella syndrome. Which of the following
(E) Genome is segmented. statements about this syndrome is 7. Parainfluenza viruses are ubiquitous and
correct? cause respiratory illnesses in people of all
3. A 2-month-old infant developed a
(A) The disease can be prevented by ages. However, reinfections with
respiratory illness that the pediatrician
vaccination of school-age children with parainfluenza viruses are common
diagnosed as bronchiolitis. The most likely
measles vaccine. because
cause
(B) Congenital abnormalities occur when a (A) Many antigenic types of parainfluenza
of the disease is
nonimmune pregnant woman is infected viruses exist, and exposure to new strains
(A) Parainfluenza virus type 4 at any time during pregnancy. results in new infections.
(B) Respiratory syncytial virus (C) Deafness is a common defect (B) Infections in the respiratory tract do
not elicit a systemic immune response. (A) Measles virus detection
(C) Limited virus replication occurs, which (B) Mumps virus (D) Can provide data about genetic
fails to stimulate antibody production. (C) Parainfluenza virus type 1 variation for molecular epidemiology
(D) Secretory immunoglobulin A antibody (D) Respiratory syncytial virus studies
in the nose is short lived, disappearing a (E) Rubella virus (E) More specific assay for parainfluenza
few months after infection. viruses than serology
10. A 3-year-old girl develops an acute
8. A 20-month-old boy had an illness respiratory virus infection that requires 12. Each of the following statements
characterized by fever, irritability, hospitalization. Ribavirin therapy is concerning measles vaccine is correct
conjunctivitis, and a brick-red rash initially considered. Ribavirin is approved for except
on the face but spreading downward and treatment of which of the following
(A) The vaccine contains live, attenuated
outward. At age 9 years, the boy had a situations?
virus.
gradual onset of severe, generalized
(A) Lower respiratory tract disease (B) The vaccine should not be given at the
neurologic deterioration. Subacute
caused by respiratory syncytial virus in same time as the mumps vaccine because
sclerosing panencephalitis (SSPE) was
infants the immune system cannot respond to
diagnosed. Which of the following
(B) Congenital rubella syndrome two viral antigens given simultaneously.
statements about SSPE is correct?
(C) Aseptic meningitis caused by mumps (C) Virus in the vaccine contains only one
(A) Defective varicella-zoster virus is infection serotype.
present in brain cells. (D) Pneumonia caused by measles virus in (D) The vaccine should not be given before
(B) High titers of measles antibody are adults 15 months of age because maternal
found in cerebrospinal fluid. (E) Encephalitis related to Nipah virus antibodies can prevent an immune
(C) The incidence of the disease is rising (F) All of the above response.
since the introduction of MMR vaccine
11. Reverse transcription polymerase 13. Each of the following statements
(D) Rapidly progressive deterioration of
chain reaction (RT-PCR) assays are useful concerning rubella is correct except
brain function occurs.
in diagnosis of paramyxovirus infections.
(E) The disease is a rare, late complication (A) Congenital abnormalities occur
Which of the following statements about
of rubella infection. primarily when a pregnant woman is
RT-PCR is not accurate?
infected during the first trimester.
9. Which of the following paramyxoviruses
(A) More sensitive assay than virus
has an HN surface glycoprotein lacking (B) Women who say that they have never
isolation
hemagglutinin activity? had rubella can, nevertheless, have
(B) Can identify virus strains
neutralizing antibody in their serum.
(C) More rapid assay than antigen
(C) In a 6-year-old child, rubella is a mild, through the bloodstream (viremia) to (B) Because there is no vaccine against
self-limited disease with few various internal organs. mumps, passive immunization is the only
complications. means of preventing the disease.
(D) Acyclovir is effective in the treatment (C) The diagnosis of mumps is made on
16. Each of the following statements
of congenital rubella syndrome. clinical grounds because the virus cannot
concerning subacute sclerosing
be grown in cell culture and serologic tests
14. Each of the following statements panencephalitis is correct except
are inaccurate.
concerning rubella vaccine is correct
(A) Immunosuppression is a frequent (D) Second episodes of mumps can occur
except
predisposing factor. because there are two serotypes of the
(A) The vaccine prevents reinfection, (B) Aggregates of helical nucleocapsids are virus, and protection is type specific.
thereby limiting the spread of virulent found in infected cells.
19. Which of the following statements is
virus. (C) High titers of measles antibody are
more likely to be true of measles (rubeola)
(B) The immunogen in the vaccine is found in cerebrospinal fluid.
than German measles (rubella)?
killed rubella virus. (D) Slowly progressive deterioration of
(C) The vaccine induces antibodies that brain function occurs. (A) Koplik spots are present.
prevent dissemination of the virus by (B) It causes birth defects.
17. Which one of the following is the best
neutralizing it during the viremic stage. (C) It causes only a mild illness.
evidence on which to base a decisive
(D) The incidence of both childhood (D) Human beings are the only natural
diagnosis of acute mumps disease?
rubella and congenital rubella syndrome host.
has decreased significantly since the (A) A positive skin test result (E) Attenuated virus vaccine is available
advent of the vaccine. (B) A fourfold rise in antibody titer to for prevention.
mumps antigen
15. Each of the following statements 1. A 63-year-old woman develops fever,
(C) A history of exposure to a child with
concerning mumps is correct except headache, malaise, myalgia, and cough. It
mumps
is early in the winter respiratory virus
(A) Mumps virus is a paramyxovirus and (D) Orchitis in young adult man
season, and the patient’s physician does
hence has a singlestranded RNA genome.
18. Which one of the following statements not know what viruses are present in the
(B) Meningitis is a recognized complication
concerning mumps is correct? community. Which of the following viruses
of mumps.
is not a cause of acute respiratory disease?
(A) Although the salivary glands are the
(C) Mumps orchitis in children before
most obvious sites of infection, the (A) Influenza virus
puberty often causes sterility.
testes, ovaries, and pancreas can be (B) Adenovirus
(D) During mumps, the virus spreads
involved as well. (C) Respiratory syncytial virus
(D) Coronavirus characterized by pneumonia and (B) Contain the largest genomes among
(E) Rotavirus progressive respiratory failure. The RNA viruses
prevention or treatment of these diseases (C) Can cause gastroenteritis
2. Based on sequence analysis and
can be accomplished by (D) Are distributed worldwide
serologic assays, the most likely origin of
the SARS coronavirus is which of the (A) A subunit vaccine 7. SARS coronavirus shares some
following? (B) A cold-adapted live-attenuated vaccine characteristics, but not all, with human
(C) The antiviral drug amantadine coronavirus HCoV-OC43. Which of the
(A) Recombination between a human and
(D) Infection control measures, including following statements is true for SARS
an animal coronavirus that created a new
isolation and wearing of protective gear coronavirus?
virus
(E) The antiviral drug acyclovir
(B) Jump of an animal coronavirus into (A) Causes annual outbreaks during the
humans 5. An epidemic of acute respiratory virus winter
(C) Mutation of a human coronavirus that infections occurred among the elderly (B) Is distributed worldwide
resulted in increased virulence residents of a nursing home. Influenza (C) Populations at high risk of disease
(D) Acquisition of human cellular genes by viruses and coronaviruses, which can include health care workers
a human coronavirus via recombination cause serious respiratory disease in the (D) Natural hosts are palm civets
that allowed viral evasion of the host elderly, are suspected. Which of the
1. Rabies virus is rapidly destroyed by
immune response following
(A) Ultraviolet radiation
3. The coronavirus SARS epidemic of characteristics is shared by these viruses?
(B) Heating at 56°C for 1 hour
2002–2003 resulted in many cases and
(A) Segmented genome (C) Ether treatment
deaths. What is the primary route of
(B) Infectious RNA genome (D) Trypsin treatment
transmission of human coronaviruses?
(C) High frequency of recombination (E) All of the above
(A) Fecal–oral during replication
2. Prions are readily destroyed by
(B) Respiratory (D) Single serotype infects humans
(C) Blood (E) Negative-sense genome (A) Ionizing radiation
(D) Perinatal mother-to-infant (B) Formaldehyde
6. The following are common
(E) Sexual activity (C) Boiling
characteristics of coronaviruses, except for
(D) Proteases
4. Coronavirus infections in humans one. Which is not correct?
(E) None of the above
usually cause a common cold syndrome.
(A) Possess cross-reactive antigens with
However, a recent outbreak of SARS was
influenza viruses
3. The presence in neurons of eosinophilic Jakob disease in humans and BSE appear places medical personnel at serious risk.
cytoplasmic inclusion bodies, called Negri to be caused by the same agent. Which of (E) Bat rabies has caused most human
bodies, is characteristic of which of the the following statements is true of both rabies cases in the United States since the
following central nervous system diseases? 1990s.
infections?
(A) Immunosuppression of the host is a 7. Infectious scrapie agent can be detected
(A) Borna disease predisposing factor. in amyloid plaques in infected brains of
(B) Rabies (B) It is an immune-mediated degenerative sheep and hamsters. The genome of the
(C) Subacute sclerosing panencephalitis neurologic disorder. infectious agent is characterized by which
(D) New variant Creutzfeldt-Jakob disease (C) There is a long incubation period of the following nucleic acid types?
(E) Postvaccinal encephalitis (months to years) from time of exposure
(A) Negative-sense, single-stranded RNA
to appearance of symptoms.
4. Which of the following statements (B) Small interfering RNA, smallest known
about rabies vaccines for human use is (D) The agent is recoverable only from the infectious RNA
true? central nervous system of an infected (C) DNA copy of RNA genome, integrated
host. in mitochondrial DNA
(A) Contain live, attenuated rabies virus
(E) The interferon response persists (D) Single-stranded, circular DNA
(B) Contain multiple antigenic types of
throughout the incubation period. (E) No detectable nucleic acid
rabies virus
(F) There is a high-titer antibody response
(C) May cause allergic encephalitis when 8. A 49-year-old man visited a neurologist
toward PrPSc protein of the agent.
prepared from nerve tissue after 2 days of increasing right arm pain
(D) Can be used only for preexposure 6. Rabies virus has a wide host range and and paresthesias. The neurologist
prophylaxis the ability to infect all warm-blooded diagnosed an atypical neuropathy. The
(E) Duck embryo vaccine is highly antigenic animals, including humans. Which symptoms increased and were
and only a single dose is needed statement about the epidemiology of accompanied by hand spasms and
human rabies is true? sweating on the right side of the face and
5. A 22-year-old man is a resident of a
trunk. The patient was admitted to the
small town near London. He likes to eat (A) Africa accounts for the majority of
hospital the day after developing
beefsteak. He develops a severe rabies fatalities.
dysphagia, hypersalivation, agitation, and
progressive neurologic disease (B) Dog bites cause most cases of human
generalized muscle twitching. Vital signs
characterized by psychiatric symptoms, rabies in England.
and blood tests were normal, but within
cerebellar signs, and dementia. Probable (C) Domestic animals are the source of
hours the patient became confused. The
bovine spongiform encephalopathy (BSE) most human rabies in the United States.
consulting neurologist suspected rabies.
is diagnosed. New variant Creutzfeldt- (D) Human-to-human rabies transmission
Rabies immune globulin, vaccine, and (D) Swine (A) A histologic picture of spongiform
acyclovir were administered. The patient (E) Rats encephalopathy
was placed on mechanical ventilation the (B) Transmissibility to animals associated
10. A runner reports an “unprovoked bite”
following day. Renal failure developed, with a long incubation period
from a neighborhood dog. The dog was
and the patient died 3 days later. Rabies (C) Slowly progressive deterioration of
captured by local animal control
test results were positive. The patient’s brain function
authorities, and it appears healthy. What
wife reported the patient had suffered no (D) Prominent intranuclear inclusions in
is the appropriate action?
bites by dogs or wild animals. The most oligodendrocytes
likely explanation for treatment failure is (A) Confine and observe the dog for 10
13. A 5-year-old boy in San Francisco
days for signs suggestive of rabies.
(A) The rabies test results were falsely reaches into a car to pet another family’s
(B) Begin postexposure prophylaxis of the
positive and the patient did not have dog and is bitten on the finger. Six weeks
bitten person.
rabies. after the bite, the child develops fever,
(C) Immediately euthanize the dog.
(B) Treatment was initiated after the headache, and a seizure. He becomes
(D) Because canine rabies has been
onset of clinical symptoms of rabies. combative and hallucinates. What is the
eliminated in the United
(C) The vaccine was directed against dog best diagnostic test to perform on the
rabies and the patient was infected with States, dog bites are no longer an patient to rule in rabies as a cause of his
bat rabies. indication for postexposure prophylaxis, illness?
(D) The rabies immune globulin should not and no further action is needed.
(A) Detection of serum antirabies antibody
have been administered as it interfered (E) Test the dog for rabies antibody.
(B) Culture of cerebrospinal fluid for virus
with the vaccine.
11. The slow virus disease that most (C) Direct fluorescent antibody stain of a
(E) Interferons—and not the treatment
clearly has immunosuppression as an biopsy from the nape of the neck
regimen administered—are the treatment
important factor in its pathogenesis is (D) Brain biopsy
of choice once rabies symptoms develop.
(E) Cerebrospinal fluid antirabies antibody
9. Which of the following animals is most (A) Progressive multifocal
leukoencephalopathy 14. Each of the following statements
commonly reported rabid in the United
(B) Subacute sclerosing panencephalitis concerning rabies and rabies virus is
States?
(C) Creutzfeldt-Jakob disease correct except
(A) Squirrels (D) Scrapie
(A) The virus has a lipoprotein envelope
(B) Raccoons
12. Scrapie and kuru possess all of the and single-stranded RNA as its genome.
(C) Rabbits
following characteristics except (B) The virus has a single antigenic type
(serotype).
(C) In the United States, dogs are the time of virus infection and tumor (D) Polyomaviruses
most common reservoir. appearance. (E) Herpesviruses
(D) The incubation period is usually long (D) Animal models seldom predict cellular
4. Retroviruses encode an enzyme called
(several weeks) rather than short (several mechanisms in human cancer.
reverse transcriptase.
days).
(E) Host factors are insignificant in
The function of the reverse transcriptase
15. A 20-year-old man, who for many influencing the development of virus-
enzyme is
years had received daily injections of induced human cancer.
growth hormone prepared from human (A) DNase activity
2. Cellular oncogenes represent activated
pituitary glands, develops ataxia, slurred (B) RNA-dependent DNA polymerase
genes involved in cancer. A second class of
speech, and dementia. At autopsy the activity
cancer genes is involved in cancer
brain shows widespread neuronal (C) DNA-dependent RNA polymerase
development only when both alleles of
degeneration, a spongy appearance due to activity
such a gene are inactivated.
many vacuoles between the cells, (D) RNA-dependent RNA polymerase
The second class of genes is called activity
no inflammation, and no evidence of virus
(E) Topoisomerase activity
particles. The most likely diagnosis is (A) Proto-oncogenes
(B) T antigen genes 5. Two months after a kidney transplant, a
(A) Herpes encephalitis
(C) Tumor suppressor genes 47-year-old man developed nephropathy.
(B) Creutzfeldt-Jakob disease
(D) Transduced genes Up to 5% of renal allograft recipients
(C) Subacute sclerosing panencephalitis
(E) Silent genes develop nephropathy. A viral cause of
(D) Progressive multifocal
some of the nephropathy cases has been
leukoencephalopathy 3. A 38-year-old woman with many
identified as
(E) Rabies lifetime sexual partners is diagnosed with
cervical cancer. This cancer is common (A) Polyomavirus BK
1. Viruses can cause cancer in animals and
worldwide and has a sexually transmitted (B) Human papillomavirus, all types
humans. A principle of viral carcinogenesis
viral etiology. The causative agent of (C) Human papillomavirus, low-risk types
is that
human cervical cancer is (D) Hepatitis C virus
(A) Retroviruses cause most types of (E) Human cytomegalovirus
(A) Hepatitis C virus
human cancer.
(B) Hepatitis B virus 6. Human papillomavirus can cause cancer
(B) Not all infections with a human cancer
(C) Human papillomaviruses, high-risk in humans and is most commonly
virus lead to tumor formation.
types associated with
(C) Short latent periods elapse between
(A) Rectal polyps contains a human cancer virus with an 12. Which of the following best describes
(B) Breast cancer RNA genome available HPV vaccines?
(C) Prostate cancer
(A) Adenoviridae (A) Live attenuated virus
(D) Anogenital cancers
(B) Herpesviridae (B) Live recombinant virus
(E) Mesotheliomas
(C) Hepadnaviridae (C) Noninfectious subunit
7. A virus that causes human cancer is also (D) Papillomaviridae (D) Toxoid
associated with a nervous system disorder
(E) Flaviviridae 13. Many of the oncogenic retroviruses
called tropical spastic paraparesis. That
carry oncogenes closely related to normal
virus is 10. Laryngeal papillomas in children are
cellular genes, called proto-oncogenes.
generally caused by the same viruses that
(A) Polyomavirus JC Which one of the following statements
cause benign genital condylomas. These
(B) Polyomavirus SV40 concerning proto-oncogenes is incorrect?
viruses are
(C) Herpes simplex virus
(A) Several proto-oncogenes have been
(D) Human T-lymphotropic virus (A) Papillomaviruses, types 6 and 11
found in mutant form in human cancers
(E) Human immunodeficiency virus (B) Polyomavirus JC
that lack evidence for viral etiology.
(C) Epstein-Barr virus
8. The polyomaviruses encode (B) Several viral oncogenes and their
(D) Molluscum contagiosum virus
oncoproteins called T antigens. These viral progenitor protooncogenes encode
(E) Papillomaviruses, types 16 and 18
gene products protein kinases specific for tyrosine.
11. Vaccines against the most common (C) Some proto-oncogenes encode cellular
(A) Are not needed for virus replication
HPV types that cause genital infections growth factors and receptors for growth
(B) Interact with cellular tumor
were approved in 2006 and 2007. They are factors.
suppressor proteins
aimed for use in which of the following (D) Proto-oncogenes are closely related
(C) Function to integrate the viral provirus
population(s)? to transposons found in bacteria.
into the cellular chromosome
(D) Mutate rapidly to allow the virus to (A) All adults, both men and women 1. HIV-1 is classified as a member of the
escape immune clearance by the host (B) All female adults Lentivirus genus in the Retroviridae family.
(E) Are not able to transform cells in (C) Women with precancerous cervical Lentiviruses
culture lesions
(A) Contain a DNA genome
(D) All adolescents and young adults,
9. Cancer viruses are classified in several (B) Cause tumors in mice
both boys and girls
virus families. The following virus family (C) Infect cells of the immune system
(E) Adolescent and young adult females
(D) Have related sequences endogenous in between the time of primary HIV infection (D) A teacher with an HIV-positive child in
normal cells and the development of AIDS. During this her classroom
(E) Cause rapidly progressive neurologic period of clinical latency (E) A baseball player whose teammate is
disease HIV-positive
(A) HIV is not detectable in the plasma
2. HIV-1 encodes an envelope (B) CD4 cell counts remain unchanged 7. A 36-year-old nurse suffered a
glycoprotein, gp120. This protein (C) Virus cannot be transmitted to others needlestick with blood from an HIV-
(D) Virus is present in lymphoid organs positive patient. Six months later, the
(A) Causes membrane fusion
(E) Neutralizing antibodies are not elicited nurse’s serum was positive in an EIA test,
(B) Binds to the viral coreceptor on the
gave equivocal results in a repeat EIA test,
cell surface 5. Viral coinfections occur in HIV-1-
and was negative by Western blot. The
(C) Is highly conserved among different infected individuals and may contribute to
nurse
isolates morbidity and mortality. The most
(D) Fails to elicit neutralizing antibody common coinfection in HIV-1-positive (A) Is probably infected with HIV
(E) Induces chemokine production persons in the United States involves (B) Is in the window between acute
infection with HIV and seroconversion
3. HIV/AIDS has become a worldwide (A) Hepatitis C virus
(C) Is probably not infected with HIV
epidemic that continues to expand. In (B) Hepatitis D virus
(D) May be infected with a drug-resistant
2009, the geographic area that had the (C) HIV type 2
strain of HIV
largest number of HIV-infected people (D) Human T-lymphotropic virus
(E) May be a long-term nonprogressor
after sub-Saharan Africa was (E) Kaposi sarcoma herpesvirus
8. A 41-year-old HIV-infected male who
(A) Central and South America and the 6. Which of the following individuals may
had refused antiretroviral therapy is
Caribbean be at increased risk of acquiring an HIV
diagnosed with Pneumocystis jiroveci
(B) East Asia, including China infection?
infection. This patient
(C) North America
(A) A grandmother living in the same
(D) South/Southeast Asia (A) Probably has a CD4 T-cell count below
household with a relative who is HIV-
(E) Eastern Europe and Central Asia 200 cells/Μl
positive
(B) Is at elevated risk for lung cancer
4. The typical course of an untreated HIV (B) A tourist in Botswana who has sex
(C) Has a life expectancy of about 5 years
infection extends over 10 or more years. with a prostitute
(D) Probably has declining levels of plasma
There is usually a long period (clinical (C) A receptionist at an AIDS clinic in a
viremia
latency) hospital
(E) Is unlikely to develop dementia at this include all of the following except is evidence that an individual is protected
stage against HIVinduced disease.
(A) Blood
9. A 48-year-old HIV-positive man with a (B) Saliva visibly contaminated with blood 14. Highly active antiretroviral therapy
CD4 count of 40 complains of memory loss (C) Urine not visibly contaminated with (HAART) is less than ideal because
to his doctor. Four months later, he blood
(A) It does not eliminate latent HIV
becomes paralyzed and dies. An autopsy (D) Genital secretions
infection
reveals demyelination of many neurons in (E) Amniotic fluid
(B) Its cost is too great for 90% of AIDS
the brain, and electron microscopy shows
12. Of the more than 1 million persons sufferers
clusters of nonenveloped viral particles in
estimated to be living with HIV in the (C) It often has severe side effects
the neurons. The most likely cause of the
United States in 2011, how many are (D) Some HIV strains are resistant to it
disease is
thought to be unaware of their infection? (E) All of the above
(A) Adenovirus type 12
(A) About 5% 15. Each of the following statements
(B) Coxsackievirus B2
(B) About 10% concerning HIV is correct except
(C) Parvovirus B19
(C) About 20%
(D) Epstein-Barr virus (A) The CD4 protein on the T-cell surface is
(D) About 25%
(E) JC virus one of the receptors for the virus.
(E) About 30%
(B) There is appreciable antigenic diversity
10. Highly active antiretroviral (F) About 50%
in the envelope glycoprotein of the virus.
combination therapy for HIV infection
13. Each of the following statements (C) One of the viral genes codes for a
usually includes a protease inhibitor such
concerning HIV is correct except protein that augments the activity of the
as saquinavir. Such a protease inhibitor
viral transcriptional promoter.
(A) Screening tests for antibodies are
(A) Is effective against HIV-1 but not HIV-2 (D) A major problem with testing for
useful to prevent transmission of HIV
(B) Seldom gives rise to resistant mutants antibody to the virus is its cross-reactivity
through transfused blood.
of HIV with human T-lymphotropic virus type 1.
(B) The opportunistic infections seen in
(C) Inhibits a late step in virus replication
AIDS are primarily the result of a loss of 1. An outbreak of mild intestinal distress,
(D) Degrades the CD4 receptor on cells
cell-mediated immunity. sleeplessness, perianal itching, and anxiety
(E) Interferes with virus interaction with
(C) Zidovudine (azidothymidine) inhibits has broken out among preschool children
coreceptor
the RNA-dependent DNA polymerase. in a private home. The most likely cause of
11. In a person with HIV infection, (D) The presence of circulating antibodies this condition is
potentially infectious fluids that neutralize HIV
(A) Trichomonas vaginalis leishmanin skin test (Montenegro B: Histologic section of a hydatid cyst
(B) Enterobius vermicularis reaction). Mild fevers occur irregularly. showing several protoscolices
(C) Ascaris lumbricoides The most likely parasitic disease is
(arrows) within a brood capsule. (Used
(D) Necator americanus
(A) Malaria with permission from
(E) Entamoeba histolytica
(B) Cutaneous leishmaniasis
Sullivan J., A Color Atlas of Parasitology,
2. Chagas disease is especially feared in (C) Visceral leishmaniasis
8th ed., 2009.)
Latin America because of its threat to the (D) Trypanosomiasis
heart and parasympathetic nervous (E) Filariasis Similar to the beef and pork tapeworms, a
system and the lack of an effective drug larva hatches from the egg, penetrates the
4. A sexually active 24-year-old woman
for the symptomatic later stages. Your gut, and migrates to various tissues,
complains of vaginal itching and a
patient is planning to reside in a especially liver, spleen (Figure 46-24A),
malodorous purulent vaginal discharge. To
Venezuelan village for 1–2 years. Which muscle, and brain. Instead of a cysticercus
verify your tentative diagnosis of
one of the following suggestions would be developing, as in the case of the beef and
trichomoniasis, you should include which
of special value for avoiding Chagas pork tapeworms, the larva of
of the following in your workup?
disease? Echinococcus develops into a fluid-filled
(A) Specific serologic test cyst called a hydatid cyst. The cyst
(A) Boil or treat all of your drinking water.
(B) Ova and parasite fecal smear contains germinal epithelium in which
(B) Sleep under a bed net.
(C) Wet mount of vaginal fluid thousands of future larvae (called
(C) Do not keep domestic pets in your
(D) Enzyme-linked immunoassay (ELISA) protoscolices) develop (Figure 46-24B).
house.
test of serum Inside the hydatid cyst, the protoscolices
(D) Never walk barefoot in the village
(E) Stool culture are contained within brood capsules. If the
compound.
hydatid cyst ruptures, the brood capsules
(E) Do not eat lettuce or other raw 5. You are working in a rural medical clinic
can
vegetables or unpeeled fruit. in China and a 3-yearold girl is brought in
by her mother. The child appears spill out of the cyst, metastasize to other
3. A 32-year-old male Peace Corps
emaciated A B sites, and develop into a hydatid cyst.
volunteer recently returned from a 2-year
Thus, ingestion of a single egg can give rise
period working in a war zone in the Figure 46-24 Echinococcus granulosus. A:
to several hydatid cysts, each containing
southern Sudan in Central Africa. He A 14-cm hydatid cyst from a
several brood capsules. Humans are
presents with marked splenic splenectomized patient. (Courtesy of
infected only by ingesting Echinococcus
enlargement, nonspecific Pathology, UCSF.)
eggs from dog feces. The dog, in turn, can
hypergammaglobulinemia, and a negative
acquire the infection and, upon testing, is
found to have a hemoglobin level of 5 of the following is an important 10. Several Papua New Guinea villagers
g/dL. Her feet and ankles are swollen, and consideration for the diagnosis? known to eat pork during
there is an extensive rash on her feet, celebrations were reported to be suffering
(A) Exposure to high-level UV radiation
ankles, and knees. The most likely parasitic from an outbreak of epileptiform seizures.
(B) The source and purification of water
infection that causes the child’s condition One of the first things you should
(C) The use of insect repellents while
is investigate is
hiking
(A) Schistosomiasis (D) The presence of domestic animals en (A) The prevalence of Ascaris infections in
(B) Cercarial dermatitis route the population
(C) Cyclosporiasis (E) The degree of contact with villagers en (B) The presence of Toxoplasma gondii in
(D) Hookworm infection route cats
(E) Trichuriasis (C) The presence of Trypanosoma brucei
8. Which one of the following diagnostic
(F) Ascariasis gambiense in the villagers
tests should be conducted for the patient
(D) The presence of Taenia eggs in the
6. Pathologic effects of filariae in humans in Question 7?
drinking water
are caused by the adult worms in all but
(A) Blood and urine bacteriologic (E) The presence of adult Taenia solium in
one species. In this case, the principal
examination the pigs
damage is caused by the microfilariae of
(B) Series of ova and parasite tests and
11. A 32-year-old male tourist traveled to
(A) Brugia malayi fecal smears
Senegal for 1 month. During the trip, he
(B) Mansonella ozzardi (C) ELISA or hemagglutination serologic
swam in the Gambia river. Two months
(C) Dracunculus medinensis tests for malaria
after
(D) Wuchereria bancrofti (D) Skin snip microfilarial test
(E) Onchocerca volvulus (E) Endoscopic exam for whipworms his return, he began complaining of
intermittent lower abdominal pain with
7. An 18-year-old male complains of 9. The parasite most likely to be
dysuria. Laboratory results of ova and
abdominal pain, bloating, frequent loose responsible for the illness of the patient in
parasites revealed eggs with a terminal
stools, and loss of energy. He returned a Question 7 is
spine. Which of the following parasites is
month ago from a 3-week hiking and
(A) Entamoeba coli the cause of the patient’s symptoms?
camping trek to the Mount Everest Base
(B) Plasmodium vivax
Camp in Nepal. The trek involved only (A) Toxoplasma gondii
(C) Trichomonas vaginalis
high-elevation hiking, since he flew in and (B) Schistosoma mansoni
(D) Naegleria gruberi
out of the 12,000-ft starting point. Which (C) Schistosoma haematobium
(E) Giardia lamblia
(D) Ascaris lumbricoides (A) Ingesting cysts from fecally malaria and indeed find red blood cell
(E) Taenia solium contaminated drinking water parasites in a thin smear of finger-stick
(B) Eating improperly cooked fish blood. To rule out the dangerous
12. What type of specimen was collected
(C) Eating improperly cooked beef falciparum form of malaria, which one of
for laboratory analysis based on the
(D) Walking barefoot in the park the following choices would fit a diagnosis
answer in the previous question?
(E) Engaging in unprotected sexual of Plasmodium falciparum malaria based
(A) Thick blood smear intercourse on a microscopic examination of the blood
(B) Stool sample (F) Getting bitten by a sandfly smear?
(C) Urine sample (G) Plunging into a natural hot spring
(A) Numerous large ovoid parasites in
(D) Blood for serology
15. A 37-year-old sheep farmer from some of the red cells
(E) Sputum sample
Australia presents with upper right (B) Enlarged, somewhat misshapen
13. A previously healthy 23-year-old quadrant pain and appears slightly parasitized red cells
woman recently returned from her jaundiced. A stool exam was negative for (C) Dividing parasites (schizonts) in red
vacation after visiting friends in Arizona. ova and parasites but a CT scan of the liver cells with 8–12
She complained of severe headaches, saw reveals a large 14-cm cyst that appears to progeny
“flashing lights,” and had a purulent nasal contain fluid. Which of the following (D) Dividing parasites in red cells with 16–
discharge. She was admitted into the parasites should be considered? 24 progeny
hospital with a diagnosis of bacterial (E) Double ring forms found in the smear
(A) Toxoplasma gondii
meningitis and died 5 days later. Which of
(B) Taenia solium 17. Given a diagnosis of Plasmodium
the following parasites should have been
(C) Taenia saginata falciparum malaria for the
considered
(D) Clonorchis sinensis patient in Question 16, which one of the
in the diagnosis? She had no prior history
(E) Schistosoma mansoni following treatment regimens is
of travel outside ofthe United States.
(F) Echinococcus granulosus appropriate?
(A) Plasmodium falciparum (G) Paragonimus westermani
(A) Oral proguanil plus atovaquone
(B) Toxoplasma gondii
16. An apparently run-down but alert 38- (B) Oral chloroquine
(C) Strongyloides stercoralis
year-old woman has spent 6 months as a (C) Intravenous chloroquine
(D) Entamoeba histolytica
teacher in a rural Thailand village school. (D) Oral proguanil
(E) Naegleria fowleri
Her chief complaints include frequent (E) Intravenous quinidine
14. How could the person have acquired headaches, occasional nausea and
the parasite in Question 13? vomiting, and periodic fever. You suspect
18. Given a diagnosis of Plasmodium (D) Bloodstream parasites reinvade the fever, and blood cultures were done.
falciparum, you should tell the patient in liver and induce a more severe disease Which of the following scenarios suggests
Question 16 that (select one) state. that the positive blood cultures resulted
(E) Misshapen infected red cells adhere to from a contaminant?
(A) There is little chance of a relapse in 1–
the interior lining of blood vessels and
3 years. (A) Two positive peripheral vein blood
block blood flow through these vessels.
(B) There is a strong likelihood that cultures with Staphylococcus aureus
resistance will necessitate additional 20. A 52-year-old male, returning from a (B) Two positive peripheral vein blood
courses of treatment. travel tour in India and Southeast Asia, cultures with Staphylococcus epidermidis
(C) Returning to the tropics would be was diagnosed with intestinal amebiasis along with two positive central line blood
dangerous because hypersensitivity to the and successfully treated with iodoquinol. cultures with Staphylococcus epidermidis
parasite may have developed. A month later, he returned to the clinic (C) One positive peripheral vein blood
(D) She must avoid mosquito bites in this complaining of the following conditions. culture and one positive central line blood
country, as they Which of the following conditions is most culture with Escherichia coli
might induce a relapse of malaria. likely the result of systemic amebiasis (D) One positive central venous line blood
(E) A course of primaquine to clear the (even though the intestinal infection culture with a Corynebacterium species
liver of hypnozoites would prevent appears to be cured)? and two negative peripheral vein blood
recurrence of falciparum malaria. cultures
(A) High periodic fever
(E) Two positive central line blood cultures
19. The seriousness of Plasmodium (B) Bloody urine
with Candida albicans
falciparum infection compared with the (C) Tender, enlarged liver
other three forms of malaria is due to (D) Draining skin lesion 2. Two days ago a 22-year-old man
which one of the (E) Enlarged painful spleen returned from a 2-week trip to

following? 1. A 47-year-old woman had a bone Mexico. Within 24 hours he developed


marrow transplant as part of her diarrhea. Which of the following will not
(A) Destruction of white blood cells
treatment for chronic myelogenous establish the etiology of his diarrhea?
compromises the immune reaction against
leukemia. While in the hospital she had a
malaria. (A) Stool culture for Salmonella, Shigella,
central venous catheter in place for
(B) Stem cells in the marrow are largely and Campylobacter
administration of fluids. In the time
destroyed. (B) Stool culture for rotavirus and
following the transplant, but before it had
(C) Extensive damage to the liver can Norwalk-like virus
engrafted, the patient had a very low
occur during the preerythrocytic phase of (C) Stool enzyme immunoassay for Giardia
white blood cell count. She developed a
the parasite cycle. lamblia antigen
(D) Stool examination for Entamoeba (E) Genotyping of his HIV-1 isolate to 7. A 45-year-old woman is admitted to the
histolytica determine its antiretroviral susceptibility hospital because of fever, a 6-kg weight
loss, and a new heart murmur. Probable
3. A 37-year-old man traveled to Peru 5. A 2-year-old child develops diarrhea.
endocarditis is diagnosed. How many
during the time of the cholera epidemic. Rotavirus infection is suspected. Which of
blood cultures over what period of time
One day after returning home he the following would be most useful in
should be done to provide evidence of
developed severe watery diarrhea. To diagnosing a rotavirus infection?
specific bacterial infection in endocarditis?
enhance the isolation of Vibrio cholerae
(A) Fluorescent antibody staining of the
from his stool, the laboratory needs to (A) One
stool specimen
include (B) Two over 10 minutes
(B) Light microscopy to detect mucosal
(C) Three over 2 hours
(A) MacConkey agar cells with cytopathic effect
(D) Three over 24 hours
(B) Campylobacter blood agar (C) Detection of virus antigen in stool by
(E) Six over 3 days
(C) Thiosulfate citrate bile salts sucrose enzyme-linked immunosorbent assay
agar (D) Virus culture 8. A 4-year-old boy develops bloody
(D) Bismuth sulfite agar diarrhea. Hemorrhagic colitis due to
6. Which of the following is appropriate to
(E) Hektoen agar Escherichia coli O157:H7 is suspected.
determine the etiologic diagnosis of
What medium should be inoculated to
4. A 42-year-old man is known to have infection?
help the laboratory staff make the
HIV/AIDS. Which of the following is the
(A) Culture and identification of the agent diagnosis of this infection?
most appropriate method to follow the
(B) DNA-DNA or DNA-RNA hybridization to
progress (A) Blood agar
detect pathogenspecific genes in patients’
(B) Sorbitol MacConkey agar
of his highly active antiretroviral therapy specimens
(HAART)? (C) Hektoen enteric agar
(C) Demonstration of a meaningful
(D) CIN (cefsulodin, irgasan, novobiocin)
(A) Determination of viral load antibody or cell-mediated immune
agar
(B) Following anti-HIV-1 antibody levels response to an infectious agent
(E) Thiosulfate citrate bile salts sucrose
(C) Using Western blot to assess his anti- (D) Morphologic identification of the agent
agar
p24 levels in stains of specimens or sections of
(D) Repeated culture of his blood for HIV-1 tissues by light or electron microscopy 9. A 43-year-old black man frequently
to determine when the culture becomes (E) Detection of antigen from the agent by drove his long-haul 18-wheel truck
negative immunologic assay through the Central Valley of California.
(F) All of the above Two months ago there was a major
windstorm while he was driving through 10. A 5-year-old kidney transplant patient protein of 60 μg/dL, glucose of 40 μg/dL,
the Valley. Two weeks after that he being treated with cyclosporine develops a and a total of 200 WBCs, predominately
developed fever with a cough and pleuritic lymphoproliferative disorder. Which of the mononuclear. The most likely cause of this
chest pain. An infiltratewas seen on chest following viruses is most likely responsible child’s infection is:
x-ray. Pneumonia was diagnosed, and the for this disorder?
(A) Bacterial§
patient was given erythromycin. The fever,
(A) Cytomegalovirus (B) Viral
cough, pleuritic pain, and infiltrate cleared
(B) Herpes simplex virus (C) Protozoan
over a 3-week period. Two weeks ago, he
(C) Coxsackie B virus (D) Fungal
developed progressively severe
(D) Hepatitis B virus (E) Mycobacterial
headaches, and for the past 2 days he has
(E) Epstein-Barr virus
had vomiting. His cerebrospinal fluid 13. In the case above, the most useful test
contains 150 white blood cells per 11. All of the following are appropriate for making a rapid definitive diagnosis of
microliter, predominantly lymphocytes, indications for the use of serologic tests the most likely causative agent is:
and the glucose concentration is low. for viruses except:
Meningitis due to Coccidioides immitis is (A) An antigen test for Streptococcus
suspected. Which of the following tests is (A) As an indication of one’s susceptibility pneumonia
the most sensitive and useful in to a particular viral infection (B) A latex agglutination test for
confirming this diagnosis? (B) For diagnosis when the virus has a long cryptococcal antigen
incubation period (C) A nucleic acid amplification test for
(A) Latex agglutination assay for (C) For screening purposes viral RNA detection
coccidioidal antibodies performed on CSF (D) For confirmation of a viral infection
(D) Culture on selective media combined
(B) Complement fixation test of (E) To monitor the response to treatment with a probe test for confirmation
cerebrospinal fluid for antibodies against (E) Giemsa-stained smear of cerebrospinal
Coccidioides immitis 12. In August, a 2-year-old boy presents
fluid
(C) Immunodiffusion test of cerebrospinal acutely with fever, signs of headache,
fluid for antibodies against Coccidioides decreased mental status, and neck 14. Susceptibility testing using an MIC
immitis stiffness. On physical examination the method, as opposed to disk diffusion, is
(D) Cerebrospinal fluid culture for fever is confirmed, mild nuchal rigidity is preferred for all of the following types of
Coccidioides immitis present, and although the child is irritable infections except:
(E) Complement fixation test of serum for and mildly somnolent, he is arousable and
(A) Urinary tract infections
antibodies against Coccidioides immitis is taking some oral fluids. The
(B) Endocarditis
cerebrospinal fluid parameters reveal a
(C) Osteomyelitis
(D) Bacteremia in a neutropenic patient
(E) Bacterial meningitis

15. Bacterial vaginosis is best diagnosed by


all of the follow except:
(A) Measurement of the vaginal pH
(B) Detection of a fishy odor when the
discharge is alkalinized with KOH
(C) Bacterial culture for aerobes and
anaerobes
(D) Examination of a Gram-stained smear
for “clue cells”

You might also like